Emergency Medicine Trauma

Pataasin ang iyong marka sa homework at exams ngayon gamit ang Quizwiz!

One Step Further Question: Is it safe to transport patients via helicopter if they have a small pneumothorax?

Answer: Yes.

One Step Further Question: What is the definitive treatment of an Achilles tendon rupture?

Answer: Similar results are seen with serial casting and surgical repair.

One Step Further Question: Which spontaneous pneumothoracies can be treated with observation and repeat assessment?

Answer: Small (< 20%), minimally symptomatic, and patient not mechanically ventilated.

One Step Further Question: What is the most common mechanism of trauma that causes urethral injuries?

Answer: Straddle injuries.

One Step Further Question: What are the muscles that comprise the rotator cuff?

Answer: Subscapularis, supraspinatus, infraspinatus, teres minor

One Step Further Question: What is the treatment for flail chest?

Answer: Supportive: analgesia and respiratory support. Wrapping the chest is not recommended.

One Step Further Question: The greater tuberosity is the insertion point of which 3 rotator cuff muscles?

Answer: Supraspinatus, infraspinatus and teres minor.

One Step Further Question: Which patients with rib fractures should be admitted?

Answer: Patients with flail chest, more than three rib fractures and older patients or those with multiple comorbidities. These patients are at increased risk of developing pneumonia after rib fractures.

One Step Further Question: What is the most common otologic sequela of lightning injuries?

Answer: Perforated tympanic membrane.

One Step Further Question: What is the "lover's triad"?

Answer: Name associated with injuries that may occur after jumping from an upper floor window and includes (1) calcaneal fracture, (2) vertebral compression fracture, (3) forearm fracture.

One Step Further Question: Which tissue has the highest conductive capacity?

Answer: Nerves.

One Step Further Question: What imaging modality can show abnormalities in patients with concussion in the acute setting?

Answer: Positron emission tomography (PET) scan.

One Step Further Question: Which type of hip dislocation is most common?

Answer: Posterior dislocations are most common (80-90%).

One Step Further Question: What is the other name of the pseudo-Jones fracture?

Answer: Tennis (or dancer's) fracture.

One Step Further Question: What temporary maneuver can be performed in patients with an unstable pelvis?

Answer: Wrap the pelvis over the greater trochanters with a bed sheet.

One Step Further Question: What are the traditional four classes of blast injuries?

Answer: Primary (tissue damage directly from the blast overpressure), Secondary (due to flying debris), Tertiary (blast causing victim displacement i.e., victim thrown), Quaternary (environmental hazards caused by explosion).

One Step Further Question: Relative to a native hip, does a prosthetic hip require more or less force to induce dislocation?

Answer: Prosthetic hip joints, unlike native hips, can dislocate with minimal trauma.

One Step Further Question: What interval should be recommended for "return to play" in athletes after an uncomplicated concussion?

Answer: No specific interval post-injury has been defined, but it is generally accepted that athletes should be asymptomatic for at least one week. At that time, he or she should be re-evaluated to ensure safe return to sports.

One Step Further Question: What are the other signs of a basilar skull fracture?

Answer: Raccoon eyes and battle signs

One Step Further Question: What is the Parkland Formula?

Answer: 4 x weight in kg x %TBSA burned (excluding first-degree burns)

One Step Further Question: What is the RBC/HPF cut-off used in pediatrics for patient discharged after blunt injury?

Answer: 50 RBC/HPF.

One Step Further Question: What is the survival rate of Emergency Department thoracotomies?

Answer: 7.4% overall.

One Step Further Question: What percentage of patients with abruption experience vaginal bleeding?

Answer: 70%.

One Step Further Question: What imaging study is required for patients with hemotympanum?

Answer: A CT scan of the temporal bone.

One Step Further Question: What is a "kissing burn"?

Answer: A burn that occurs at the flexor creases of the extremities. It is due to current flowing across opposing skin surfaces as the extremity is forced into flexion by an electric shock.

One Step Further Question: What is a subdural hygroma?

Answer: A collection of clear, xanthochromic blood-tinged fluid in the dural space.

One Step Further Question: Which artery is at risk for injury in a Smith's fracture (reverse Colles')?

Answer: Radial artery.

One Step Further Question: When should a hanging cast be placed in a humerus fracture as opposed to a sugar tong splint?

Answer: A hanging cast is used for humerus fractures that are grossly displaced or comminuted.

One Step Further Question: What is the initial diagnostic tool used to diagnose a subdural hematoma?

Answer: A non-contrast computed tomography scan of the head.

One Step Further Question: What is a subluxed tooth?

Answer: A tooth loosened in its socket as a result of a force.

One Step Further Question: What is the most common cause of an acute hemarthroses after a sports-related knee injury?

Answer: ACL tear.

One Step Further Question: What is the primary concern with an expanding neck hematoma?

Answer: Airway compromise and difficult intubation.

One Step Further Question: What is luxatio erecta?

Answer: An inferior glenohumeral dislocation. It is the least common type of shoulder dislocation and associated with axillary artery injury.

One Step Further Question: Which incomplete cord syndrome has the poorest prognosis?

Answer: Anterior cord syndrome; most patients do not regain any motor function.

One Step Further Question: What is the most commonly sprained ankle ligament?

Answer: Anterior talofibular ligament.

One Step Further Question: What vascular structure is at risk of injury with a proximal fibular fracture?

Answer: Anterior tibial artery.

One Step Further Question: What defines a positive diagnostic peritoneal aspiration?

Answer: Aspiration of > 10 mL of frank blood.

One Step Further Question: What findings constitute a positive DPL?

Answer: Aspiration of > 10mL of blood or any bile, feces or urine or lavage fluid with > 100,000 RBC/mm3, >500 WBC or the presence of amylase in the aspirated fluid.

One Step Further Question: How long after an acute injury should a bone scan be performed?

Answer: At least 72 hours.

One Step Further Question: What eye finding is concerning for non-accidental trauma in children?

Answer: Bilateral retinal hemorrhages.

One Step Further Question: What pathology is suggested by pupillary asymmetry in a head trauma patient?

Answer: Brain herniation syndrome.

One Step Further Question: Which incomplete spinal cord syndrome has the best prognosis of full recovery?

Answer: Brown-Séquard syndrome.

One Step Further Question: What is the most common cranial nerve injured from a basilar skull fracture?

Answer: CN VII; facial droop.

One Step Further Question: What is the traditional diagnostic approach to Zone III neck injuries?

Answer: CT angiogram and angiography of the neck.

One Step Further Question: What effect does hyperventilation have on the cerebral vasculature?

Answer: Cerebral vasoconstriction.

One Step Further Question: Do children with exposure to household electrical currents need continuous cardiac monitoring?

Answer: No. Healthy children with exposure to regular currents without water only require an EKG.

One Step Further Question: Is a hangman's fracture often seen with suicidal hangings?

Answer: No. Most people die from asphyxiation.

One Step Further Question: What is the diagnosis of a patient with acute crush injury, distal neurovascular compromise, and increased interfascial pressures?

Answer: Compartment syndrome, an orthopedic emergency requiring immediate fasciotomy to normalize interfascial compartment pressures.

One Step Further Question: What CT finding can often distinguish venous and arterial sources of pelvic hemorrhage?

Answer: Contrast extravasation is 80%-90% sensitive for identification of arterial hemorrhage, most commonly arising from injury to the superior gluteal or obturator arteries. When present, arterial bleeding should be treated by emergent angioembolization.

One Step Further Question: What is the most common eye injury seen in the ED?

Answer: Corneal abrasion.

Which of the following is considered a "hard" sign in penetrating neck trauma? Absent radial pulse Hoarseness Minor hematemesis Subcutaneous emphysema

Correct Answer ( A ) Explanation: Stable patients with penetrating neck trauma should be evaluated for the presence of "soft" and "hard" signs of aerodigestive or neurovascular injury. Most patients with hard signs benefit from surgical intervention. An absent radial pulse is a hard sign suggestive of significant injury Hematemesis (C), hoarseness (B), and subcutaneous emphysema (D) are all soft signs in penetrating trauma and will require a full diagnostic workup.

A 37-year-old woman presents with a stab wound to the abdomen. Her vitals are HR 157, BP 95/50, RR 22, and oxygen saturation 96%. Physical examination reveals a single stab wound to the left upper quadrant and is otherwise unremarkable. FAST exam is positive for fluid in the splenorenal space. What management should be pursued at this time? CT scan of the abdomen and pelvis Diagnostic peritoneal lavage Left tube thoracostomy Transport to the operating room to undergo explorative laparotomy

Correct Answer ( D ) Explanation: This patient is suffering from hemorrhagic shock from a penetrating abdominal injury and requires immediate operative management. Hemorrhagic shock in this patient is evidenced by a marked tachycardia. In young patients, a fall in blood pressure can be a late finding of hemorrhagic shock as these patients are able to maximally vasoconstrict their peripheral vasculature in an effort to maintain perfusion. Heart rate, however, will rise earlier in the course. The assessment of a trauma patient with hemorrhagic shock should focus first on stabilizing the airway and breathing and then on reversible causes of circulatory collapse. These reversible causes include hemopneumothorax, pericardial tamponade, displaced long bone fractures and, in some cases, pelvic bleeding from the fracture. This patient does not exhibit any of these reversible causes. The FAST exam demonstrates free peritoneal fluid in the splenorenal space. FAST exam has a sensitivity of 60-95% for detecting as little as 100 ml of free fluid and an excellent specificity. As a result of the high specificity, a positive study should be immediately acted upon without delay and the patient should be brought to the operating room to undergo an explorative laparotomy. In the image below, the patient has free fluid in the presplenic space, which is another possible location for fluid to accumulate in the left upper quadrant. A CT scan (A) of the abdomen and pelvis represents the optimal imaging modality for delineating all injuries in an abdominal trauma. CT is very sensitive for detecting vascular, solid, and hollow organ pathology. However, CT scan takes time and delaying operative management in a patient with penetrating trauma and hemorrhagic shock can result in increased morbidity and mortality. Diagnostic peritoneal lavage (DPL) (B) consists of two procedures. The first is aspiration and if negative, the clinician should proceed to lavage. DPL has been supplanted in recent years for the diagnosis of intraabdominal injuries after trauma by FAST examination as bedside ultrasound is faster and less invasive. However, in patients with multiple trauma and hemodynamic instability and an equivocal FAST, DPL can be used to differentiate an abdominal injury from a pelvic injury (which will have a negative DPL). In the presence of a clearly positive FAST exam, DPL is not necessary. A tube thoracostomy (C) is not indicated for this patient as there are no signs of hemo or pneumothorax.

Rapid Review Tension Pneumothorax

Tension Pneumothorax Patient with a history of chest trauma PE will show diminished or absent breath sounds, tracheal deviation away from the side of the injury, hypotension, jugular venous distension Diagnosis is made clinically Treatment is needle decompression of the chest in the second intercostal space in the midclavicular line followed by chest tube insertion

One Step Further Question: What is the most common cause of fetal demise after trauma?

Answer: Maternal death. Placental abruption is the next most common cause.

One Step Further Question: What is the most common vessel injured in an epidural hematoma?

Answer: Middle meningeal artery.

One Step Further Question: What other classic skin infection does Pseudomonas cause?

Answer: Hot tub folliculitis.

One Step Further Question: What is Cushing's response?

Answer: Hypertension, bradycardia, and decreased respiratory rate in response to elevated intracranial pressure.

One Step Further Question: What is the ED management of patellar tendon ruptures?

Answer: Immobilize the knee in extension with a knee immobilizer, and apply ice and a compressive dressing.

One Step Further Question: What are the indications for operative fixation of a flail chest segment?

Answer: Inability to wean patient from ventilator, severe chest wall instability, persistent pain and progressive pulmonary function decline.

One Step Further Question: Which ligament connects the medial aspect of the cuneiform to the second metatarsal base?

Answer: Lisfranc ligament.

One Step Further Question: What are the most important factors in the development of wound infections after mammal bites?

Answer: Location (hand and below the knee), delay in care, and host medical history.

One Step Further Question: What is a critical component of the approach to airway control in patients with neurogenic shock?

Answer: Maintenance of cervical and spinal immobilization.

One Step Further Question: What is the recommended dose of mannitol in patients with suspected or confirmed increased intracranial pressure?

Answer: 1 - 2 grams/kg intravenous.

One Step Further Question: What percentage of patients with a calcaneal fracture will have an associated vertebral compression fracture?

Answer: 10%.

One Step Further Question: What three components of the Canadian cervical spine rules are not present in the NEXUS cervical spine rules?

Answer: Age, mechanism, neck rotation.

One Step Further Question: What is the most common cause of lethal hemorrhage in blunt abdominal trauma in pediatrics?

Answer: Although the spleen is injured more frequently than the liver, the most common cause of lethal hemorrhage occurs in severe liver injury.

A 43-year-old man presents after being struck by a motor vehicle on his right side and falling onto his left arm. His trauma evaluation is negative except for pain in the mid-right arm. A radiograph is seen above. Secondary injury to which of the following is the most common complication? Axillary artery Brachial artery Median nerve Radial nerve

Correct Answer ( D ) Explanation: The most common complication from a midshaft humerus fracture is a radial nerve injury occurring in up to 20% of cases. Frequently this is a transient neurapraxia that resolves over the course of several months. In penetrating trauma the injury may be permanent and will often lead to operative exploration. Clinically, patients may exhibit radial nerve palsy with wrist drop. Minimally displaced humeral shaft fractures are usually treated with a coaptation splint and sling and swathe. The axillary artery (A) does not travel through this region and is not a typical complication of this injury. Brachial artery (B) injuries may occur although they are more likely in the context of penetrating injury to this region or a severely displaced or angulated fracture. Median nerve (C) injuries are rarely seen with humerus fractures.

One Step Further Question: What is the classic X-ray finding of a posterior shoulder dislocation?

Answer: "Light bulb sign."

One Step Further Question: What antibiotic is first-line therapy for human bite infection prophylaxis?

Answer: Amoxicillin-clavulanate.

One Step Further Question: What are two contraindications to replantation?

Answer: An unstable patient with other life-threatening injuries and severe crush injury.

One Step Further Question: Which ligament is most commonly damaged in an inversion injury of the ankle?

Answer: Anterior talofibular ligament.

One Step Further Question: How is a distracting injury defined in the NEXUS criteria?

Answer: Any injury thought to have the potential to impair the patient's ability to appreciate other injuries.

One Step Further Question: How much does succinylcholine elevate serum potassium concentration?

Answer: Approximately 0.5 mEq/L.

One Step Further Question: What medication should be avoided in patients with a hyphema and a history of sickle cell disease?

Answer: Carbonic anhydrase inhibitors can cause RBCs to sickle in the anterior chamber, which can lead to increased IOP.

One Step Further Question: Which metaphyseal fractures are concerning for child abuse?

Answer: Corner and bucket-handle fractures.

One Step Further Question: What is Babinski's sign?

Answer: Dorsiflexion of the great toe and fanning of the other toes when the sole is stimulated.

One Step Further Question: Of the total fluid calculated by the Parkland formula, what percentage should be given over the first eight hours from the time of injury?

Answer: During the first eight hours, 50%.

One Step Further Question: Who is at risk for delayed subdural hematoma?

Answer: Elderly and anticoagulated patients.

One Step Further Question: What is the characteristic ECG finding in a patient with a pericardial effusion?

Answer: Electrical alternans.

One Step Further Question: What is the Ellis classification of a tooth fracture with exposed pulp?

Answer: Ellis III. The Ellis classification of I, II and III are used to describe the enamel, dentin and pulp fractures.

One Step Further Question: An accumulation of blood in the potential space between the dura and skull best describes which type of head injury?

Answer: Epidural hematoma.

One Step Further Question: What class of antibiotics is associated with tendon rupture?

Answer: Fluoroquinolones. "FluoroquinoLONES hurt attachments to your BONES."

One Step Further Question: When is surgery recommended for treating an orbital blow-out fracture?

Answer: Fracture >50% of the floor, extraoccular muscle entrapment, enophthalmos or diplopia.

One Step Further Question: What type of scaphoid fracture is the most common?

Answer: Fractures of the scaphoid waist.

One Step Further Question: What is the most common facial fracture?

Answer: Isolated nasal bone fracture.

One Step Further Question: What happens to the A-a gradient in the setting of a pulmonary contusion?

Answer: It increases due to arterial hypoxemia.

One Step Further Question: A bubbling wound in neck trauma is indicative of injury to what structure?

Answer: Larynx or trachea.

One Step Further Question: Of the three categories of blunt pelvic trauma (lateral compression, AP compression, and vertical shear), which type is the most common cause of pelvic fractures?

Answer: Lateral compression.

One Step Further Question: Which nerve root level does the Achilles reflex test?

Answer: Level S1.

One Step Further Question: What is the most consistent physical exam finding in patients with compartment syndrome?

Answer: Loss of two-point discrimination.

One Step Further Question: What is the outcome of most CSF leaks?

Answer: Most heal spontaneously in one week.

One Step Further Question: How do you test the motor and sensory function of the axillary nerve?

Answer: Motor function is tested by having the patient abduct the arm, which uses the deltoid muscles. Sensory function is tested by sensation over the inferior region of the deltoid muscle, which tests the superior lateral cutaneous branch of the axillary nerve

One Step Further Question: What are the motor deficits seen with injury to the axillary nerve?

Answer: Paralysis of the teres minor and deltoid muscles, resulting in loss of abduction of the arm, weak flexion, extension, and rotation of shoulder and is associated with flat shoulder deformity.

One Step Further Question: What is the most serious complication of an Ellis class III fracture?

Answer: Pulpitis or infection of the tooth pulp.

One Step Further Question: What portion of the Glasgow coma score was found to correlate with outcome nearly as well as the composite score?

Answer: The motor score.

One Step Further Question: What does the base deficit crudely represent?

Answer: The physiologic endpoint that distinguishes trivial blood loss from clinically significant hemorrhage.

One Step Further Question: What artery may be injured or transected after a knee dislocation?

Answer: The popliteal artery.

One Step Further Question: What is Seidel sign?

Answer: The presence of a stream of fluorescein dye (after application) secondary to globe rupture.

One Step Further Question: How do cycloplegics work?

Answer: They relax ciliary muscle spasm and prevent contraction of the iris, reducing pupillary photoresponse.

One Step Further Question: In what age group are supracondylar fractures common?

Answer: They typically occur in children between five and ten years of age.

One Step Further Question: What is the recommend splint for a scaphoid fracture?

Answer: Thumb spica splint.

One Step Further Question: What is a potential danger of using a plastic "contact lens" device for an ocular chemical burn?

Answer: Trapping some of the original chemical under the lens.

Which of the following physical exam findings is most suggestive of a Le Fort III fracture? CSF rhinorrhea Infraorbital anesthesia Limited upward gaze Mobility of the mandible

Correct Answer ( A ) Explanation: Le Fort fractures involve the maxillary bone and surrounding structures. They are classified as type I, II, or III (see figure below), depending on the fracture pattern. All Le Fort fractures result in mobility of the hard palate as the injury patterns lead to dissociation of the maxilla from the rest of the bony skull. Whereas Le Fort I fractures are isolated to the lower face, type II and III injuries may be associated with cribriform plate disruption and associated CSF rhinorrhea. Infraorbital anesthesia (B) involving the ipsilateral cheek and lip is associated with inferior orbital wall (orbital floor) fractures due to injury of the infraorbital nerve. Limited upward gaze (C) may also occur with orbital floor fractures due to entrapment of the inferior rectus and inferior oblique muscles. Mobility of the mandible (D) can occur with mandibular fractures. Le Fort fractures involve the midface, not the mandible.

A 42-year-old man presents to the ED with acute onset pain and blurry vision of his left eye. He states that a small explosion in his auto shop sent a piece of shrapnel into the affected eye. Examination with your penlight reveals a teardrop-shaped pupil. No foreign body is visualized. Visual acuity is markedly diminished in the affected eye. What is the most appropriate next step in management? Elevate the head of the bed to at least 30 degrees and place eye shield over left eye Evert the lids to assess for foreign bodies Obtain an MRI of the orbit to evaluate the extent of soft tissue damage Provide antibiotic prophylaxis against endophthalmitis

Correct Answer ( A ) Explanation: The patient's presentation is concerning for a penetrating globe rupture. An irregularly shaped pupil in the setting of trauma is suggestive of this injury. Other signs associated with globe rupture include leakage of vitreous humor or prolapse of internal structures (e.g. the iris). Elevating the patient's head helps to reduce ocular swelling and allows red blood cells to settle in the case of vitreous bleeding. The patient should also have an eye shield placed over the affected eye and be given antiemetics. The patient should be made NPO and ophthalmology should be consulted on an emergent basis for definitive surgical management. Antibiotic prophylaxis (D) is important for globe rupture management but is not the most appropriate next step. All attempts should be made to avoid raising intraocular pressure. Therefore, the clinician should not evert the lids (B) or attempt to obtain an intraocular pressure with the use of a tonometry pen. An MRI (C) is contraindicated because the patient could have a metallic foreign body. CT scan with fine cuts through the orbits is the recommended imaging modality to identify the foreign body and assist with the diagnosis of globe rupture.

The shoulder is most vulnerable to an anterior glenohumeral dislocation in which of the following positions? Abduction and external rotation Abduction and internal rotation Adduction and external rotation Adduction and internal rotation

Correct Answer ( A ) Explanation: The shoulder is most vulnerable when abducted and externally rotated. A fall or tackle with the arm in this position can cause an anterior shoulder dislocation. Posterior shoulder dislocations are less common and are associated with grand mal seizures and electric shock. Abduction and internal rotation (B), adduction and external rotation (C), adduction and internal rotation (D) are not associated with an increased risk of anterior shoulder dislocation.

An 11-year-old girl presents after getting hit in the eye with a golf ball. She complains of right eye pain and decreased vision. Examination reveals moderate periorbital swelling, conjunctival hemorrhage, and a teardrop shaped pupil. Visual acuity is 20/30 in the left eye and light perception only in the right eye. Which of the following is the most appropriate management? Eye shield, CT scan of the orbit, and emergent ophthalmology consultation Measure intraocular pressure and emergent ophthalmology consultation Ocular ultrasound and emergent ophthalmology consultation Timolol drops, intravenous acetazolamide, and emergent ophthalmology consultation

Correct Answer ( A ) Explanation: This patient is suffering from a ruptured globe resulting from direct trauma to the eye. Blunt trauma to the eye causes a sudden elevation in intraocular pressure leading to scleral rupture. Patients will present with ocular pain and decreased vision. Examination often reveals chemosis or subconjunctival hemorrhage or both overlying the scleral defect. If rupture occurs at the limbus, an irregularly shaped (teardrop) pupil may be seen. Management of suspected globe rupture involves protecting the eye (optimally with a curved eye shield that does not touch the globe), CT scan of the orbit, 30-degree head-of-bed elevation, and emergent ophthalmology consultation. Broad-spectrum intravenous antibiotics should be administered as well to reduce the development of endophthalmitis. Historically, succinylcholine was contraindicated as a paralytic agent if the patient required intubation because it was believed that increasing intraocular pressure could lead to ocular extrusion. Recent studies have demonstrated a lack of increased poor outcomes with succinylcholine. Although a decreased intraocular pressure (< 10 mm Hg) supports the diagnosis of a globe rupture, tonometry (B) should not be performed in patients with a high suspicion of a globe rupture as any increase in pressure can worsen the defect or lead to extrusion of intraocular materials. Similarly, ocular ultrasound (C) should be avoided for the same reason. Timolol drops and acetazolamide (D) are two treatments directed at reducing intraocular pressure in acute angle closure glaucoma

34-year-old man presents to the emergency department complaining of extreme left lower leg pain. Three hours earlier, his leg was crushed between his truck and trailer. On physical exam, the pain is exacerbated by passively dorsiflexing the ankle and the leg feels hard and firm. There is diminished sensation in the left foot. Which of the following is the most likely diagnosis? Acute arterial occlusion Compartment syndrome Deep venous thrombosis Superficial thrombophlebitis

Correct Answer ( B ) Explanation: The man in this case most likely has acute compartment syndrome. Compartment syndrome occurs when tissue pressure within a closed compartment compromises perfusion and results in muscle and nerve ischemia. In the extremities, compartment syndrome may occur acutely, usually following trauma, or chronically, seen usually in athletes. Acute compartment syndrome is much more common than chronic compartment syndrome. Fractures are the most common cause of acute compartment syndrome. Other potential causes include crush injuries, thermal burns, highly constrictive bandages, penetrating injuries, bleeding disorders, animal envenomations, and extravasation of intravascular fluids. The anterior compartment of the lower leg is the most common site. One of the most common and earliest symptoms of ACS is pain out of proportion to the apparent injury. Physical manifestations may include pain with passive muscle stretching, firm "wood-like" feeling of the compartment, pallor from compromised vasculature, diminished sensation, or muscle weakness. Motor deficits are late findings. The diagnosis is based on clinical findings and serial measurements of compartment pressures. Immediate surgical consultation should be obtained if acute compartment syndrome is suspected, as this is a surgical emergency. Initial management involves removing any dressings, splints, or casts and keeping the limb level with the torso. Definitive treatment is a fasciotomy. Complications of untreated acute compartment syndrome include muscle contractures, sensory deficits, paralysis, and infection. Acute arterial occlusion (A) manifests with the 5 P's: pain, pallor, pulselessness, paresthesias, and paralysis. Atrial fibrillation is one of the most common causes of acute arterial occlusion. Based on this man's age and history of acute trauma, acute compartment syndrome is more likely. Deep venous thrombosis (C) occurs when blood coagulates within one of the deep veins. Symptoms of deep vein thrombosis include edema, pain, tenderness, warmth, and erythema. Deep vein thrombosis risk factors include immobility, hypercoagulability, and vascular injury. It is unlikely that the man in this case developed a deep vein thrombosis in the three hours after his initial injury. Superficial thrombophlebitis (D) is caused by thrombus formation in a vein located near the skin. Superficial thrombophlebitis is typically seen in patients who are hypercoagulable or immobilized, in patients with varicose veins or systemic lupus erythematosus, and in patients with venous catheters. Superficial thrombophlebitis typically causes induration, erythema, and tenderness along the affected vein.

A 66-year-old woman is a restrained driver in a motor vehicle collision who presents to the ED with chest pain and an oxygen saturation of 93% on 2 L nasal cannula. She has ecchymosis and tenderness over her right thorax. Breath sounds are equal. Which of the following is true regarding her condition? Injury is due to the accumulation of air within the pleural cavity Symptoms quickly improve The initial chest radiograph may be non-diagnostic There is diffuse lung involvement

Correct Answer ( C ) Explanation: A pulmonary contusion occurs most commonly from blunt trauma and is the result of a direct bruise to the lung parenchyma followed by alveolar edema and hemorrhage. Although the initial radiographic findings (patchy, irregular to dense pulmonary infiltrates over the injured area) may be non-diagnostic and initial measurements of gaseous exchange may be normal, pulmonary function may be compromised over the ensuing few hours. Subsequent radiographs taken over several hours may demonstrate irregular opacification of the pulmonary parenchyma in a non-lobular pattern. Chest CT scans may demonstrate a contusion. Uncomplicated pulmonary contusions typically develop over the first 24 hours and resolve in approximately one week. Patients with severe pulmonary contusions experience difficulty breathing and hypoxia. Repeat chest radiographs show an increasing opacity in the affected lung fields. The condition is exacerbated if extensive crystalloid fluid resuscitation has been performed. Blood gas evaluation reveals an increased A-a gradient. Treatment is initially supplemental oxygen administration to reverse hypoxia; however, subsequent intubation and ventilation with positive end-expiratory pressure may be required. Other adjunctive therapies include pulmonary toilet and analgesia. Air within the pleural cavity (A), or pneumothorax, is part of the differential diagnosis for this patient but is less likely to occur than a pulmonary contusion with a blunt trauma mechanism. Pulmonary contusions tend to worsen over time (B) and the patient's initial presentation can change rapidly as there is an increase in alveolar edema and hemorrhage. Lung involvement is usually localized to a segment or a lobe (D). Adult respiratory distress syndrome (ARDS) is associated with diffuse lung involvement.

An 18-year-old man is rushed to the ED by his friends after being stabbed in the chest. Vital signs are T 37.2°C, HR 118, BP 70/40, RR 22, oxygen saturation is 98% on non-rebreather mask (NRB). Two large-bore IVs are established and normal saline is infusing. You obtain the above image by bedside ultrasound. What is the most appropriate next step in management? Consult cardiology Emergency department thoracotomy Endotracheal intubation Pericardiocentesis

Correct Answer ( D ) Explanation: The ultrasound image shows a large pericardial effusion. In the setting of penetrating trauma to the chest and hypotension, pericardial tamponade is likely present. Crystalloid fluids should be started immediately to improve left ventricular filling pressures, and emergent pericardiocentesis (preferably under ultrasound guidance) should be initiated. Aspiration of as little as 5 to 10 mL of blood may result in dramatic clinical improvement. Cardiology consultation (A) will delay management and is not required in the acute setting of traumatic pericardial tamponade. If pericardiocentesis is unsuccessful or the clinical status deteriorates, a thoracotomy (B) should be performed. The patient's airway is patent, and he is oxygenating well. At this time, he does not require endotracheal intubation (C). However, if his condition deteriorates, he will then require a definitive airway. Care should be taken when initiating intubation and positive pressure ventilation as it may decrease venous return and worsen the signs and symptoms of tamponade.

Which of the following fractures is most commonly non-operative in adults? Galeazzi Monteggia Radial head fracture Tibial plateau fracture

Correct Answer ( C ) Explanation: The majority of patients with a radial head fracture will not require operative management. Radial head fractures are typically caused by indirect trauma (e.g., fall on an outstretched hand). There are four types of radial head fractures ranging from the more common type I (undisplaced fracture) and type II (marginal fracture with minimal displacement) to the rarer type III (comminuted fracture) and type IV (fracture with dislocation). Type I and II injuries are usually treated with a sling and range of motion exercises. It is important to fully test the patient's range of motion. If the range of motion is limited by pain, the joint should be injected with an anesthetic agent (lidocaine, bupivacaine) and tested again. Continued limited range of motion in spite of pain control suggests the presence of entrapped fragments and orthopedic surgery may be required. Galeazzi (A) and Monteggia (B) fractures of the forearm have poor outcomes with nonsurgical management. The majority of tibial plateau fractures (D) also require surgery for acceptable functional outcomes.

A 20-year-old man presents with left rib pain after falling while playing soccer and striking his chest. Vital signs are normal. On physical examination, the patient has tenderness to palpation over the 4th rib in the midaxillary line. What management is indicated? Analgesia and discharge home Application of a rib belt Chest X-ray CT scan of the chest

Correct Answer ( C ) Explanation: This patient presents with signs and symptoms consistent with a rib fracture. A chest X-ray should be performed to rule out any other pathology including pneumothorax and pulmonary contusion. Rib fractures are a common injury after thoracic trauma and the incidence increases with increasing age. They may be associated with a number of potential complications including pulmonary contusions, hemothorax, pneumothorax and post-traumatic pneumonia. Fractures are most common at the posterior angle, which represents the weakest area. The ribs most commonly fractured are the 4th - 9th ribs. The 11th - 12th ribs are mobile, which reduces the risk of fracture. However, fractures of these ribs are more likely to be associated with intraabdominal injuries. Rib fractures should be suspected based on history and clinical evaluation. Patients will present with chest pain and tenderness over the area. Imaging should be obtained to rule out the more serious associated complications of pneumothorax, hemothorax and pulmonary contusion. Chest X-ray is the appropriate modality for this but often will not demonstrate the presence of a single rib fracture when it is in fact present. This is particularly true of non-displaced fractures. Rib belts (B) are discouraged as they may decrease the depth of respiration and lead to atelectasis and pneumonia. CT scan of the chest (D) is not routinely required for management of a simple rib fracture. Analgesia and discharge home (A) is likely to occur once more serious pathology is ruled out with a chest X-ray. Patients with rib fractures should also receive an incentive spirometer to help reduce the complication of pneumonia.

Which of the following is the most common complication associated with an anterior shoulder dislocation? Axillary nerve injury Bankart lesion Greater tuberosity fracture Hill-Sachs defect

Correct Answer ( D ) Explanation: A Hill-Sachs deformity is the most common complication of an anterior shoulder dislocation, occurring in up to 40% of cases. The defect is a depression fracture of the posterolateral surface of the humeral head that results from compression of the dislocated head by the lower glenoid rim. Prolonged dislocation leads to larger deformity size. Not surprisingly, a Hill-Sachs defect is also more likely with recurrent anterior dislocations. Brachial plexus injury or damage to the axillary nerve (A) occurs in up to 14% of anterior shoulder dislocations. Axillary nerve injury is usually a neuropraxia and complete recovery is expected. A Bankart lesion (B) is a disruption of the glenoid labrum, occasionally with concurrent bone fragment avulsion and occurs with between 10% and 20% of traumatic anterior shoulder dislocations. Fracture of the greater tuberosity (C) occurs in up to 15% of anterior dislocations.

One Step Further Question: What is the most common type of major joint dislocation?

Answer: Glenohumeral joint (shoulder).

One Step Further Question: What is the most commonly injured nerve during a shoulder dislocation?

Answer: The axillary nerve.

One Step Further Question: Why does a tension pneumothorax cause hypotension?

Answer: Progressive mediastinal shift and increased intrathoracic pressure impedes venous blood return to the heart, causing a marked decrease in preload.

One Step Further Question: What is the proposed mechanism of improved maternal hemodynamics after perimortem cesarean section?

Answer: Relief of aortocaval compression by the fetus.

One Step Further Question: What is SCIWORA?

Answer: Spinal Cord Injury Without Radiographic Abnormalities. This is seen in children as a result of transient subluxation.

One Step Further Question: What happens to WBC count during pregnancy?

Answer: The WBC count increases during pregnancy. It is not uncommon to see levels between 15,000 and 25,000/µL.

One Step Further Question: Which nerve is most often injured with shoulder dislocations?

Answer: The axillary nerve.

One Step Further Question: What nerve is at risk of being compressed in a perilunate dislocation?

Answer: The median nerve.

One Step Further Question: What is the total body surface area of the palm?

Answer: The total is 1% (not including fingers).

One Step Further Question: How many grades of acromioclavicular (AC) separation are there?

Answer: There are 6 grades of AC separation. Grades I, II and III are the most common.

One Step Further Question: What is the most consistent finding in cauda equina syndrome?

Answer: Urinary retention has a specificity of 90%.

One Step Further Question: What effect does hypoventilation have on intracranial pressure?

Answer: Vasodilation of cerebral vasculature and increased intracranial pressure.

One Step Further Question: Which part of the scaphoid is most commonly fractured?

Answer: Waist.

One Step Further Question: Which physical maneuver can be used to test for scapholunate instability?

Answer: Watson's scaphoid shift test.

One Step Further Question: What deformity of the hand is associated with a radial nerve injury?

Answer: Wrist drop. The radial nerve provides innervation to the extensor muscles of the wrist.

One Step Further Question: Which ligament is responsible for causing the injury in central cord syndrome?

Answer: Ligamentum flavum buckles into the cord, causing a contusion of the central part of the cord.

One Step Further Question: What is the most common mechanism for cervical spine injuries?

Answer: Motor vehicle collisions (40%).

One Step Further Question: What is the most common cause of death in nonpenetrating cardiac injuries?

Answer: Myocardial rupture.

One Step Further Question: Does a "negative" CT scan of the cervical spine rule out all clinically significant injuries?

Answer: No, ligamentous injuries are not always identified by CT scanning.

One Step Further Question: What is the teardrop sign?

Answer: Herniated contents from the orbit into the maxillary sinus often look like a teardrop on CT or plain film of the face.

One Step Further Question: What is meant by the term "blowout" fracture of the orbit?

Answer: "Blowout" fractures are caused by direct trauma to the globe resulting in rapidly increased intraorbital pressure and fracture of the orbital walls. The thinnest portions of the bony orbit, the inferior and medial walls, are most commonly implicated.

One Step Further Question: What is the normal pressure of a tissue compartment?

Answer: < 10 mmHg.

An 18-year-old woman presents with a stab wound to the anterior neck. Which sign requires immediate operative management? Expanding hematoma Minor hemoptysis Subcutaneous emphysema Venous oozing from neck wound

Correct Answer ( A ) Explanation: A patient with an expanding or pulsatile hematoma is at high-risk for a serious vascular injury and should immediately be transported for surgical intervention after initial resuscitation. There are a number of hard signs of vascular injury in penetrating neck injuries. These include active hemorrhage, expanding or pulsatile hematoma, bruit, thrill, massive hemoptysis, shock, major hematemesis or aerodigestive track injury. Patients with these signs require surgical intervention regardless of imaging findings. Control of the patient's airway should be undertaken early as many of these injuries can lead to precipitous decompensation and airway obstruction. Patients with soft signs of vascular injuries should have further diagnostic testing done to confirm the presence or absence of a surgical lesion. Soft signs of vascular injury include minor hemoptysis (B), subcutaneous emphysema (C), venous oozing from the neck (D), and nonpulsatile, nonexpanding hematomas.

Which of the following describes a patient with Brown-Séquard syndrome? Ipsilateral loss of motor strength, vibratory sensation, and proprioception with contralateral loss of pain and temperature sensation below the level of injury Ipsilateral loss of pain and temperature and contralateral loss of motor strength, vibratory sensation, and proprioception Pain, loss of temperature below the level of injury, and complete loss of motor function, but retains proprioception and the ability to sense vibration and deep pressure Preservation of motor function with loss of proprioception and vibration below the level of injury

Correct Answer ( A ) Explanation: Brown-Séquard syndrome results from hemitransection of the spinal cord with unilateral damage to the corticospinal and spinothalamic tracts. This injury is usually the result of penetrating injuries or a lateral mass fracture of the cervical spine. It is also caused by spinal cord tumors, infections, and ischemia. It is rarely seen in its pure form, typically occurring with incomplete involvement of related tracts. Prognosis is excellent, with most patients recovering. Pain, temperature, and complete loss of motor function below the level of injury, but retains proprioception and the ability to sense vibration and deep pressure describes anterior cord syndrome (C). Ipsilateral loss of pain and temperature and contralateral loss of motor strength, vibratory sensation, and proprioception is not consistent with any incomplete cord syndrome (B). Preservation of motor function with loss of proprioception and vibration below the level of injury is consistent with posterior cord syndrome (D).

An 18-year-old woman is involved in a head-on motor vehicle collision with significant front-end damage. Her vital signs in the ED are heart rate 125 beats per minute, blood pressure 90/70 mm Hg, respiratory rate 17 breaths per minute, pulse oximetry of 98% on room air, and her GCS is 15. Physical exam reveals an agitated diaphoretic patient. Her breath sounds are equal bilaterally, the neck veins are distended, and there is a large area of ecchymosis on her anterior chest wall. Her abdomen is soft and nondistended and pelvis is stable. Her pulses are palpable and equal in all four extremities. Which of the following is the most likely diagnosis? Cardiac tamponade Massive hemothorax Pulmonary contusion Tension pneumothorax

Correct Answer ( A ) Explanation: Cardiac tamponade results from accumulation of blood within the pericardial sac and is most commonly the result of penetrating trauma. However, patients who sustain significant blunt thoracic trauma (as in this patient, who has diffuse chest wall ecchymosis) are at risk for cardiac rupture. If these patients survive long enough to make it to the hospital, they often exhibit tamponade and shock. Cardiac tamponade is associated with tachycardia and jugular venous distention (distended neck veins). It is also associated with muffled heart sounds but these are rarely appreciated in an ED. Massive hemothorax (B) can occur from injury to the lung parenchyma; the chest wall, including the intercostal or internal mammary arteries; or the heart and great vessels. A massive hemothorax is defined as the presence of >1 L of blood in the pleural space and is always associated with the systemic signs of shock and hypoperfusion. Massive hemothorax is not associated with distended neck veins. Significant pulmonary contusion (C) alters pulmonary ventilation-perfusion matching and thus causes hypoxemia. A tension pneumothorax (D) can present clinically similar to cardiac tamponade. However, lung sounds are decreased or absent with a tension pneumothorax. Moreover, using bedside ultrasound, significant pericardial effusion can be readily identified confirming the diagnosis of cardiac tamponade.

A 24-year-old woman presents to Urgent Care after tripping over her backpack and hitting her head at home. A friend noted brief period of unconsciousness lasting less than 3 seconds. Prior to arrival to urgent care, she vomits once. On your physical exam, she is noted to have a normal physical exam with normal vital signs. According to the Canadian CT Head Rule, which of the following is the most appropriate next step in management? Observation Transfer to emergency department for a CT scan of the head with contrast Transfer to emergency department for a CT scan of the head without contrast Transfer to emergency department for an MRI scan of the head

Correct Answer ( A ) Explanation: Head injuries can be life threatening therefore it is critical to identify those patients at risk for significant pathology. Serious head trauma can result in an epidural hematoma, subdural hematoma, subarachnoid hemorrhage, intracerebral hemorrhage, and skull fracture. For minor head trauma, guidelines help clinicians decide on whether a head CT scan is needed to diagnose or rule out an intracranial injury. One set of rules called the Canadian CT Head Rule devised a set of criteria seen below on which patients with minor head trauma require a head CT scan. The patient in the clinical scenario does not meet any of these criteria and therefore can undergo a period of observation. The patient should be watched closely for signs of increased intracranial pressure (due to expanding hematoma) such as vomiting, confusion, motor or sensory deficits, visual changes, altered consciousness. It should be noted that the Canadian CT Head Rule is a guideline and any time a clinician feels a patient may have an intracranial injury despite not meeting the Canadian CT Head Rule, the clinician should go ahead and get the head CT scan. The patient does not meet any of the high-risk criteria in the Canadian CT Head Rule and can be observed. A CT head scan without contrast (C) is the standard test of choice in patients suspected of having an intracranial injury. CT head scan with contrast (B) is not the recommended study in patients with suspected intracranial hemorrhage since blood and contrast are both hyperdense on CT scanning. Head CT scan with IV contrast is reserved for patients with intracranial lesions such as metastatic brain cancers and abscesses. MRI (D) is not the first line imaging modality for head trauma since it is time consuming and is not superior in identifying bleeding compared to a CT scan in the acute setting.

A 17-year-old male restrained driver is transported to the ED after his motor vehicle was struck from behind at a high speed. He arrives in full spinal precautions. His vital signs are within normal limits. On exam, he has no motor ability distal to T10. There is also a loss of pain and temperature sensation with preservation of proprioception and vibration. Which of the following cord injuries did this patient most likely sustain? Anterior cord syndrome Brown-Séquard syndrome Central cord syndrome Posterior cord syndrome

Correct Answer ( A ) Explanation: Incomplete spinal cord injuries are described by four distinct patterns and characterized by variable loss of motor and sensory function. The anterior cord syndrome is characterized by injury to the anterior two-thirds of the spinal cord. The mechanism of injury is usually a compression or flexion type with acute cord impingement but may also be caused by vascular injury. Clinically, patients present with loss of pain and temperature below the level of injury, and complete loss of motor function, but they retain proprioception as well as vibratory and deep pressure sensation. The prognosis for full recovery is poor, with minimal chance of return to meaningful function. Brown-Séquard syndrome (B) results in loss of ipsilateral motor and dorsal column function and contralateral pain and temperature sensation. Central cord syndrome (C) presents clinically with sensory and motor deficits that are more pronounced in the upper extremities. Posterior cord syndrome (D) is rare and results in loss of proprioception and vibration, with preserved motor function.

A 47-year-old man presents as a trauma activation after a head-on motor vehicle crash at 20 mph. He was a restrained driver and the airbags did deploy. There were no drugs or alcohol involved. He is uncertain if he lost consciousness. EMS extricated him, put him in spinal immobilization, and splinted an obviously fractured femur. On primary survey, his airway is protected, he is breathing easily, and is normotensive. Vital signs are within normal limits. Secondary survey is notable for a GCS score of 15, a mildly tender right trapezius muscle, and an ecchymotic and deformed left femur with normal distal neurovascular function. The presence of which historical or physical finding necessitates cervical spinal imaging? Distracting injury Mechanism of injury Paraspinal tenderness Possible loss of consciousness

Correct Answer ( A ) Explanation: Neck pain after a traumatic insult is common. Motor vehicle collisions, falls, and violence are the leading causes of spinal injury. However, it can be difficult to determine which injuries require imaging for spinal column damage. Cervical spine injury has a high potential for morbidity. The (National Emergency X-Radiography Utilization Study) NEXUS is one of the best known clinical decision rules for clinical cervical spinal clearance. The five criteria are shown below. If all five of the criteria are satisfied, the NEXUS rules indicate that the patient can be clinically cleared of a cervical spinal injury. Presence of a single criterion necessitates imaging. A femur fracture is a distracting injury, thereby necessitating cervical imaging. Mechanism of injury (B) is not a part of the NEXUS criteria. Only midline cervical spinal tenderness is included in the NEXUS criteria, therefore paraspinal tenderness (C) does not necessitate further spinal imaging. If the patient currently has a normal level of alertness and consciousness, then imaging is not indicated. A possible loss of consciousness (D) does not require cervical imaging.

Which of the following statements is true for a pregnant woman in the third trimester of pregnancy? Blood pressure returns to near normal after declining in the second trimester Gastric emptying time is reduced Heart rate is slower than during the first trimester Hematocrit is increased

Correct Answer ( A ) Explanation: Pregnancy causes major physiologic changes and altered anatomic relationships involving nearly every organ system of the body. Pregnancy results in a 5-15 mm Hg fall in systolic and diastolic pressures during the second trimester, but pressures return to near normal levels in the third trimester. Gastric emptying (B) time is prolonged, not reduced, during pregnancy so you should always presume that the stomach of a pregnant patient is full. Heart rate (C) increases gradually by 10-15 beats per minute during pregnancy and reaches a maximum rate by the third trimester. In pregnancy, a physiologic anemia exists where there is a smaller increase in red cell volume relative to plasma volume (which increases steadily throughout pregnancy and plateaus at 34 weeks gestation). As a result, the hematocrit is decreased (D) in pregnancy.

A 22-year-old woman presents with pain and swelling of the plantar surface of her foot. She reports that two days prior to arrival she was walking in sneakers and stepped on a nail that punctured her foot. Physical examination demonstrates 3 cm of warm, blanching erythema without induration or fluctuance. Her tetanus vaccination is up-to-date. An X-ray does not show any foreign body. Which antibiotic is an appropriate choice to cover the organism at risk from her shoe? Ciprofloxacin Dicloxacillin Linezolid Trimethoprim-sulfamethoxazole

Correct Answer ( A ) Explanation: Puncture wounds on the bottom of the foot are at high risk for infection and also foreign body retention. Particularly in the setting of a puncture wound that passes through the sole of a shoe, Pseudomonas aeruginosa is the causative organism requiring treatment. In theory, the organism has an affinity for rubber or plastic materials and therefore is transferred to the skin through the puncture. In general, Pseudomonas rarely causes skin and soft tissue infections. Ciprofloxacin is an oral fluoroquinolone with adequate Pseudomonas aeruginosa coverage. Care must be taken in children with ciprofloxacin because of concern over the development of tendonopathy. In considering all skin and soft tissue infections, Staph and Strep are still the most common causative organisms, but this particular clinical scenario mandates coverage for Pseudomonas. Dicloxacillin (B) is a penicillin derivative commonly used to treat infections caused by Staph aureus. Staph aureus coverage is required in skin infections given the prevalence of that organism in cellulitis infections. However, it does not cover Pseudomonas. Linezolid (C) is an oxazolidinone antibiotic with activity against Gram-positive organisms. It is typically reserved for resistant infections like Methicillin-resistant Staph aureus and Vancomycin-resistant Enterococcus. It is available as an oral agent allowing for transition to oral therapy from IV medication in the hospital. Trimethoprim-sulfamethoxazole (D) is a sulfa-based antibiotic, which in the setting of skin infections has widespread activity against community-associated Methicillin-resistant Staph aureus. It does not have any activity against Pseudomonas and would not be indicated in this clinical scenario.

Which of the following is the most common type of shoulder dislocation? Anterior Inferior Posterior Superior

Correct Answer ( A ) Explanation: Shoulder dislocations are very common, due in part to the inherent instability of the shoulder joint. Anterior dislocations are most common and make up 95-97% of all shoulder dislocations. The mechanism of injury is generally due to a blow to the arm that is abducted, externally rotated, and extended. Sports activities such as blocking a basketball shot, motor vehicle accidents and a fall on an outstretched arm can all lead to anterior shoulder dislocation. Treatment involves reduction of the shoulder, immobilization and referral to an orthopedic specialist within one week of the injury. Inferior dislocation (B) and superior dislocation (D) are the least common types of shoulder dislocation. Posterior dislocation (C) is the second most common type of shoulder dislocation, but only occurs 2-4% of the time. Posterior dislocation can be seen with a blow to the anterior shoulder or post-seizure due to violent muscle contractions.

A 16-year-old boy is brought to the emergency department after being hit by a car while riding his bike. He is somnolent, opens his eyes to pain, responds to questions with inappropriate words, and uses his left hand to stop the physician attempting to start an IV in his right arm. What is the patient's Glasgow coma score? 10 11 12 9

Correct Answer ( A ) Explanation: The Glasgow coma scale (GCS) is a grading of patient responsiveness based on three components: eye opening (1 to 4 points), verbal response (1 to 5 points), and motor response (1 to 6 points). The severity of traumatic brain injury (TBI) is defined using the GCS with mild TBI being defined as a GCS of 14-15, moderate TBI a GCS of 9-13, and severe TBI a GCS 3-8. While other grading tools for mental status exist, the GCS is commonly used due to its interobserver reliability and prognostic value. The Glasgow coma scale has been shown to have prognostic value in traumatic brain injury, spontaneous subarachnoid hemorrhage, and bacterial meningitis. Limitations of the scale include its decreased utility in the setting of drugs, alcohol, sedation, and intubation and its reliance on behavioral response without regard for the underlying pathophysiology. The patient above would receive a score of 2 for eye opening, 3 for verbal response, and 5 for motor response for a total of 10. The patient's GCS is E2V3M5 for a composite score of 10, not 11 (B), 12 (C), or 9 (D).

A 36-year-old woman presents to the ED one day after falling off a bar stool and striking her head on the floor. She denies loss of consciousness. Which of the following statements regarding this patient's condition is correct? Cranial nerve entrapment is a complication Localizing signs are often present on exam Prophylactic antibiotics should be started Skull radiographs are a quick and easy way to make the diagnosis Surgical decompression is usually required for definitive management

Correct Answer ( A ) Explanation: The image denotes ecchymosis in the postauricular area, which develops in the setting of a basilar skull fracture. In such a case, the fracture line communicates with the mastoid air cells, resulting in accumulation of blood in the cutaneous tissue. This is also referred to as Battle's sign. Basilar skull fractures often compress and entrap the cranial nerves that pass through the foramen. Localizing signs (B) are rare. Indirect evidence of the injury, however, is common and includes visible bleeding from the fracture into surrounding soft tissue, such as Battle's sign or "raccoon eyes," hemotympanum, and blood in the sphenoid sinus. A basilar skull fracture serves as an entrance for bacteria into the brain and can be complicated by the development of meningitis. Cerebrospinal fluid (CSF) leaks may be noted by clear or pink rhinorrhea or otorrhea. Most CSF leaks resolve spontaneously. Antibiotics (C) are generally not given prophylactically. Skull radiographs (D) are poor at identifying a basilar skull fracture. CT is the best radiographic tool to aid in identifying the fracture site. Generally, patients with a basilar skull fracture do not require treatment (E) other than pain medication, antiemetics, and observation.

What is the most commonly fractured carpal bone? Capitate Lunate Pisiform Scaphoid

Correct Answer ( D ) Explanation: The scaphoid is the most commonly fractured carpal bone, accounting for 70% of all carpal fractures. Fracture is most common between the ages of 15 and 40 years and often due to a fall on an outstretched hand or a sports-related incident. Fractures are seen in three different anatomic locations: tuberosity and distal pole, waist, and proximal pole. Patients will have dorsal radial wrist pain, limited range of motion of the wrist and thumb, and tenderness to palpation of the anatomic snuffbox. Radiographic diagnosis of scaphoid fracture is challenging even with dedicated scaphoid views, failing to detect up to 15% of fractures. In patients where fracture is suspected clinically, they should be placed in a thumb spica splint and undergo repeat imaging in seven to ten days. Complications include nonunion and avascular necrosis secondary to the distal to proximal blood flow of the scaphoid.

A 3-year-old boy presents to the ED with an injured lip after chewing on an electrical cord at home. Which of the following statements is correct regarding electrical lip burns? Delayed hemorrhage from the labial artery may occur after the eschar separates Injury from this type of electrocution is limited to the lip and teeth It is important to debride dead tissue from the wound to promote healing Jaw dislocations are a common complication

Correct Answer ( A ) Explanation: The patient sustained an electrical lip burn, which commonly occurs when a child bites on an electrical cord. Asymptomatic patients with low-voltage injuries can be discharged after basic wound care. A significant risk of delayed bleeding from the labial artery exists when the eschar separates. This usually occurs 3 to 14 days post injury. Injuries that occur (B) from chewing an electrical cord are usually arc burns that may involve the orbicularis oris muscle, the oropharynx, and damage to the developing dentition. The burned tissue should not be debrided (C) due to the risk of bleeding from underlying structures. The wound should be cleansed and a petroleum-based antibiotic ointment applied. Jaw dislocations (D) are not associated with electrical lip burns.

19-year-old man presents for evaluation after a fall. The patient was rollerblading and fell backwards onto pavement. There was no loss of consciousness and his only complaint is a headache. On physical examination, the patient has a GCS of 15. There is no scalp hematoma. His cervical spine is non-tender. On inspection of the ear canal, you see blood behind the tympanic membrane. What injury is associated with this physical exam finding? Basilar skull fracture Depressed skull fracture Intracerebral contusion Traumatic subarachnoid hemorrhage

Correct Answer ( A ) Explanation: The physical examination of this patient demonstrates hemotympanum, which is blood behind the tympanic membrane. This finding is associated with a basilar skull fracture which is a linear fracture often through the temporal bone causing bleeding in the middle ear. Patients with this finding need a CT scan of the brain, with attention paid to the temporal bone area. Basilar skull fractures may be associated with dural tears which may be a portal of entry for infection requiring prophylactic antibiotics. A depressed skull fracture (B) would be on the external calvarium and not lead to findings in the middle ear. These may be palpated on physical examination or may just be associated with a scalp hematoma overlying the fracture site. Intracerebral contusions (C) occur from direct trauma to the head and are an area of bleeding within the brain parenchyma. Multiple contusions may occur as a result of the coup/contrecoup mechanism. Contusions may also occur under the site of a depressed skull fracture. Traumatic subarachnoid hemorrhage (D) is the most common abnormality seen on CT scan after head injuries. Isolated traumatic subarachnoid injuries tend to have good outcomes; although, severe cases of hemorrhage may lead to cerebral vasospasm.

Which of the following is the most concerning complication of this injury? Avascular necrosis Infection Malunion Osteoarthritis

Correct Answer ( A ) Explanation: The radiograph demonstrates a fracture at the waist or middle third of the scaphoid. The blood supply to the scaphoid penetrates the cortex on the dorsal surface near the tubercle waist area (distal aspect of scaphoid). Therefore, there is no direct blood supply to the proximal portion of the bone. Because of this tenuous blood supply, scaphoid fractures have a tendency to develop avascular necrosis (AVN). The more proximal the fracture, the greater the likelihood of developing AVN. Infection (B) is an uncommon complication and not the most concerning complication of an isolated scaphoid fracture. Malunion (C) is a complication due to improper healing of the fracture. It mainly occurs when a scaphoid fracture goes unrecognized or there is early discontinuation of immobilization. This can lead to arthritis (D) over time due to misalignment from the abnormal motion and collapse of the bone fragments. If caught before arthritis has developed, surgery may be performed to try to improve scaphoid healing.

A 33-year-old man presents with left shoulder and upper arm pain after a fall. Physical examination reveals a "squared-off" appearance of the shoulder. Which nerve is most commonly affected in this injury? Axillary nerve Median nerve Radial nerve Ulnar nerve

Correct Answer ( A ) Explanation: This patient presents with an anterior shoulder (glenohumeral) dislocation and the axillary nerve is a common concomitant injury. The glenohumeral joint represents the most commonly dislocated major joint. The joint may dislocate anteriorly (most common), posteriorly, or inferiorly. The injury is most commonly sustained in young patients during athletic activities involving rapid movements and in older people with axial trauma (e.g., fall on an outstretched arm). Patients present with severe pain and the arm is typically held in slight abduction and external rotation. The acromion becomes prominent and the shoulder appears "squared-off" as opposed to the typical rounded appearance. A complete neurovascular examination should be performed on all patients with glenohumeral dislocations. Due to its location, injury to the axillary nerve is common, occurring in up to 54% of patients (but may be as low as 5%). The axillary nerve provides sensation to the lateral aspect of the shoulder. Motor function of this nerve can be tested by examining the function of the deltoid muscle. The majority of patients with axillary nerve injury have neuropraxia, which typically recovers over time without specific intervention. The median nerve (B) may be injured during a posterior elbow dislocation. Radial nerve injuries (C) should be suspected in humeral fractures. Ulnar nerve injures (D) are uncommonly seen in anterior elbow dislocations.

A 38-year-old man presents to the ED with his friend. They were playing racquetball and the man was hit directly in the eye with the ball. Examination reveals limitation of upward gaze, periorbital erythema, and subconjunctival hemorrhage of right eye. Imaging of the orbit would most likely show a fracture of which bone? Frontal Maxillary Sphenoid Zygomatic

Correct Answer ( B ) Explanation: A blowout fracture typically refers to a fracture of the inferior orbital wall fracture. The most common causes are direct blunt injury such as a fist, sports' ball injury, and motor vehicle collisions. Symptoms include upward gaze diplopia, decreased extraoccular eye movements due to entrapment of the inferior rectus muscle, periorbital ecchymoses, eyelid edema and subconjunctival hemorrhage. Suspicion is based on the history, while diagnosis requires radiographic or computed tomography confirmation. Typical radiographic findings include a teardrop sign (a mass of herniated orbital contents, fat and the inferior rectus muscle), orbital lucency, and sinus air-fluid levels. Pure blowout fractures occur most frequently in the inferior orbital floor. Since the floor (inferior wall) is the most common site of orbital blowout, the maxillary bone is most commonly fractured. The medial wall is the second most commonly fractured area. The orbit is a pyramidal-like structure made up of 6 areas: superior orbital wall (frontal bone) (A), inferior orbital wall or floor (maxillary bone), medial wall (lacrimal and ethmoid bones), lateral wall (zygomatic bone) (D), apex (sphenoid bone) (C), and base (anterior eyeball surface).

A 24-year-old man comes to the emergency department complaining of right shoulder pain. He was playing basketball and fell on his outstretched arm. Physical examination shows deformity of the right shoulder. His X-ray is shown above. Injury to which of the following is most likely associated with this event? Axillary artery injury Axillary nerve injury Biceps tendon rupture Reverse Hill-Sachs deformity

Correct Answer ( B ) Explanation: Anterior shoulder dislocations are the most common type of shoulder dislocation. The axillary nerve is the neurologic structure most at risk for injury during a shoulder dislocation. Sensation over the deltoid muscle is the way in which axillary nerve function is tested on physical examination. Additionally, distal nerve function of the median, radial and ulnar nerves should be evaluated as part of the complete evaluation of the shoulder joint. It is important to document all findings of the neurovascular examination prior to any attempts as a baseline. Posterior dislocations are associated with significant injury 30% of the time to intrathoracic and mediastinal structures. These complications include injury to the great vessels, tracheoesophageal fistula, pneumothorax, and brachial plexus injuries. Biceps tendon rupture (C) is rare in the setting of shoulder dislocations. An injury to ligaments of the rotator cuff is more common. The Hill-Sachs deformity is an injury to the posterolateral humeral head present in up to 50% of anterior shoulder dislocations. A reverse Hill-Sachs deformity (D) is an impaction fracture of the anteromedial head common in posterior shoulder dislocations. An axillary artery injury (A) is not common with anterior shoulder dislocations. Scapular injuries, including scapulothoracic dissociation, are a more likely cause of axillary artery injury.

A 15-year old boy presents to the ED after falling two stories down an elevator shaft. He complains of severe bilateral heel and back pain. On exam, he is alert with normal vital signs. His dorsalis pedis, popliteal, and femoral pulses are strong; no other deformity is recognized. Radiographs of the bilateral heels reveal calcaneal fractures. What additional test should be considered in this patient? Compartment pressures Complete radiographs of the spine CT angiography Retrograde urethrogram

Correct Answer ( B ) Explanation: Bilateral calcaneal fractures are usually secondary to an axial loading injury, most commonly associated with a fall from a significant height. Vertebral compression fractures occur concomitantly in 10% of calcaneal fractures. Therefore, this patient requires a complete radiographic spinal evaluation. Other associated injuries include fractures of the talus, pilon, tibial plateau, hip, and acetabulum. Compartment pressures (A) help to diagnose compartment syndrome, which is most common after crush injuries or fractures with marked swelling. The most common sign is pain with passive stretch of muscles within the affected compartment. Calcaneal and spinal fractures are not commonly associated with compartment syndrome and further investigation is not required without additional signs or symptoms pointing toward this diagnosis. CT angiography (C) is useful for detecting vascular injuries in the setting of trauma. Although aortic injury can occur with rapid deceleration injuries such as a fall from height, this patient has normal vital signs and is complaining only of heel and back pain. CT angiography is not indicated. A retrograde urethrogram (D) is a procedure used to detect injury to the urethra. Urethral injuries most often accompany blunt abdominal and pelvic trauma.

42-year-old man presents to the ED with an amputation of his left thumb just proximal to the interphalangeal joint. The injury occurred one hour ago at a rural construction site while the patient was operating a power miter saw. The thumb is brought in, in a sandwich bag, along with the patient. Which of the following is true regarding predictors of successful replantation? Crush injuries have a high success rate of replantation Digits have better tolerance for ischemia than limbs have The amputated part should be kept cold and dry The patient's social history adds little value to the success rate of replantation

Correct Answer ( B ) Explanation: Ischemia time is one of the most important predictors of successful replantation. Digits have less muscle mass to oxygenate and tolerate ischemia better than amputations more proximally along the limb. Replantation of limbs must be completed within four to six hours, but digits can tolerate an ischemic time of up to eight hours, given proper preservation. Crush injuries (A) have a low success rate for replantation due to the significant destruction of neurovascular structures. The amputated body part should be irrigated with normal saline to remove gross contamination, wrapped in sterile gauze moistened with saline, and placed in a sterile, watertight container. This container should be placed in ice water, but the digit itself should not be submerged. It should not be kept dry (C). Research has repeatedly shown that tobacco use (D), especially smoking after surgery, will worsen the chance of successful replantation. Obtaining a social history is important in these cases.

A 12-year-old boy presents to the ED after his brother bumped into his elbow while he was using a cotton-tipped swab to clean out his ear. He denies dizziness, vertigo, nausea, or hearing loss. On exam, you note a tear of the tympanic membrane and a small amount of blood in the external canal. Which of the following is the most appropriate next step in management? Begin antibiotics Discharge with instructions to keep the ear canal dry Emergency department consultation to ENT Pack the ear with gauze

Correct Answer ( B ) Explanation: Perforation of the tympanic membrane (TM) can result from a penetrating object, loud noise, infection, lightning strike, or rapid change in pressure. Otoscopic exam usually reveals a tear with immobility of the membrane. Acute perforations have irregular borders with blood on the edges or in the canal. Patients may complain of hearing loss, nausea, vomiting, vertigo, or facial palsy. Most perforations heal spontaneously, and since this patient is asymptomatic, he can be discharged home with instructions to keep the ear dry. If the patient was symptomatic, it would suggest the presence of concurrent injury to the inner ear structures and require emergent ENT referral (C). Furthermore, antibiotics (A) are not recommended in patients with ruptured tympanic membranes unless caused by an infection, associated with contamination (e.g. contaminated water), or if the canal is occluded by blood or drainage such that the tympanic membrane cannot be visualized. Packing the ear with gauze (D) is not recommended, though some people find it helps keep the ear canal dry during a shower.

Which of the following is feature of the injury shown above? Hematoma formation will not cross cranial suture lines It is often associated with concurrent brain injury and parenchymal damage It most commonly results from an arterial bleed Risk for this injury decreases with decreasing ag

Correct Answer ( B ) Explanation: Subdural hematomas are caused by tearing of bridging dural veins due to sudden acceleration-deceleration. This results in accumulation of blood in the space between the dura mater and arachnoid mater. Extensive atrophy of the brain, as seen in the elderly and alcoholics, is a major risk factor, as it allows for generation of more force from sudden acceleration-deceleration. Other risk factors include age < 2 years, low cerebrospinal fluid volume, prior head trauma, and anticoagulation. While subdural hematomas accumulate more slowly due to the venous nature of the bleed, subdural hematomas are more likely to be associated with concurrent brain injury and parenchymal damage. There is no specific clinical syndrome associated with SDH and presentations can range from asymptomatic to altered mental status and death. Because hematoma formation occurs between the dura and arachnoid, subdural bleeds readily cross cranial suture lines. Subdural hematomas can be acute, subacute or chronic. Radiographic findings include a "crescent shaped" lesion that crosses suture lines on non-contrast CT. CT may also be able to help identify the chronicity of a SDH, with more acute hematomas appearing hyperdense (white) and more chronic hematomas appearing isodense. Subdural hematomas, unlike epidural hematomas, can cross suture lines (A) because the blood accumulates between the dura and arachnoid. They can rarely occur due to arterial bleeds (C) and are most commonly the result of tearing of bridging dural veins. While subdural hematomas are more common in the elderly due to brain atrophy, the risk of subdural hematoma formation by age is bimodal (D), with children under the age of 2 years also at high risk. Subdural hematoma formation in young children should raise suspicion for non-accidental trauma.

Which of the following regarding the "rule of nines " is correct regarding infants? The body is proportionally larger in infants than in adults The head is proportionally larger in infants than in adults The legs are proportionally larger in infants than in adults The "rule of nines" is not reliable in infants; another scale should be used

Correct Answer ( B ) Explanation: The "rule of nines" is frequently used to help determine burn size in acutely burned patients. Infants and children have a relatively larger portion of the body surface area in the head and neck, which makes up for the relatively smaller surface area of the lower extremities. Infants have 21% of the total body surface area in the head and neck (18% of head alone, compared to 9% in adults) and 14% in each leg, which incrementally approaches the adult proportions with increasing age. Body proportions (A) are similar in infants, children, and adults, making up 18% of the anterior body and 18% of the posterior body. The legs are proportionally smaller (C) in infants and children than in adults (13.5% vs. 18%, respectively). Although the rule of nines requires modest modification in infants and children (D), it is still valid and can be utilized. There are specific formulas available to help determine the total body surface area in the pediatric population, such as the Berkow formula.

A football player presents with acute anterior knee pain and significant knee effusion. He felt a pop while he was tackled. Which of the following physical examination tests is best used to diagnose a tear of the anterior cruciate ligament? Anterior drawer test Lachman test McMurray test Patellar grind test

Correct Answer ( B ) Explanation: The anterior cruciate ligament (ACL)is the primary stabilizer of the knee. It resists anterior translation of the tibia on the femur and is the most frequently injured major ligament of the knee. A tear results from deceleration,hyperextension or marked internal rotation of the tibia on the femur. The majority of ACL tears occur during sporting activities such as skiing, football, soccer, and basketball.Clinically, there is usually rapid development of hemarthroses, causing significant swelling. Almost half of individuals report feeling or hearing a pop, which is the most reliable factor. There is usually instability of the knee or a wobbly feeling. The Lachman test is the most sensitive test (approximately 98% sensitivity) showing increased anterior tibial displacement and a soft end point. MRI is rarely necessary to make a diagnosis. Rest, ice, crutches, immobilization and NSAIDs are common initial treatment options. Some competitive young patients may choose reconstruction as the best possible chance to return to successful play. The anterior drawer test (A) carries a 50% false-negative rate. As such, it is much less appropriate then the Lachman test for diagnosing ACL injury. The McMurray test (C) is used to diagnose meniscal, not ACL, injury. The patellar grind test (D) evaluates the articular surface of the patella, and is used in the evaluation of patellofemoral syndrome, not ACL tears.

A patient presents to the ED after a trip and fall while stepping down from a curb. Which of the following injuries should be highly suspected based on the knee radiograph seen above? Patella fracture Patellar tendon rupture Quadriceps tendon rupture Tibial plateau fracture

Correct Answer ( B ) Explanation: The extensor mechanism of the knee may be disrupted at four locations: (1) quadriceps tendon, (2) patella, (3) patellar tendon, and (4) tibial tubercle. The radiograph shows superior displacement of the patella, also known as patella alta, which is diagnostic of patellar tendon rupture. This injury is most commonly seen in patients who fall while descending a staircase or stepping down from a curb. Clinical exam will reveal superior displacement of the patella with inferior pole tenderness and swelling. The most significant finding on exam will be an inability to actively extend the affected knee or to maintain the passively extended knee against gravity. Patella fractures (A) are usually seen on radiographs, with transverse fractures present in more than half of all cases. The quadriceps tendon (C) is considered the primary dynamic stabilizer of the knee. Rupture will also present with an inability to actively extend the knee or to maintain the passively extended knee against gravity. However, the radiograph will show patella baja (low-riding patella) rather than patella alta. Tibial plateau fractures (D) are sometimes subtle on knee radiographs but should not lead to a high-riding patella. It should be noted that patella "tendon" rupture is a misnomer since this is describing a connection between two bones and should be regarded as a patella "ligament" rupture.

An 8-year-old boy is brought to the ED with right wrist pain after a fall in the playground. He has tenderness over the distal radius. After receiving ibuprofen, he is sent to radiology for plain films of the wrist. In anticipation of the results, which of the following is true regarding distal radius fractures in children? Reduction is not required in greenstick fractures with <30 degree of angulation Salter-Harris Type IV fracture requires surgical management The diagnosis of a Salter-Harris Type I fracture is made radiographically There is a high risk of vascular compromise with torus fractures

Correct Answer ( B ) Explanation: The fracture line of Salter-Harris Type IV fracture begins at the articular surface, crosses the epiphysis and growth plate, and extends into the metaphysis, splitting off a metaphyseal fragment. Operative management is required to ensure anatomic reduction, avoid angular deformity, and prevent loss of joint function. This type of fracture is associated with growth disturbances. Greenstick fractures (A) have cortical disruption on one side with intact periosteum on the compression side. Despite being an incomplete fracture, those with >10 degrees of angulation require reduction and immobilization. Salter-Harris Type I fractures (C) extend through the physis. Often there are no radiographic findings. Soft tissue swelling over the growth plate and minimal physeal widening, along with point tenderness on clinical examination, are suggestive of a Type I fracture. Torus fractures (D) are the most common pediatric radial fractures. Complications are rarely seen, likely because there is no true cortical disruption. These fractures usually heal in 2-4 weeks with immobilization.

Which of the following is associated with the condition seen in the above image? Exophthalmos Infraorbital anesthesia Monocular diplopia Paralysis of downward gaze

Correct Answer ( B ) Explanation: The image above represents an inferior orbital wall fracture, or blowout fracture. This typically results from a direct blow to the orbit. Force transmitted through the fluid-filled globe results in a fracture of the inferior or medial orbital walls. Adipose tissue, the inferior rectus muscle, or the inferior oblique muscle can then herniate into the sinus cavity and become entrapped. Infraorbital anesthesia is associated with this fracture due to injury to the infraorbital nerve. Other findings on physical exam include enophthalmos due to herniation of globe contents before the onset of edema. Diplopia on upward gaze occurs with entrapment of the inferior rectus muscle leading to binocular diplopia. When the patient is asked to look up, the unaffected eye will look up, but the entrapped eye cannot. Treatment includes oral antibiotics to treat sinus pathogens, decongestants, and instructions to avoid nose blowing or other behaviors that increase intraocular pressure. Follow-up with a specialist is recommended. Exophthalmos (A) occurs with retrobulbar hematoma. Enophthalmos is associated with globe rupture and blowout fracture. Binocular diplopia, rather than monocular diplopia (C), is associated with a blowout-fracture due to entrapment of the inferior rectus muscle. When this occurs, patients experience paralysis of upward gaze, not downward gaze (D).

Which type of cerebral herniation is associated with the finding above? Central transtentorial Cerebellotonsillar Uncal Upward transtentorial

Correct Answer ( C ) Explanation: Cerebral herniation occurs when increasing cranial volume and intracranial pressure (ICP) overwhelm the natural compensatory capacities of the CNS. Increased ICP may be due to post-traumatic brain swelling, edema formation, or traumatic mass lesion expansion. When ICP is high enough, the intracranial contents shift and herniate through the cranial foramen. The most common clinically significant traumatic herniation syndrome is uncal herniation, a form of transtentorial herniation. These are associated with traumatic extra-axial hematomas in the lateral middle fossa or the temporal lobe. As the uncus is compressed, cranial nerve III is compressed. The first signs of compression include anisocoria, ptosis, impaired extraocular movements, and a sluggish pupillary light reflex ipsilateral to the expanding mass lesion. As the compression progresses, the ipsilateral pupil dilates and becomes nonreactive. An expanding lesion at the frontal or occipital aspect of the brain causes central transtentorial herniation (A). Bilateral central pressure is exerted on the brain. The patient begins to experience altered mental status, bilateral motor weakness, and miotic pupils. As central herniation progresses, both pupils become midpoint and lose light responsiveness. Cerebellotonsillar herniation (B) occurs when the cerebellar tonsils herniate downward through the foramen magnum. This is usually caused by a cerebellar mass or a large central vertex mass causing displacement of the brainstem. Clinically, patients exhibit sudden respiratory and cardiovascular collapse as the medulla impinges. Pupils become pinpoint and a flaccid quadriplegia develops due to bilateral compression of the corticospinal tracts. Upward transtentorial herniation (D) occurs as a result of an expanding posterior fossa lesion. The patient's level of consciousness rapidly declines, and pinpoint pupils develop from compression of the pons. Downward conjugate gaze is accompanied by the absence of vertical eye movements.

A 17-year-old baseball player presents to the emergency department complaining of sudden left shoulder pain. The patient states he heard his shoulder "pop" and felt pain as he was extending his arm back to throw a fastball. On examination, the patient is holding his left shoulder in abduction and external rotation with loss of the deltoid contour. There is limited and painful range of motion. Which of the following is the most likely diagnosis? Acromioclavicular joint injury Anterior glenohumeral dislocation of left shoulder Midshaft fracture of the left humerus Posterior glenohumeral dislocation of the left shoulder

Correct Answer ( B ) Explanation: The patient suffered an anterior glenohumeral dislocation of his left shoulder. The shoulder is the most commonly dislocated joint in the body. Dislocations can occur from a trauma (95%) or from laxity of the ligaments (5%). The glenohumeral ligaments, the rotator cuff muscles, and the joint capsule or labrum stabilizes the shoulder. The shoulder joint is often injured during a fall on an outstretched arm, engaging in sports activities, a direct blow to the joint, or grand mal seizures. Patients often report hearing or feeling a pop during the incident and present with significant pain, limited range of motion, swelling, or deformity. Anterior shoulder dislocations are the most common type; however, they can occur posteriorly and inferiorly. Anterior dislocations result from a force applied to the shoulder while the arm is abducted and externally rotated such as while throwing a ball. On examination, the arm is held in abduction and external rotation (down adjacent to the body) with a loss of the deltoid contour when compared to the other shoulder, and the humeral head can often be palpated anteriorly beneath the clavicle. The patient will have very limited range of motion and be unable to touch the opposite shoulder. Obtaining pre-reduction and post-reduction radiographs and neurovascular examinations are imperative to successful management. Complications of shoulder dislocations include fractures (e.g. Hill-Sachs lesion), soft tissue injury (e.g. Bankart lesion), nerve injury (e.g. axillary nerve), or vascular injury (e.g. axillary artery).

A patient presents to the ED after falling on his outstretched hand. On exam, there is dorsal wrist pain and swelling, with localized tenderness on palpation of the capitate. You obtain the radiograph seen above. Which of the following is the correct diagnosis? Lunate dislocation Perilunate dislocation Radiocarpal dislocation Scapholunate dislocation

Correct Answer ( B ) Explanation: The patient sustained a perilunate dislocation. This is the result of excessive hyperextension. On the lateral radiograph, the lunate remains in position relative to the distal radius, but the capitate is dorsally dislocated in relation to the lunate. The patient should be immobilized with a volar splint and an orthopedist should be consulted. A lunate dislocation (A) is commonly referred to as a "spilled teacup" on radiography because the lunate is displaced and tilted volarly, which makes it looks like a spilled teacup. A radiocarpal dislocation (C) is rare and requires a significant amount of force. A scapholunate dislocation (D) occurs when there is disruption of the scapholunate ligament and a gap between the lunate and the scaphoid is produced. The characteristic radiographic finding of this injury is often referred to as the "Terry Thomas" or "Dave Letterman" sign because it mimics the space between the front incisors that, among other things, both of these comedians are known for.

A 45-year-old inmate from prison presents with a puncture wound to his neck from an unknown object. On exam, you find a comfortable appearing male who is able to speak clearly. He has a 2-mm punctate wound approximately 3 cm lateral to the thyroid cartilage that penetrates the platysma. You note a small amount of oozing blood. Vital signs are BP 145/93, HR 67, RR 23, and 98% on RA. Which of the following is the most appropriate next step? Attempt to locate the oozing vessel and clamp it CT angiogram of the neck Discharge if asymptomatic after 6 hours of observation Transfer to the operating room for surgical exploration

Correct Answer ( B ) Explanation: The traditional approach to neck injuries is based on the location of the injury. There are 3 zones of the neck, each with underlying structures vulnerable to injury. Zone 1 is located from the cricoid cartilage to the sternum. Potential injured structures include subclavian arteries and veins, the apices of the lungs, esophagus, great vessels of the neck, recurrent nerve, and trachea. Zone 2 is located from the cricoid cartilage to the angle of the mandible. Potential injured structures include larynx, pharynx, base of the tongue, carotid artery, jugular vein, and phrenic, vagus, and hypoglossal nerves. Zone 3 is superior to the angle of the mandible. Potential injured structures include internal and external carotid arteries, vertebral artery, and several cranial nerves. In the clinical scenario above, the injury involves Zone 2 of the neck. If the puncture violates the platysma and the patient is stable, then management is conservative and the patient should undergo a CT angiogram of the neck to identify any injured vessels. Other studies may be conducted (e.g., endoscopy, bronchoscopy, and laryngoscopy) to exclude occult injury and identify candidates for surgical exploration and injury repair. If the wound does not penetrate the platysma, a period of observation and discharge (C) is acceptable. Any patient who is hemodynamically unstable and in shock and has an expanding hematoma or uncontrolled bleeding, regardless of the zone that is injured, should be taken emergently to the operating room (D). All bleeding should be controlled with pressure, not with clamps (A). Blind clamping can cause organ, vessel, or nerve damage.

A 23-year-old G1P0 woman at 28 weeks gestation presents after a minor motor vehicle collision with abdominal pain and vaginal bleeding. She states she was a belted, front-seat passenger and the car was hit from behind. She has stable vitals signs. Physical exam reveals a well-appearing woman with a gravid uterus and a non-tender abdominal examination. A sterile speculum examination reveals a closed os with a small amount of dark blood. A transabdominal ultrasound shows a fetus of approximately 28 weeks age with a fetal heart tone at 160 beats/min and a negative FAST examination. Which of the following is the most appropriate next step in management? Discharge home with obstetrics follow up the next day Immediate transfer to obstetrics service for fetal monitoring Prepare patient for emergency department C-section Repeat abdominal examination and ultrasound in six hours

Correct Answer ( B ) Explanation: This patient is at high risk of placental abruption and should be immediately transferred to an obstetrics unit for fetal monitoring once she is determined to be stable for transfer. Bleeding complicates roughly 4% of pregnancies in the second half of gestation. The two most important diagnoses are placental abruption and placenta previa. In placenta previa, the placenta implants over the cervical os. Bleeding occurs when the lower uterine wall elongates leading to tearing of placental vessels. Bleeding is typically painless and bright red in placenta previa. Abruption occurs when a normally implanted placenta separates from the uterine wall due to sheering forces (in the case of trauma) or spontaneous hemorrhage into the decidua basalis (non-trauamatic). Abruption can often go unnoticed as there may be no vaginal bleeding (concealed hemorrhage) associated with it but it is usually associated with pain. Fetus-threatening abruption can occur in the presence of a negative ultrasound. Fetal distress and death occurs in up to 15% of patients with placental abruption as a result of interruption of placental blood and oxygen supply. Patients with significant abruption or signs of fetal distress in a viable fetus may require early delivery. Although the transabdominal ultrasound is reassuring, discharge home with follow up (A) is inappropriate as ultrasound is often unable to detect small placental abruptions. Delaying obstetrics monitoring for six hours for a repeat examination and ultrasound (D) is unnecessary and potentially harmful as it will not lead to reassurance of fetal well-being. Although it is possible that the patient will have to opt for early delivery (C), this decision should not be made without further obstetrics evaluation.

A 52-year-old man presents with worsening right groin pain and inability to bear weight six hours after driving his car into a tree. His AP pelvis radiograph is pictured above. Which complication is he at increased risk for due to his delayed presentation? Acetabular fracture Femoral head avascular necrosis Sciatic neuropraxia Urethral disruption

Correct Answer ( B ) Explanation: This patient presents with a posterior hip dislocation caused by a head-on motor vehicle collision. Approximately 90% of all hip dislocations are posterior. On exam, the patient's leg will characteristically be internally rotated and adducted. Dislocation of a native hip requires a high-energy mechanism and is an orthopedic emergency. Urgent reduction is necessary because the risk of avascular necrosis of the femoral head (and subsequent degenerative joint disease) increases with prolonged (> six hours) dislocation time. Hip dislocation is commonly associated with acetabular fractures (A). Post-reduction imaging should be obtained to further evaluate for these fractures and to ensure appropriate joint positioning. However, the risk of associated fractures is not increased if reduction of a dislocated hip is delayed. Sciatic nerve injury (C) is a complication of reduction that occurs in approximately 10% of patients with posterior hip dislocation; however, the risk of this complication is not a function of the time to reduction. Urethral injuries (D) may be seen with some pelvic fractures and straddle injuries but are unlikely to coexist with an isolated hip dislocation

A 34-year-old man presents to the emergency department with left arm pain after a fall on an outstretched hand while walking his dog. An X-ray is obtained as seen above. What eponym is associated with this fracture? Colles fracture Galeazzi fracture Hutchinson fracture Monteggia fracture

Correct Answer ( B ) Explanation: This patient sustained a Galeazzi fracture, which is a fracture of the middle to distal radial shaft and subluxation or dislocation of the distal radioulnar joint (DRUJ). The mechanism of injury is usually secondary to a fall that causes an axial load on a hyperpronated arm. Injury to the ulnar nerve and anterior interosseous branch of the median nerve can occur. In contrast to skeletally immature patients who are typically treated with closed reduction and casting, adults require immediate orthopedic consultation with open reduction and internal fixation (ORIF). A Colles fracture (A) is a fracture of the distal radius with dorsal and radial displacement of the wrist and hand and typically originates from a fall on an outstretched hand (FOOSH). A Hutchinson fracture (C), also known as a chauffeur fracture or backfire fracture, is an intra-articular fracture of the radial styloid process. The name originates from early chauffeurs who sustained a direct blow to the wrist from the backfiring of the starting crank of a vehicle. A Monteggia fracture (D) is a fracture of the proximal third of the ulna with a radial head dislocation. Monteggia fracture-dislocations are generally treated with open reduction and internal fixation of the ulna with closed reduction of the radial head dislocation. Monteggia fractures are less common than Galeazzi fractures.

A 17-year-old man presents after being punched in the face. He has significant swelling around the left eye and limitation of upward gaze. A CT scan of the orbits is performed. What clinical finding may be present with this injury? Afferent pupillary defect Hemotympanum Hypoesthesia of the upper cheek Instability of the nasal bridge

Correct Answer ( C ) Explanation: A blowout fracture of the orbital floor is the most common simple fracture of the orbit. Typically a blunt force is applied directly to the orbit resulting in a rapid rise in intraorbital pressure causing the orbital contents to herniate through the floor. On CT scan, the contents of the orbits are seen hanging into the maxillary sinus, sometimes referred to as the teardrop sign. With herniation of the contents through the orbital floor, certain physical examination findings may be present. If the inferior rectus muscle is entrapped, the patient will have impairment of upward gaze resulting in diplopia. Additionally, the infraorbital nerve may be compressed or stretched as a result of the injury leading to hypoesthesia of the upper cheek on the affected side. A complete neurologic examination is important in this injury. Additionally, as a result of the trauma, patients may develop a retrobulbar hematoma and evaluation for proptosis and measurement of the intraocular pressure may be indicated. An afferent pupillary defect (A) does not result from an orbital floor injury. The pupillary response is facilitated by cranial nerves II and III, which do not pass through the orbital floor. It is also associated with retinal injuries. Hemotympanum (B) is the presence of blood behind the tympanic membrane seen on otoscopy. This physical examination finding is associated with a basilar skull fracture. Instability of the nasal bridge (D) may be seen in isolated nasal bone fractures or in a Le Fort II or III fracture.

A 47-year-old woman presents after accidentally splashing an ammonia solution into her eye. She is in extreme pain and unable to keep her eye open. Irrigation is initiated immediately. Which of the following is the end point of irrigation? Absence of photophobia Five liters of saline pH of 7.2 Relief of pain

Correct Answer ( C ) Explanation: Alkaline burns to the eye are the most significant injury because of rapid liquefactive necrosis. Alkaline solutions include ammonia and lye and are found in solvents, detergents, cement and drain cleaners. Alkaline solutions are able to penetrate through the structures of the eye because they are more lipophilic than acidic solutions. The necrosis is able to damage all structures of the eye including the cornea, iris and lens leading to permanent visual impairment and blindness. Immediate irrigation is essential to attempt to remove all remnants of the alkaline solutions. Irrigation is facilitated by the administration of topical anesthetics. Large quantities of irrigation are necessary until a stable pH of 7.0-7.2 is obtained. It is suggested that some of the buffered eye wash solutions may be better for irrigation of caustic eye injuries. Additional therapy includes topical erythromycin if there is no corneal defect and a normal anterior chamber on slit lamp examination. However, if there is either a corneal defect or clouding of the anterior chamber, a cycloplegic agent must be added to the topical antibiotics. Patients may continue to have some photophobia after irritation and the absence of photophobia (A) is not the most important end point of therapy. The goal of therapy is to achieve a neutral pH after irrigation. Patients will often require large volumes of irrigation which may be five liters of saline (B) although this number is arbitrary and does not direct therapy. Patients may not have complete relief of pain (D) after the burn. It is important that ophthalmology be involved in the care of these patients for follow-up. A cycloplegic agent may assist in minimizing pain from the injury.

Which of the following is the most frequent clinical finding in cauda equina syndrome? Decreased rectal sphincter tone Saddle anesthesia Urinary retention Weakness in dorsiflexion of the great toe

Correct Answer ( C ) Explanation: Cauda equina syndrome results from a sudden compression of multiple lumbar and sacral nerve roots. It is usually caused by central disk herniation, but it can also result from other compressive disorders such as a spinal epidural abscess, hematoma, trauma, or malignancy. Patients typically present with back pain and radiculopathy at multiple spinal levels. The most consistent examination finding is urinary retention, occurring in 90% of patients. If a patient's postvoid residual urine is < 100 mL, cauda equina syndrome is highly unlikely. Decreased rectal sphincter tone (A) occurs in 60% to 80% of cases and saddle anesthesia (B) in 75%. Other findings seen with cauda equina include paresthesia of the first dorsal web space of the foot and weakness in dorsiflexion of the great toe (D), both due to compression of the L5 nerve root.

A 69-year-old man with a history of atrial fibrillation had a syncopal episode while on vacation in India and struck his head on the stairs. He was transported to the local clinic to be evaluated. His wife accompanied him and she informs the provider that her husband is on a "blood thinner" but she is not sure which one. The patient is alert but is not oriented to time or place. You suspect a subdural hematoma but there is no access to a CT scanner at this time. His vital signs are within normal limits. While awaiting transportation to a facility with a CT scan, which of the following labs should be ordered? Alcohol level Basic metabolic profile INR, PT, aPTT, and platelet count Vitamin K level

Correct Answer ( C ) Explanation: Elderly patients on anticoagulation therapy are at high risk of subdural hematomas when they fall. In general, elderly patients are at risk of falls because they tend to have balance difficulties and decreased vision in older age. A non-contrast CT of the head is the gold diagnostic standard for diagnosing a subdural hematoma. A subdural hematoma usually appears as a hyper dense, crescent-shape-mass between the skull and the brain. A coagulation panel is necessary in the workup of a suspected subdural hematoma because individuals on anticoagulants or who are alcoholics may have an associated coagulopathy placing them at higher risk for a subdural hematoma. In addition, if INR levels are found to be elevated, reversal agents (e.g. vitamin K and fresh frozen plasma can be administered). Therefore, all patients with a head injury should have at least a basic coagulation panel (INR, PT, aPTT, and platelet count). Fresh frozen plasma or platelets should be given as needed. Vitamin K (D) is the antidote for an elevated INR. It is not routinely part of the workup for a patient on anticoagulation who had a fall. Although intoxication is a risk factor for falls and subsequently subdural hematomas, an alcohol level (A) is not beneficial in the workup and treatment of a subdural hematoma. A basic metabolic panel (B), although helpful, is not necessary in the workup of a subdural hematoma since the results will not determine the specific treatment if blood products need to be given.

A 17-year-old man presents after being thrown a far distance off of a horse. Which of the following is consistent with an anterior cord syndrome? Isolated motor function loss Loss of all motor and sensory function Loss of pain and temperature, loss of motor function Upper greater than lower motor weakness

Correct Answer ( C ) Explanation: In order to fully understand the different syndromes of injuries to the spinal cord, it is imperative to understand the location of the tracts of the cord. The posterior columns carry tracts responsible for ipsilateral position and vibratory sensation. The lateral spinothalamic tract carries fibers for contralateral pain and temperature. The lateral corticospinal tract is responsible for ipsilateral motor function. Syndromes may be incomplete depending on how much of the cord is affected by the injury. In the anterior spinal cord syndrome, just the posterior columns are preserved and so patients lose all pain and temperature sensation as well as motor function. Most cases of anterior cord syndrome follow aortic surgery, but it has also been reported in the setting of hypotension, infection, vasospasm, or anterior spinal artery ischemia or infarct. In trauma, typically hyperflexion of the cervical spine causes the injury to the spinal cord. Loss of all motor and sensory function (B) occurs with a complete transection of the spinal cord. Most commonly this occurs after a significant trauma. Isolated motor function loss (A) is not a classic syndrome and would result from a small area of injury on the cord just involving the corticospinal tract. Upper greater than lower motor weakness occurs (D) with a central cord syndrome. Sensory involvement is variable although burning dysesthesias in the upper extremities may occur. Most commonly the syndrome occurs after a fall or motor vehicle accident. Anterior cord syndrome may also be known as ventral cord syndrome.

16-year-old girl is walking down her sidewalk on a winter morning and slips on a patch of ice, striking her chin as she falls forward. EMS place her in a cervical collar before she arrives to the ED. On exam, the patient is able to slightly wiggle her big toe and can feel light touch on her foot but has no movement or sensation in her arms. Which of the following is the most likely diagnosis? Anterior cord syndrome Brown-Séquard syndrome Central cord syndrome Posterior cord syndrome

Correct Answer ( C ) Explanation: Incomplete spinal cord injuries are described by four distinct patterns and characterized by variable loss of motor and sensory function. The central cord syndrome is common and typically results from a forced hyperextension injury. This most commonly occurs in an elderly patient with pre-existing (but often undiagnosed) cervical osteoarthritis but it occurs in patients of any age. The injury involves the central portion of the cord. Clinically, patients present with motor and sensory deficits more pronounced in the upper (compared to the lower) extremities. This is due to the more peripheral positioning of the lower extremity axons within the spinal cord tracts. The prognosis is average in adults with a 50% chance of regaining some function. Children have better prognoses than adults, and approximately 75% of them recover. Anterior cord syndrome (A) presents with complete loss of motor, pain, and temperature below the injury, but retains proprioception and vibratory sensation. Brown-Séquard syndrome (B) results in loss of ipsilateral motor and dorsal column function and contralateral pain and temperature sensation. Posterior cord syndrome (D) is rare and results in loss of proprioception and vibration, with preserved motor function.

A 45-year-old man presents to the emergency room after tripping over a curb. He is unable to put any weight on his right foot. On physical exam, there is bruising over the medial plantar surface and tenderness over the tarsometatarsal joint. Radiographs reveal an avulsion fracture of the second metatarsal and widening of the space between the medial cuneiform and base of the second metatarsal. Which of the following is the most appropriate treatment? Midfoot arthrodesis Non-weight bearing cast immobilization for 8 weeks Open reduction and internal fixation Walking boot for four weeks

Correct Answer ( C ) Explanation: Open reduction and internal fixation is indicated for Lisfranc injuries with any evidence of instability or bony fracture. A Lisfranc injury is characterized by a disruption the tarsometatarsal joints, which connect the forefoot to the midfoot. The injury can range from a mild sprain to severe dislocations with fracture. Lisfranc injuries are more common in males and in the third decade of life. The injury usually results from excessive indirect rotational forces and axial loading through a hyper-plantar flexed foot. Common causes of Lisfranc injuries include motor vehicle accidents, falls, or sports. Patients usually present with severe foot pain and an inability to bear weight. Physical exam may reveal midfoot bruising of the plantar surface, generalized swelling, and tenderness of the tarsometatarsal joint. Anteroposterior, lateral, and oblique radiographs are first line imaging. Stress radiographs may be necessary if non-weight bearing radiographs are unremarkable and there is high suspicion. Radiographic findings may include disruption of second metatarsal, avulsion fragments, or malalignment of the fourth metatarsal and the cuboid bone. Nonoperative management is indicated in patients with no displacement on stress radiographs and no evidence of bony injury. Operative management is indicated in patients with any evidence of instability or fracture. Posttraumatic arthritis is the most common complication of Lisfranc injuries. Midfoot arthrodesis (A) is an operative procedure that is typically used for chronic Lisfranc injuries that have resulted in advanced midfoot arthrosis and progressive arch collapse. The patient in this case has an acute injury that is best managed with open reduction and internal fixation in 2-3 weeks after the soft tissue swelling has subsided. Non-weight bearing cast immobilization for 8 weeks (B) would be indicated for nondisplaced injuries. A short leg cast should be worn for at least 6 weeks. Early return to activity can result in chronic pain and arthropathy. Closed reduction is extremely difficult to maintain with casting alone. A walking boot for 4 weeks (D) is not appropriate management for this unstable injury and could lead to progressive disability and chronic pain. A walking boot may be appropriate for post-operative care.

An 82-year-old woman with osteoporosis falls against her kitchen table. She presents with acute right shoulder pain, proximal right upper extremity edema and near-absent active shoulder joint range of motion. Which of the following is the most appropriate next step in management? Obtain a scapular-Y radiograph Passive range of motion measurement Perform a distal neurovascular examination Perform a Hawkins testing

Correct Answer ( C ) Explanation: Proximal humerus fractures are most commonly seen in elderly patients. The most common type of fracture occurs at the surgical neck (the epiphyseal-diaphyseal junction, just inferior to both tuberosities). As with any fracture, it is imperative to assess distal neurovascular supply with motor, sensory, and pulse examination. Complications from shoulder injuries include injury to the brachial plexus and vascular such as the axillary artery. The Hawkins test (D), which usually follows Neer's testing of the shoulder, is used to evaluate subacromial impingement in the setting of rotator cuff syndrome or tear, the results of which are most likely noncontributory given this presentation. Passive range of motion measurements (B) will most likely be noncontributory and difficult to obtain. Radiographs (A) occur after the clinician is assured that no acute neurovascular compromise is present.

A 63-year-old man presents with right arm pain after a fall. His X-ray is shown above. What structure is commonly injured with this fracture? Axillary nerve Median nerve Radial nerve Ulnar nerve

Correct Answer ( C ) Explanation: Radial nerve injury is the most common nerve injury seen after humeral shaft fractures. These fractures usually occur from a direct blow to the arm and can be seen in falls and motor vehicle collisions. Patients present with severe pain, arm swelling, and decreased range of motion. The arm can be shortened or rotated in a complete fracture depending on the location of the fracture. A complete neurovascular exam should be performed as with all fractures and dislocations. The radial nerve may be injured during humeral fracture in up to 20% of patients. The injury is usually a neuropraxia and resolves spontaneously in most patients. However, this recovery can take months. Humeral fractures rarely need specific reduction maneuvers for treatment. They should be placed in a sugar tong splint and placed in a sling. Gravity alone is typically successful in fracture reduction. The axillary nerve (A) may be injured during glenohumeral dislocations. The median nerve (B) may be injured during posterior elbow dislocations. Anterior elbow dislocations can be associated with ulnar nerve injury (D).

Which of the following trauma patients meets National Emergency X-Radiography Utilization Study (NEXUS) criteria for avoidance of cervical spine imaging? A 22-year-old man with no posterior midline cervical spine tenderness, with slurred speech and a history of alcohol use A 32-year-old man with no posterior midline cervical spine tenderness with a GCS of 14 A 64-year-old man with no posterior midline cervical spine tenderness nor pain when he turns his head to the left An 18-year-old woman with no posterior midline cervical spine tenderness and with mild numbness to her right arm

Correct Answer ( C ) Explanation: The National Emergency X-Radiography Utilization Study (NEXUS) criteria allow clinicians to clear low-risk patients of c-spine injury clinically, obviating the need for radiography. Midline posterior bony cervical spine tenderness is present if the patient reports pain on palpation of the posterior midline neck from the nuchal ridge to the prominence of the first thoracic vertebra, or if pain can be elicited with direct palpation of any cervical spinal process. Patients are considered intoxicated if any of the following is present: a recent history provided by the patient or an observer of intoxication or ingestion of an intoxicating substance, evidence of intoxication on physical exam (e.g., slurred speech, ataxia, dysmetria, or other cerebellar findings), or any behavior consistent with intoxication. Patients may also be considered intoxicated if tests of bodily fluids are positive for alcohol or drugs that affect the level of alertness. An altered level of consciousness can include any of the following: a GCS of 14 or less; disorientation to person, place, time, or events; an inability to remember three objects at five minutes; or a delayed or inappropriate response to external stimuli. A focal neurologic deficit is defined by any neurologic finding on motor or sensory exam. No precise definition of painful or distracting injury is possible. Determination of this is left to the treating clinician. Therefore, only the 64-year-old man with no posterior midline cervical spine tenderness nor pain when he turns his head to the left can be clinically cleared.

A 52-year-old woman is brought into the ED after her vehicle was struck on the driver's side door while attempting to back out of her driveway. Medics report significant damage to the vehicle with intrusion into the passenger compartment. She complains of pain in her left shoulder and hip. On arrival, her vitals are BP 135/90, HR 90, RR 14, and pulse oximetry 97% on room air. She has tenderness with compression of the iliac crests. A radiograph of the pelvis shows a left superior pubic ramus fracture. Which of the following injuries is also most likely to be identified? Bladder rupture Rectal tear Sacral fracture Urethral disruption

Correct Answer ( C ) Explanation: The bony pelvis is comprised of the ilium, ischium, pubis, sacrum, and coccyx and is stabilized by some of the strongest ligamentous structures in the body. Collectively, these bones form the pelvic ring. Similar to other ring structures (C1 vertebra), stress to the ring with lateral compression force will often result in disruption of the ring at multiple sites. The identification of a single pelvic fracture should compel the clinician to search for additional bony injury, such as a sacral fracture. Sensitivity of plain films for identifying pelvic fractures is < 80%. Pelvic CT scan is recommended when plain films are negative yet clinical suspicion of occult or additional fractures exists. The bony pelvis is also a region of complicated anatomy containing structures of the GI and GU systems, as well as important vessels and nerves. In the evaluation of the pelvic trauma patient, injury to these nonosseous structures should also be considered. Bladder rupture (A) occurs most commonly in severe lateral compression fractures and is unlikely in cases of pelvic trauma without fractures of the bony pelvis. Retrograde cystography with plain films or CT scan is the test of choice to evaluate for bladder injury. Rectal tear (B) is rare in blunt pelvic trauma and requires a significant mechanism. Gross blood on the digital rectal exam should prompt further investigation and surgical consultation. Urethral disruption (D) should be suspected in anterior-posterior compression type pelvic injuries and may be suggested by blood at the urethral meatus, high-riding prostate, and perineal or scrotal hematoma. If any of these findings is identified, it is recommended to perform a retrograde urethrogram prior to Foley catheter placement.

A 24-year-old man presents to the ED after a head-on motor vehicle collision. A radiograph of his cervical spine is obtained as seen above. What is the diagnosis? Cervical strain Extension teardrop fracture Hangman's fracture Jefferson fracture

Correct Answer ( C ) Explanation: The hangman's fracture, or spondylolysis of C2, occurs when the cervicocranium (skull, atlas, and axis) is thrown into extreme hyperextension as a result of abrupt deceleration. Bilateral fractures of the pedicles of the axis (C2) occur with or without dislocation. Although a hangman's fracture is unstable, cord damage is often minimal because the AP diameter of the neural canal is greatest at the C2 level, and the bilateral pedicular fractures permit the spinal canal to decompress itself. Originally described in victims of hanging injury, today it is most often the result of head-on motor vehicle collisions. Cervical strains (A) are one of the most common injuries encountered by people in motor vehicle collisions. It is commonly referred to as whiplash. Patients often complain of pain and tenderness, but this injury is not associated with radiographic findings. The extension teardrop fracture (B) occurs when abrupt extension of the neck causes the anterior longitudinal ligament to pull the anteroinferior corner of the vertebral body away from the remainder of the vertebra, producing a triangular-shaped fracture radiographically similar to the flexion teardrop fracture. Extension teardrop fractures usually occur in the lower cervical vertebrae (C5-C7) from diving injuries. It is considered unstable and is associated with central cord syndrome caused by the ligamentum flavum buckling into the spinal cord. A Jefferson fracture (D) is an extremely unstable injury of C1.

Which of the following is seen in the image above? Epidural hematoma Intracerebral hematoma Subdural hematoma Traumatic subarachnoid hemorrhage

Correct Answer ( C ) Explanation: The image above represents an acute subdural hematoma (SDH). Acute SDHs are associated with a high incidence of underlying brain injury. The presentation varies with the severity of the underlying injury, but patients commonly present with a diminished level of consciousness, headache, and focal neurologic deficits corresponding to the area of injury. If sufficient bleeding occurs, intracranial pressure increases and herniation may occur. The characteristic appearance of an SDH on head CT scan is a collection of hyperdense blood in a crescent-shaped pattern conforming to the convexity of the hemisphere and often extending past cranial sutures. At times, the injury causes a minimal amount of bleeding and the patient does not immediately seek medical care. The SDH undergoes lysis over a period of several days and eventually organizes into an encapsulated mass. The characteristic CT scan appearance of an epidural hematoma (A) is a hyperdense lenticular collection of blood that indents adjacent brain parenchyma and does not extend beyond cranial sutures where the dura is attached. The characteristic CT scan appearance of a traumatic subarachnoid hemorrhage (D) shows increased density within the basilar cisterns. Blood can also be seen within the interhemispheric fissures and sulci. The characteristic CT scan appearance of an intracerebral hematoma (ICH) (B) is a well-defined hyperdense, homogenous area of hemorrhage. An ICH may be detected on the first CT scan immediately after injury but often is not seen for several hours or days. Unlike contusions, ICHs are usually deep in the brain tissue and often become well demarcated over time.

A 42-year-old woman presents to the emergency department with left wrist pain. She slipped and fell on the ice, falling forward onto her left hand. Her wrist radiograph is shown above. What is the diagnosis? Colles' fracture Lunate dislocation Perilunate dislocation Scapholunate dissociation

Correct Answer ( C ) Explanation: The patient has a perilunate dislocation. Perilunate dislocations are part of a class of carpal ligamentous injuries which begin with scapholunate joint disruption and have progressive carpal instability. A stage I injury, or scapholunate dissociation, results in widening of the scapholunate joint on a posterior-anterior (PA) radiograph, dubbed the "Terry Thomas sign" after a British comedian with a gap between his front teeth. A stage II injury, or perilunate dislocation, is best seen on the lateral radiograph of the wrist, showing the capitate bone dislocated and dorsally displaced relative to the lunate. The lunate maintains its articular connection to the radius, distinguishing it from a lunate dislocation. The PA view of a perilunate dislocation shows an overlap of the carpal rows, and may also reveal an associated scaphoid, radial styloid, or capitate fracture. The most common mechanism for a perilunate dislocation is a fall onto an outstretched hand (FOOSH). Pain, swelling, and tenderness are noted over the dorsum of the wrist, particularly in the region of the scapholunate ligament. Patients with suspected perilunate dislocation require emergent orthopedic consultation for open fracture reduction and stabilization. Complications include median nerve compression and avascular necrosis. A stage III injury is similar to a perilunate dislocation but includes a dislocation of the triquetrum. Finally, a stage IV injury, or lunate dislocation, involves rotation of the lunate in a volar direction, with a "spilled teacup sign" of the dislocated lunate on lateral radiographs or the "piece of pie sign" referring to the triangular appearance of the displaced lunate on PA radiographs. A Colles' fracture (A) is a distal radius fracture with dorsal displacement and angulation. Lunate dislocation (B) is characterized by volar rotation of the lunate relative to the radius with a "spilled teacup sign" on lateral radiographs. In this images, the lunate still articulates with the radius, which is characteristic of a perilunate dislocation. Scapholunate dissociation (D) is the first stage of carpal ligamentous injuries and is characterized by widening of the scapholunate space on a PA radiograph

A 16-year-old boy presents complaining of right ankle pain after injuring himself while playing basketball. He states that he "twisted his ankle" when he landed on another players foot trying to get a rebound. He has not been able to walk on his right foot since this occurred two hours ago and cannot do so in the ED. On examination, his pulses are intact. Strength and sensation are normal. The ankle is swollen laterally. He is tender over the lateral malleolus and at the base of the fifth metatarsal. No tenderness or swelling is noted over the proximal tibia and fibula. Which of the following is true regarding emergency department radiographic work-up of this patient? A complete ankle series is the only indicated study A complete foot series is the only indicated study Both a complete ankle and a complete foot series are indicated No radiographic imaging is indicated at this time

Correct Answer ( C ) Explanation: The patient should undergo X-rays of both the ankle and the foot. The Ottawa Ankle Rules (OAR) are a clinical decision making tool used to help determine the need for radiographic imaging following blunt ankle injury. They apply to acute injuries that have occurred within the previous 48 hours, however, are not useful in the subacute or chronic phase. The OAR were not designed to be general guidelines for foot injuries; the foot rules only apply to the midfoot and do not apply to the hindfoot or forefoot. The patient has bony tenderness in the foot and therefore should have a complete series of foot X-rays. By only obtaining a complete ankle series (A), a clinically significant foot injury may be missed. The patient has bony tenderness in the ankle and therefore should have a complete series of ankle X-rays. By only obtaining a complete foot series (B), a clinically significant ankle injury may be missed. According to OAR this patient should have radiographic imaging in the ED (D). The rules do not apply when the patient is acutely intoxicated, difficult to assess, has altered mental status, multiple injuries, diminished sensation, or neurologic deficits.

A 2-year-old girl is brought to the ED by her father who reports that she placed her finger in a wall electrical socket. In the ED, the child is easily consolable but holds her right index finger in the air. Visual inspection reveals localized erythema at the distal tip but good capillary refill. There are no other signs of trauma. Which diagnostic test should be performed next? Basic metabolic panel Chest radiograph Electrocardiogram Urinalysis

Correct Answer ( C ) Explanation: There are two basic types of electrical current: alternating (AC) and direct (DC). This child sustained a low-voltage (<1000 volts) electrical injury from an unprotected wall socket. In the United States, household wiring has 120 volts of alternating current (AC), with a frequency of 60 Hz. Alternating current causes continuous tetanic muscle contraction between 8 and 22 mA that does not allow the individual to release the electrical source. This child sustained a superficial thermal burn and most likely did not become part of the circuit, given that there are no clear entry or exit wounds. However, patients sustaining such an injury still warrant an ECG and cardiac monitoring in the ED. The most common ECG abnormality is sinus tachycardia and nonspecific ST-T wave changes that resolve spontaneously. Atrial or ventricular ectopy, fibrillation, interval prolongations, or blocks may also be seen. Asystole and ventricular fibrillation are more commonly seen in higher-voltage exposures. Although a urinalysis (D) to screen for myoglobin and a basic metabolic panel (A) to check renal function might be performed, it is not necessary, given the nature of this child's injury. Creatine phosphokinase (CPK) and serial cardiac-specific troponins may be obtained if skeletal muscle or cardiomyocyte damage is suspected. A chest radiograph (B) will provide no useful information, given the clinical exam.

EMS calls notifying you that there is a mass casualty situation. There was an explosion at a construction site and they will be bringing 20 ambulatory, well appearing patients to your emergency department. After checking airway, breathing, and circulation, what test, if normal, would allow you to rapidly discharge patients? Abdominal ultrasound Ophthalmoscopic evaluation of the eye Otoscopic evaluation of tympanic membranes Oxygen saturation

Correct Answer ( C ) Explanation: This mass casualty scenario is concerning for blast injuries. The most commonly injured organs in blast injuries are the ears as the tympanic membrane (TM) sustains damage at lower pressure than any other structure. Otherwise asymptomatic patients with normal TM evaluation are at very low risk of having any occult primary blast injury and can be discharged with precautions. Patients with TM injury should be further evaluated for serious pathology. Oxygen saturation (D) should be obtained as part of the continued workup. However, those with pulmonary injuries can have delayed deterioration. Patients with TM rupture and normal oxygen saturation should have a chest X-ray and be monitored for six to eight hours looking for clinical deterioration. Abdominal ultrasound (A) can be used looking for hemorrhage but CT scan is necessary to diagnose hollow organ injuries, which are more common in blast injuries. Ocular injuries are uncommon but ophthalmoscopic evaluation (B) is indicated for ocular complaints.

A 25-year-old man presents after falling face forward off his bike. He sustained an abrasion inside his upper lip and complains of a broken front tooth. He brought the fractured fragment with him. On examination, the bony structures of the jaw are non-tender. There is no malocclusion. Tooth #8 has a fracture and in the center of the exposed area is a small pink dot. What is the most appropriate plan for this patient? Irrigation of the tooth Placement of the tooth fragment in Hank's solution Urgent dental consult Viscous lidocaine for pain control

Correct Answer ( C ) Explanation: This patient has a dental fracture with exposed pulp. This is an urgent dental condition requiring an urgent dental consult. The most superficial dental fractures involve only the enamel on the surface and treatment is mostly cosmetic and aimed at dulling any sharp edges. Fractures that expose dentin will have an ivory-yellow appearance. In younger patients, there is less dentin relative to the pulp and treatment is aimed at protecting any pulp contamination with placement of a calcium-hydroxide dressing. Younger patients need more urgent follow-up with a dentist. The most significant dental fractures involve the pulp as in this clinical scenario. The tooth should be gently wiped clean with gauze and inspected for a drop of blood or pink blush which represents pulp exposures. The area is usually exquisitely painful. Urgent consult in the ED is recommended, but follow-up within 24 hours is absolutely required for evaluation and possible root canal and extraction of the pulp. If dental follow-up will be delayed, the tooth should be covered with moist cotton and sealed with dry foil or a temporary commercial sealant. In order to clean the tooth, a clean gauze should be used to wipe off the surface, not irrigation of the tooth (B) as the patient will be extremely sensitive to water and all efforts should be maintained to avoid pulp contamination. Hank's solution (A) is a physiologic solution in which an avulsed tooth may be placed while waiting for reimplantation into the socket. There is no role in a partial tooth fracture. Viscous lidocaine (D) is not an appropriate analgesic for a dental fracture. Oral analgesics or dental block should be provided for pain control.

A 53-year-old man presents with a burning sensation in both hands and weakness in his arms after a fall. The patient states he fell forward and his neck bent backwards. Physical examination reveals 4/5 strength in both arms. Which of the following is the most likely diagnosis? Anterior cord syndrome Brown-Sequard Syndrome Central cord syndrome Complete spinal cord syndrome

Correct Answer ( C ) Explanation: This patient presents with bilateral upper extremity paresthesias and weakness after an extension injury of the neck consistent with a central cord syndrome. Injury to the central cord commonly occurs after a fall or motor vehicle collision where there is hyperextension at the neck. The syndrome presents with bilateral motor paresis. Sensory impairment may be present as well. The upper extremities are affected more often and to a greater degree than the lower extremities. Additionally, distal muscle groups are more affected than proximal ones. Bladder dysfunction and burning dysesthesias may also be present. The prognosis typically depends on the severity at presentation Anterior cord syndrome (A) is characterized by loss of motor function below the lesion and preservation of position, touch, and vibration sensation. Brown-Sequard syndrome (B) presents with ipsilateral motor function loss and contralateral pain and temperature sensation loss. Complete spinal cord syndrome (D) presents with complete loss of sensory, autonomic, and voluntary motor function distal to the lesion.

Which of the following fractures is always considered an unstable spinal injury? Burst fracture Clay shoveler fracture Flexion teardrop fracture Simple wedge fracture

Correct Answer ( C ) Explanation: To assess the stability of spinal injuries below C2, it is helpful to view the spine as consisting of two columns. Alternating vertebral bodies and intervertebral disks held in alignment by the anterior and posterior longitudinal ligaments form the anterior column. The pedicles, transverse processes, articulating facets, laminae, and spinous processes form the posterior column, which contains the spinal canal. It is held in alignment by the nuchal ligament complex (supraspinous, interspinous, and infraspinous ligaments), the capsular ligaments, and the ligamentum flavum. If both columns are disrupted, the spine will move as two separate pieces, and there is high likelihood of that movement causing or worsening spinal cord injury. If only one column is disrupted, the other column resists further movement, and the likelihood of a spinal cord injury occurring is less and depends on the strength of the intact ligaments. A flexion teardrop fracture results when severe flexion forces cause anterior displacement of a wedge-shaped fragment (resembling a teardrop) of the anteroinferior portion of the involved vertebral body. This fracture involves complete disruption of the ligamentous structures at the level of the injury. This leads to a highly unstable fracture. Vertical compression injuries occur in the cervical and lumbar regions, which are capable of straightening at the time of impact. When forces are applied from either above (skull) or below (pelvis or feet), one or more vertebral body endplates may fracture. The nucleus pulposus of the intervertebral disk is forced into the vertebral body, which is shattered outward, resulting in a burst fracture (A). This is a stable fracture because there is no ligamentous disruption. However, fracture fragments may impinge on or penetrate the ventral surface of the spinal cord and cause an anterior cord syndrome, therefore it is sometimes considered an unstable fracture. A clay shoveler fracture (B) is an oblique fracture of the base of the spinous process of one of the lower cervical segments. Because this injury involves only the spinous process, it is stable and not associated with neurologic involvement. A simple wedge fracture (D) does not involve the posterior column and is therefore considered a stable fracture rarely associated with neurologic injury.

An 18-year-old woman presents with chest pain and shortness of breath after being struck in the chest by a baseball bat. Vitals are HR 133, BP 80/40, and oxygen saturation 91% on room air. On physical examination, the patient has decreased breath sounds on the right side. Which of the following is the most appropriate next step in management? Chest X-ray CT scan of chest Needle thoracostomy second intercostal space, midclavicular line Tube thoracostomy eigth intercostal space, anterior axillary line

Correct Answer ( C ) Explanation: This patient presents with signs and symptoms of a tension pneumothorax (PTX) and should have an emergent needle thoracostomy at the second intercostal space midclavicular line followed immediately by tube thoracostomy. Tension PTX is a clinical diagnosis in which the patient presents (typically after trauma) with chest pain, shortness of breath, and dyspnea. Physical examination reveals hypoxia, decreased breath sounds and frequently, displacement of the trachea away from the affected side. Increased pressure in the hemithorax with the tension PTX causes decreased pulmonary venous return leading to hypotension and tachycardia. These patients should immediately have needle thoracostomy performed by placement of a 14 gauge catheter into the second intercostal space in the midclavicular line. Needle decompression acts to relieve the pressure and can immediately lead to improved blood pressure and oxygenation. Needle thoracostomy must be followed by chest tube placement for definitive drainage. The optimal location for chest tube placement is in the fifth intercostal space in the midaxillary line. Chest X-ray (A) and CT scan of the chest (B) are helpful in diagnosing a PTX but should not delay treatment in a patient with a tension PTX. Chest tube placement (D) is in the fifth intercostal space in the midaxillary line.

A 22-year-old man presents to the ED after being struck in the face with a beer bottle during a bar fight. On physical exam, he has unilateral periorbital ecchymoses, tenderness around the orbit, infraorbital anesthesia, and swelling. Extraocular movements, visual acuity, and intraocular pressure are normal. What is the next step in management? Administer intramuscular tetanus toxoid Discharge patient with return precautions for vision changes Obtain a CT scan of the orbits Place a protective shield over the affected eye

Correct Answer ( C ) Explanation: This patient's presentation is concerning for an orbital fracture. The inferior and medial orbital walls are the thinnest and, consequently, the most frequently fractured. Physical exam findings of an orbital fracture include infraorbital anesthesia, subcutaneous emphysema, and binocular diplopia on upward gaze due to inferior rectus entrapment. However, absence of these findings does not exclude this diagnosis. With a suggestive mechanism and symptoms, CT imaging is indicated to rule out orbital fracture. CT has largely replaced the Waters' view on plain films, which examines for herniation of orbital contents into the maxillary sinus. Orbital fractures are among the most common facial fractures and up to 33% have associated ocular injuries (traumatic iritis, retinal detachment). Tetanus toxoid (A) should be considered in all trauma patients but is not indicated in this patient without associated wounds. Although this patient's exam is reassuring, discharging him home with return precautions for vision changes (B) without imaging may result in missed orbital fractures and other ocular injuries. Eye shields (D) are primarily used to protect open or suspected open globe injuries. Given that this patient has normal acuity and intraocular pressure, an open globe injury is unlikely.

A 3-year-old boy presents with his mother after he tripped and sustained an injury to his mouth. The mother has his avulsed tooth soaking in a container of milk. Which of the following is true concerning the management of dental avulsions? A primary central incisor should be reimplanted A primary tooth avulsion often leads to a cosmetic deformity of the secondary tooth Avulsed teeth should be held by their root to avoid contaminating the crown Milk is an acceptable storage medium for an avulsed tooth

Correct Answer ( D ) Explanation: A dental avulsion occurs when a tooth is completely removed from its socket. Management of recovered avulsed teeth depends on the age of the patient and the length of time that the tooth has been absent from the oral cavity. Avulsed primary teeth in a pediatric patient age 6 months to 6 years are not replaced in the socket. The best environment for an avulsed tooth is in its socket. The greatest success for reimplantation is 30 minutes. The worst situation is to allow the tooth to be transported in a dry medium. Storage in plain water is not much better. Although saliva is a reasonable storage medium, milk is preferable because of its osmolarity and essential ion concentration of calcium and magnesium. The best storage and transport medium is Hanks solution, a balanced pH cell culture medium. Reimplanting primary teeth (A and B) ankylose or fuse to the bone so that although the dentofacial complex grows downward and forward, the reimplantation site does not. There also may be interference with the eruption of the permanent tooth. Hence primary teeth should not be reimplanted. Avulsed teeth should be handled only by the crown (C) so that the remaining periodontal ligament fibers are not damaged.

Which of the following is true regarding hyperventilation in the head-injured patient? It improves oxygenation, which helps to reduce secondary brain injury It induces cerebral vasodilation, which helps to reduce intracranial pressure Target PaCO2 is 20-25 mm Hg The onset of effect is within 30 seconds

Correct Answer ( D ) Explanation: Acute hyperventilation in a severe head trauma patient can be a life-saving intervention. The goal is to prevent or delay herniation until a more definitive procedure can be performed. The onset of effect is within 30 seconds. Hyperventilation is thought to cause vasoconstriction of the cerebral vasculature leading to a reduction of intracranial pressure. However, prolonged hyperventilation is not recommended because it may lead to profound vasoconstriction and ischemia resulting in poorer outcomes. Hyperventilation should be employed only when a patient experiences an acute neurologic decline or demonstrates signs consistent with herniation. Hyperventilation reduces the PaCO2 and is not used to improve oxygenation (A). Preventing hypoxemia is critical in a head-injured patient to reduce secondary brain injury. This can be achieved by establishing a definitive airway and maintaining a PaO2 greater than 80 mm Hg. Hyperventilation induces vasoconstriction (B), not vasodilation. The target PaCO2 is 30-35 mm Hg (C). A PaCO2 less than 25 mm Hg is associated with profound reductions in cerebral blood flow and a worse outcome.

32-year-old man presents after falling off a porch onto the driveway an hour ago. His family reports that he did not lose consciousness, but has become progressively more drowsy since the accident. He has vomited twice. His non-contrast head computed tomography scan is shown above. What is the most likely etiology of this condition? Tear of a bridging vein Tear of a dural sinus Tear of the anterior cerebral artery Tear of the middle meningeal artery

Correct Answer ( D ) Explanation: An epidural hematoma is a collection of blood that forms between the dura and the inner table of the skull. Direct trauma to the skull is often the cause, with the temporoparietal region the most likely site involved. An associated skull fracture is seen in the majority of patients with an epidural hematoma. The source of the bleeding is often arterial and most commonly the result of a tear in the middle meningeal artery. Arterial bleeding allows for rapid expansion of the hematoma and results in symptoms that typically develop within hours of the injury. Patients classically present with a history of a head injury that produces a loss of consciousness followed by a lucid interval. The patient then develops a second episode of decreased consciousness. They may also complain of a severe headache, dizziness, nausea, and vomiting. This classic presentation is present in about half of patients. Diagnosis is made by CT scan which shows a hyperdense biconvex collection of blood. The margins are sharply defined and do not cross the suture lines. Depending on the size of the hematoma, there may be an associated midline shift or signs of herniation. Emergent neurosurgical consultation for evacuation of the hematoma and control of the bleeding is indicated. Long-term prognosis depends on the underlying brain injury but is improved significantly if the hematoma is rapidly detected and evacuated. A subdural hematoma is associated with an acceleration-deceleration injury and is the result of a tear in the bridging veins (A). Head CT will show a crescent-shaped collection of blood that may cross suture lines. A tear of a dural sinus (B) can also cause an epidural hematoma but is less frequent than an injury to the middle meningeal artery. Symptoms typically develop less rapidly in these patients. The anterior cerebral artery (C) is not involved in epidural hematomas.

An 18-year-old man is in a motor vehicle collision and suffers an estimated acute blood loss of two liters. Which of the following statements is correct regarding his condition? He will have decreased urine output but no change in respiratory rate He will have tachycardia but no change in his blood pressure His estimated blood loss is approximately 15% of total blood volume His pulse pressure will be narrowed

Correct Answer ( D ) Explanation: Hemorrhagic shock results from a rapid reduction in blood volume, which causes baroreceptor activation and leads to vasoconstriction, increased cardiac contraction, and tachycardia. The traumatic hemorrhage is classified based on clinical signs and is a useful tool for estimating the percentage of acute blood loss. A two-liter acute blood loss represents about 30-40% of total blood volume (Class III hemorrhage) leading to signs of inadequate perfusion including marked tachycardia and tachypnea, altered mental status, and a fall in systolic blood pressure. This is associated with a slight increase in diastolic pressure resulting in a narrow pulse pressure. No class of hemorrhage has a decreased urine output without changes in respiratory rate (A). Urine output is an indicator of volume status and begins to decline in Class II hemorrhage. In response to a decrease in oxygen carrying capacity caused by acute blood loss, respiratory rate begins to rise. Heart rate (B) begins to increase in Class II hemorrhage, but blood pressure remains unchanged. Blood pressure begins to decline in Class III hemorrhage. Class III hemorrhage is associated with 30%-40% (C) of estimated blood loss.

A 13-year-old girl with no past medical history presents with a 3 cm laceration to the left forearm secondary to a dog bite. The bite occurred 1 hour ago. The wound is superficial with no local crush injury. What management should be pursued3 Irrigation and primary repair Irrigation, no closure Irrigation, no closure, antibiotics Irrigation, primary repair and antibiotics

Correct Answer ( D ) Explanation: Low risk lacerations sustained from dog bites can be primarily closed. The inflicted wound should have a thorough neurovascular examination and should be explored for foreign bodies and injury to deeper structures. If the wound is superficial without presence of significant crush injury or tissue loss, primary closure is the best approach for cosmetic outcomes. Prophylactic antibiotic use is somewhat controversial although most sources recommend them for deep puncture wounds, most wounds requiring closure, moderate to severe crush wounds, wounds involving the hands, genitalia or joints, and in patients who are immunocompromised. Although this is a relatively low-risk wound, given that it meets criteria for primary closure (clinically uninfected, less than 12 hours old, and not on the hand or foot), prophylactic antibiotics are recommended. Therefore, irrigation and primary repair (A) alone is incorrect. Irrigation and no closure (B) or secondary closure is not superior to primary closure. Irrigation, no closure, and antibiotics (C) is incorrect as primary closure in wounds not involving significant crush injury, without signs of infection, and not located on the hand or foot has not been shown to increase the rate of infection, provided the closure is done within 12 hours.

Based on NEXUS criteria, which of the following patients with no cervical spine tenderness is eligible to be cleared clinically? 19-year-old man who presents after a bicycle accident with an open left radius and ulnar fracture 22-year-old man who presents after a motorcycle accident with a GCS score of 13 54-year-old man who presents after a fall down five steps and a serum ethanol level of 140 60-year-old woman with a GCS score of 15 who presents after a slip and fall down 10 steps and sustained a 2 cm superficial laceration noted on her left shin

Correct Answer ( D ) Explanation: The 60-year-old woman who presents after a slip and fall down 10 steps and sustained a 2 cm superficial laceration noted on her left shin is eligible to be "clinically cleared" from her cervical collar and not undergo further cervical spine imaging. The 5 NEXUS criteria include no midline cervical tenderness, no focal neurologic deficit, normal alertness, no intoxication, and no painful, distracting injury. Although the patient sustained an injury, a 2 cm superficial laceration is not considered to be a distracting injury. The first cervical spine rule to be developed was the National Emergency X-Radiography Utilization Study (NEXUS) Low-Risk Criteria (NLC). This developed out of a multicenter prospective observational study of 34,069 patients seen in 21 U.S. emergency departments. The goal being to reduce unnecessary radiation exposure and decrease overall health costs. There are five criteria that are required. When all five are present the patient is low-risk for injury. In the study, only eight of the 818 patients who had an injury (two actually clinically significant) were missed when all the criteria were met. The criteria are 99% sensitive, 12.9% specific, and have a negative predictive value of 99.8%. The 19-year-old man (A) who presents after a bicycle accident with an open left radius and ulnar fracture has a painful, distracting injury (open ulnar and radial fracture). The 22-year-old man (B) who presents after a motorcycle accident with a GCS score of 13 has altered mental status. The 54-year-old man (C) who presents after a fall down five steps and a serum ethanol level of 140 is intoxicated. Therefore, all of these individuals should undergo imaging of the cervical spine and are not eligible to have their cervical spine cleared clinically.

A 62-year-old woman presents complaining of ankle pain following a fall from slipping on ice last night. She is able to walk across the emergency department and back but states that the pain becomes unbearable toward the end. Her posterior lateral malleolus and her anterior medial malleolus are tender to palpation. Which of the following is the correct indication for ankle radiography in this patient? Age Inability to bear weight after six steps Tenderness over the anterior medial malleolus Tenderness over the posterior lateral malleolus

Correct Answer ( D ) Explanation: The Ottawa Ankle Rule is a clinical decision-making rule that was developed to allow clinicians to effectively select patients who are candidates for ankle radiography after acute ankle trauma. It consists of five criteria: tenderness over the posterior lateral malleolus, tenderness over the posterior medial malleolus, tenderness over the navicular bone, tenderness over the base of the fifth metatarsal, and inability to bear weight for four steps. If any one of the criteria is present, the patient is a candidate for plain films of the ankle. Age (A) is not a criterion for ankle radiography according to the Ottawa Ankle Rule, although an age of 55 years or greater is a criterion for knee radiography according to the Ottawa Knee Rule. Inability to bear weight after six steps (B) and tenderness over the anterior medial malleolus (C) are not criteria included in the Ottawa Ankle Rule.

17-year-old woman presents after an isolated head trauma. On primary evaluation, she opens her eyes to painful stimuli, has incomprehensible speech and withdraws to pain. Which of the following represents her Glasgow Coma Scale (GCS) score? A12 B15 C3 D8

Correct Answer ( D ) Explanation: The patient presents after a trauma with head injury and altered level of consciousness compatible with a GCS of 8. The GCS is an objective method of measuring and following the neurologic status of a patient with head trauma. The scale was developed to clinically evaluate head trauma patients 6 hours after trauma. However, the GCS has widely been applied to the acute setting as well. This application is limited by a host of factors including concomitant injuries, vital sign abnormalities, intoxication and age. Despite this, the inter-rater reliability of the GCS makes it a simple tool to communicate mental status in trauma patients across providers. There are three components to the GCS exam: eye opening (maximum 4 points), verbal stimulus response (maximum of 5 points) and motor response (maximum of 6 points). This patient receives 2 points for eye opening (opens to pain), 2 points for verbal stimulus response (incomprehensible speech) and 4 points for motor response (withdraws from pain) giving her a total of 8 points.

A 20-year-old man presents with a gunshot wound to the right flank. Vitals are BP 75/48, HR 133, RR 32, and oxygen saturation 93%. The patient is intubated and packed red blood cells are administered. A bedside FAST examination is negative. Physical examination reveals a single wound to the right flank. What management should be pursued? CT scan of the abdomen Diagnostic peritoneal lavage (DPL) Observation and repeat FAST Transfer to operating room

Correct Answer ( D ) Explanation: The patient presents with a gunshot wound (GSW) to the abdomen and hemodynamic instability mandating an exploratory laparotomy. The diagnostic workup and management of patients with gunshot wounds is complicated in patients with hemodynamic stability. However, management in the unstable patient with a penetrating injury is more straightforward. Initial stabilization should start with airway, breathing, and circulation. Volume resuscitation should be administered in the form of packed red blood cells (PRBCs). The patient should be exposed to search for all wounds as gunshot wounds can often be missed in the axilla and groin if they are not systematically examined. A FAST examination should be performed as it may rapidly reveal the presence of intraperitoneal blood. Regardless of the findings of the FAST examination, patients with gunshot wounds to the abdomen require exploratory laparotomy if they are hemodynamically unstable, have peritoneal signs, gastrointestinal hemorrhage, intraperitoneal air, evidence of a diaphragmatic injury, or signs of evisceration. A CT scan of the abdomen (A) may reveal the exact location of injury but will delay care in this unstable patient. DPL (B) was widely used prior to the FAST examination to identify intraabdominal bleeding after trauma. However, in this patient, the findings of DPL would not change management. Observation and repeat FAST examination (C) can be performed in select patients with penetrating injuries if they do not have any of the previously mentioned hard signs requiring immediate operative care.

A 43-year-old man presents with right hip pain after an MVC and inability to ambulate. Physical examination reveals a right leg that is flexed and internally rotated with strong pulses. An X-ray of his pelvis is seen above. What management is indicated? Admission with urgent orthopedic consultation Application of a pelvic binder Placement of a traction pin Sedation and orthopedic reduction

Correct Answer ( D ) Explanation: The patient presents with a posterior hip dislocation requiring emergent reduction in the Emergency Department. Dislocation of a native hip is a true orthopedic emergency as a delay in reduction of >6 hours leads to avascular necrosis of the femoral head. Native hip dislocations are typically associated with high-energy trauma. A common story is an MVC where a flexed hip strikes the dashboard but it can also be seen in sports injuries. In a posterior dislocation, the leg will be held with the hip flexed, adducted and internally rotated. In an anterior dislocation the hip will be abducted in flexion and external rotation. If the patient has neurovascular compromise and an obvious dislocation, consideration should be made for reduction without delay (including X-rays). In the neurovascularly intact patient, X-rays should be obtained looking for position of the femoral head in relation to the acetabulum and for concomitant fracture. A number of reduction techniques are employed but the key is expedient reduction.Correct Answer ( D ) Explanation: The patient presents with a posterior hip dislocation requiring emergent reduction in the Emergency Department. Dislocation of a native hip is a true orthopedic emergency as a delay in reduction of >6 hours leads to avascular necrosis of the femoral head. Native hip dislocations are typically associated with high-energy trauma. A common story is an MVC where a flexed hip strikes the dashboard but it can also be seen in sports injuries. In a posterior dislocation, the leg will be held with the hip flexed, adducted and internally rotated. In an anterior dislocation the hip will be abducted in flexion and external rotation. If the patient has neurovascular compromise and an obvious dislocation, consideration should be made for reduction without delay (including X-rays). In the neurovascularly intact patient, X-rays should be obtained looking for position of the femoral head in relation to the acetabulum and for concomitant fracture. A number of reduction techniques are employed but the key is expedient reduction.Correct Answer ( D ) Explanation: The patient presents with a posterior hip dislocation requiring emergent reduction in the Emergency Department. Dislocation of a native hip is a true orthopedic emergency as a delay in reduction of >6 hours leads to avascular necrosis of the femoral head. Native hip dislocations are typically associated with high-energy trauma. A common story is an MVC where a flexed hip strikes the dashboard but it can also be seen in sports injuries. In a posterior dislocation, the leg will be held with the hip flexed, adducted and internally rotated. In an anterior dislocation the hip will be abducted in flexion and external rotation. If the patient has neurovascular compromise and an obvious dislocation, consideration should be made for reduction without delay (including X-rays). In the neurovascularly intact patient, X-rays should be obtained looking for position of the femoral head in relation to the acetabulum and for concomitant fracture. A number of reduction techniques are employed but the key is expedient reduction. A native hip dislocation should not be admitted without attempting reduction first (A) as a delay in treatment leads to increased morbidity. Pelvic binders (B) are used when patients have open book pelvic fractures after trauma. Placement of a traction pin (C) is indicated for patients with certain femur fractures.

34-year-old man presents with right side chest pain after a fall from his bike. Physical examination is unremarkable except for tenderness over the right clavicle. A chest X-ray is shown above. Which of the following is the management most likely indicated for this patient? Admit for operative management CT scan of the chest Immediate Orthopedic consultation for closed reduction Sling for comfort and follow up

Correct Answer ( D ) Explanation: The patient presents with an angulated clavicle fracture. He should be placed in a sling for comfort and given follow up with orthopedics. Clavicle fractures account for 5% of adult fractures and are the most common fracture in kids. They are typically sustained after a fall resulting in direct force applied to the lateral aspect of the shoulder and most commonly occur in the middle third of the clavicle. Patients with clavicle fractures present with pain to the area and hold the affected extremity close to the body. The presence of tenting of the overlying skin requires immediate reduction to prevent a closed fracture from turning into an open one. Non-displaced and minimally displaced clavicle fractures typically heal with conservative management including pain control, immobilization (for comfort) with a sling, and follow up with an orthopedic surgeon. Some clavicle fractures will not heal well and require surgery (A) but admission after injury is only required for those with open fractures. CT scan of the chest (B) is only required if other concomitant injuries are suspected. Orthopedic consultation and reduction (C) is only required for clavicle fractures with tenting.

What is the most important initial step in managing the severe head-injured patient? Administer mannitol to reduce intracranial pressure Control scalp hemorrhage Obtain a head CT scan to evaluate for intracranial bleeding Secure the airway

Correct Answer ( D ) Explanation: The primary focus of treatment for patients with suspected severe head injury is to prevent secondary brain injury. Securing the airway is always the critical first step in a patient who has sustained severe head trauma because it is essential to maintain normal oxygen levels. Transient respiratory arrest and hypoxia may occur, which can cause secondary brain injury. If the patient has a depressed mental status (GCS score < 8) or any signs of airway compromise, it is critical to obtain a definitive airway (endotracheal intubation). Once the airway is deemed secure, other aspects of management should occur rapidly. Mannitol (A) is used in patients to reduce elevated intracranial pressure due to a severe head trauma. Indications for its use include acute neurologic deterioration (e.g., development of a dilated pupil, hemiparesis, or loss of consciousness) while the patient is being managed. After securing the airway, attention can be paid to injuries such as a scalp wound, and (B) interventions to control hemorrhage can be pursued. Although most scalp wounds do not lead to hypotension, it is possible to lose large amounts of blood from bleeding scalp wounds, particularly in children. Obtaining a head CT scan (C) is a critical step in the head-injured patient; however, priority is given to airway management. It is not wise to send patients at risk for respiratory failure to the CT scanner where they may decompensate.

A 35-year-old man presents to the ED with shortness of breath and chest pain. Vital signs are blood pressure 132/75, heart rate 106, respiratory rate 22, oxygen saturation 96% on room air. Physical exam is notable for tachycardia and tachypnea with respirations labored and shallow. You also note inward movement of a segment of the chest wall with inspiration and outward movement of the segment of chest wall with expiration. Which of the following is the most likely cause for this patient's diagnosis? Abdominal ascites Chronic illness Malignancy Thoracoabdominal trauma Viral infection

Correct Answer ( D ) Explanation: This individual is presenting with flail chest. Flail chest occurs when a segment of the thoracic cage is detached from the rest of the chest wall. This occurs when contiguous ribs are fractured in multiple places. This creates a segment of the chest wall (the flail segment) that does not move in unison with the rest of the thorax, resulting in paradoxical movement of the chest wall. When the diaphragm contracts and produces negative inspiratory pressure, the flail segment, no longer anchored to the rib cage, tends to move inward and, during expiration, moves outward. Flail chest is associated with significant blunt thoracic trauma and underlying pulmonary contusion. If the flail segment is large enough, mechanical ventilation may be required. Abdominal ascites (A) can cause a decrease in functional residual capacity and subsequent tachypnea; however, it is not associated with paradoxical movement of the chest wall. Many chronic illnesses (B) are complicated by chest pain and shortness of breath; however, paradoxical movement is an unlikely finding. Malignancy (C) is associated with pericardial effusion, which may cause shortness of breath or chest pain, but not paradoxical movement of the chest wall. Viral infection (E) can cause pericarditis, myocarditis, or pneumonia, or lead to the sensation of shortness of breath and chest pain. Retractions may even be seen with severe bronchial inflammation, but neither this nor any other focal infections will produce paradoxical movement of the chest wall.

An 18-year-old man presents to the ED after he slipped off his skateboard and struck his head on the ground. He is awake and oriented but keeps asking, "What happened?" On exam, you note a right-sided scalp hematoma. A CT scan of the head is obtained, and the above finding is noted. Which of the following is the most appropriate next step in management? Emergency decompression by trephination Hyperventilate with bag-valve mask Neurology consultation and supportive care Neurosurgical consultation

Correct Answer ( D ) Explanation: This is an epidural hematoma with resultant midline shift of the brain. The biconvex (football) shape and hyperdense appearance on computed tomography is classic for an epidural hematoma. This injury requires prompt neurosurgical evaluation and operative intervention to evacuate the hematoma, decompress the shift, and control arterial bleeding. If surgery is performed before clinical deterioration, the functional outcome is generally excellent; however, without operative management, mortality can be as high as 100%. The patient requires decompression, but because there are no clinical signs of brainstem herniation, emergency trephination (Burr hole) (A) is not indicated. Hyperventilation (B), hypertonic saline, and Mannitol are all reserved for patients with increased intracranial pressure, impending or evolving herniation (abnormal posturing, coma, contralateral hemiparesis, ipsilateral nonreactive, dilated pupil). This patient is perseverating (repetition of a particular response); a sign of traumatic brain injury not specific for increased intracranial pressure, epidural hematomas, or herniation. Although the hemorrhage appears small, this is a true neurosurgical emergency. Neurology consultation (C) will unnecessarily delay definitive care and it is not appropriate.

A 70-year-old woman presents to the ED with right wrist pain after she tripped and fell at the grocery store. There is swelling on the volar aspect of the wrist. Radiographs are obtained. Which of the following statements regarding this injury is correct? Carpal fractures are common The distal radius ulnar joint is frequently involved. The fracture is associated with dorsal displacement The median nerve is at risk for injury

Correct Answer ( D ) Explanation: This patient sustained a flexion-type (Smith's) fracture often described as a reverse Colles' fracture. In general, pain and swelling will be apparent over the volar aspect of the wrist. The clinical appearance of this fracture is often described as a garden spade deformity. The median nerve is at risk for injury in a Smith's fracture. Treatment includes reduction and immobilization in a long-arm or sugar-tong splint with orthopedic referral. Unstable and intra-articular fractures will ultimately require operative intervention. Associated carpal fractures (A) are uncommon. The distal radius ulnar joint (DRUJ) (B) is not commonly involved in a Smith's fracture. It is more commonly involved in Colles' fractures. Dorsal displacement (C) of the distal radius fracture is termed a Colles' fracture. A Smith's fracture is displaced volarly.

A 42-year-old farmer presents by ambulance after being pinned against a wall by a tractor. He was non-ambulatory at the scene. On arrival to the ED, his vitals are BP 95/60, HR 125, RR 18, and pulse oxygenation 98% on room air. He is pale and diaphoretic, complaining of severe pain in his hips and groin. A FAST exam shows no intraperitoneal or intrathoracic free fluid and no evidence of pericardial effusion. His pelvic radiograph is seen above. What is the most appropriate next step in the management of this patient? AObtain a retrograde urethrogram BPerform a digital rectal exam to evaluate the prostate CRock and compress the pelvis DWrap the pelvis with a bed sheet across the trochanters

Correct Answer ( D ) Explanation: Wrapping the pelvis may be life-saving in a patient presenting with signs of hemorrhagic shock and radiographic evidence of significant pelvic ring disruption (i.e., widening of the pubic symphysis). In trauma patients with shock and pelvic fractures, it is important to first exclude other sites of hemorrhage before attributing shock to pelvic hemorrhage. Wrapping the pelvis helps to minimize the size of an expanding pelvic hematoma by reducing the volume of the pelvis and helping to tamponade disrupted vessels. Although the latter will not stop arterial bleeding, the vast majority (> 80%) of hemorrhage associated with a fractured pelvis comes from the low-pressure venous system, a circumstance amenable to "sheeting" the pelvis. Although a severe anterior-posterior (AP) compression, or "open-book" pelvic fracture, as pictured above, will often result in palpable bony instability of the pelvis, it is important to avoid rocking the pelvis (C). This allows for greater expansion of a hematoma. Instead, the recommended physical exam maneuver to assess for bony stability is to only laterally compress the pelvis. If the iliac crests are mobile, they should be held in compression until the pelvis can be wrapped. Additionally, if instability is noted on exam, it should be diagnosed only once because repeat examination can worsen the vascular disruption. The retrograde urethrogram (A) is used to assess for urethral disruption, a commonly associated injury in cases of AP compression of the pelvis. However, this exam should not be performed in hemodynamically unstable patients. Similarly, a digital rectal exam (B) can provide valuable information about possible associated nerve, bowel, or urethral injury, but it should not delay wrapping the pelvis and initiating resuscitation in hemodynamically unstable patients.

One Step Further Question: How many mL of acute blood can accumulate in the pericardial sac before clinical tamponade sets in?

Answer: From 60 to 100 mL.

One Step Further Question: What antigenic marker is iritis associated with?

Answer: HLA-B27, which is also strongly linked to spondyloarthritis such as ankylosing spondylitis.

One Step Further Question: Other than cosmetic outcome, what are the 2 primary goals of laceration repair of the ear?

Answer: Rapid coverage of auricular cartilage to limit infection and prevention of hematoma.

One Step Further Question: What is Stener's lesion?

Answer: Soft tissue interposition from the adductor aponeurosis associated with a ulnar collateral ligament rupture.

One Step Further Question: What chronic finger deformity can result from an untreated mallet finger?

Answer: Swan neck deformity.

One Step Further Question: What is the most common ulna fracture?

Answer: The midshaft ulna fracture, often occurring due to a "nightstick" type injury mechanism.

One Step Further Question: Is Osteonecrosis more likely with fractures of the scaphoid in the proximal or distal pole?

Answer: The more proximal the fracture, the greater the likelihood of osteonecrosis.

One Step Further Question: If a patient with globe trauma must be intubated, should you worry about the choice of paralytic?

Answer: There is a theoretical concern that succinylcholine may increase intraocular pressure and promote further extrusion. An alternative agent should be considered.

One Step Further Question: What is the normal ICP?

Answer: 15 mm Hg.

One Step Further Question: What method can be used as a quick screen triage for victims of blast injuries?

Answer: Check for tympanic membrane rupture; no rupture is associated with a low chance of hollow organ injury

Which of the following radiographic signs supports the diagnosis of a Galeazzi fracture? Distal radioulnar joint dislocation Midshaft ulna fracture Radial head dislocation Radial head fracture

Correct Answer ( A ) Explanation: A Galeazzi fracture is characterized by a fracture of the distal third of the radial shaft and disruption of the distal radioulnar joint (DRUJ). The mechanism of injury is normally a fall onto an outstretched hand with forced pronation or, less commonly, a direct blow to the forearm. Although the radial fracture is usually obvious both clinically and radiographically, the distal radioulnar joint can appear normal on radiographs despite subluxation or dislocation. A concurrent ulnar styloid fracture is common. Because of the instability of the fracture, operative fixation is usually indicated. Midshaft ulna fractures are associated with Monteggia fracture dislocations in which there is a midshaft ulna fracture (B) and dislocation of the radial head (C). Radial head fractures (D) are relatively common in adults. They typically occur from a fall on an outstretched hand.

Which of the following patients requires emergent consultation with an orthopedic specialist? A 36-year-old woman with a femur fracture and distal cyanosis A 58-year-old man with extension-based chronic low back pain and leg numbness A 75-year-old ambulatory woman with a tibial osteosarcoma An 18-year-old man with a reduced sacral ala fracture and BP 130/86

Correct Answer ( A ) Explanation: All of these patients are potential referrals. However, emergent referral is needed when there is a fracture with neurovascular symptoms or suspected compartment syndrome, or a fracture which is open, displaced, unstable, or irreducible. Distal cyanosis in the setting of a femur fracture strongly suggests vascular compromise (likely femoral artery damage). The 18-year-old's normotension suggests clinical stability (D). Although the sacral ala (wing) lies near large vessels (internal iliac artery and its many branches) and the lumbosacral plexus, we are told that the fracture has been reduced. Routine referral is all that is required for chronic, nonprogressive neurological symptoms (B). It is likely the 58-year-old has spinal stenosis. An orthopedist, as well as an oncologist, would likely be involved in the 75-year-old's care (C). Workup, however, will proceed as an outpatient.

Which of the following wounds is a potential candidate for primary closure? Facial laceration from a dog bite Laceration on sole of the foot after stepping on an unknown object in a stream Laceration over the metacarpal-phalangeal joint sustained during a fistfight Puncture wounds on the leg as a result of a dog bite

Correct Answer ( A ) Explanation: Although dog bite wounds are high risk for infectious complications, those that are cosmetically deforming are often closed primarily, especially on the face. Dog bites are high risk for infection from organisms commonly found in the dog's mouth, which include S. aureus, P. multocida, and S. viridans. Although controversy exists over management of animal bite wounds, generally it is agreed that wounds that are cosmetically deforming (including those on the face) should be repaired primarily. All bite wounds must be thoroughly irrigated and cleansed. Wounds on the extremities and that are not as cosmetically concerning are generally left open or loosely approximated Certain lacerations are best managed when left open or closed in a delayed fashion. These include lacerations that are heavily contaminated or have extensive associated tissue damage, human bites, animal bites that are not cosmetically significant, and contaminated lacerations on the bottom of the feet. A laceration on the plantar aspect of the foot sustained on an unknown object while walking in a stream (B) would be considered contaminated. Lacerations over the MCP joints (C) that occur during a fistfight are considered to be human bites and should not be closed primarily. Puncture wounds on the leg (D) from a dog bite are considered high risk for infection and are not cosmetically significant, therefore should not be closed primarily.

A 27-year-old man presents after a fall. He struck his leg on a table and sustained a fracture of the fibular neck. Which of the following nerves is commonly injured in this type of injury? Common peroneal Femoral Posterior tibial Saphenous

Correct Answer ( A ) Explanation: Fractures of the fibula usually occur as a result of direct blunt trauma or varus stress to the knee. Isolated fractures of the fibula are frequently inconsequential because the fibula is almost completely a non-weight-bearing bone. The exception to this is the significant injury of a Maisonneuve fracture, in which the proximal fibular fracture is associated with disruption of the medial portion of the ankle joint. The common peroneal nerve passes around the neck of the fibula and its function must be evaluated in the setting of a proximal fibula fracture. The common peroneal nerve is a branch of the sciatic nerve and divides into the deep and superficial branches. These nerves provide sensation to the anterior and lateral portions of the leg and to the top of the feet. The motor innervation is for dorsiflexion of the foot. As a result, a patient with a peroneal nerve injury will develop a foot drop. The femoral nerve (B) is the largest branch of the lumbar plexus arising from L2-L4. It emerges from the psoas muscle in the upper thigh and does not anatomically cross near the proximal fibula. The posterior tibial nerve (C) is the larger branch of the sciatic nerve (along with the peroneal nerve). In the popliteal fossa, the posterior tibial nerve travels with the popliteal vein and artery and gives off branches innervating the gastrocnemius, popliteus and soleus muscles. The saphenous nerve (D) is a cutaneous branch of the femoral nerve. It travels along the medial aspect of the leg along the tibia and is not at risk of injury from a fibular fracture.

A patient arrives to the ED 15 minutes after being involved in a motor vehicle collision. He is conscious, and there is no obvious trauma. He is immobilized on a long spine board with a cervical collar in place. His BP is 60/40 mm Hg and HR is 40 bpm. His skin is warm. Which of the following is true regarding his condition? Hemodynamic compromise is expected The condition is associated with neuropraxia from an incomplete spinal cord injury This state is permanent; patients who survive will require long-term vasoactive medications Vasoactive medications should be avoided

Correct Answer ( A ) Explanation: Neurogenic shock occurs after an injury to the spinal cord. Neurogenic shock implies a hemodynamically compromised patient. It is manifested by hypotension and bradycardia. Shock tends to occur more commonly in injuries above T6, secondary to disruption of the sympathetic outflow and to unopposed parasympathetic tone. Loss of vascular tone leads to hypotension. Rarely, atrioventricular block is present. Treatment of neurogenic shock is with fluid and vasopressors. Neurogenic shock must be differentiated from "spinal" shock, which refers to neuropraxia (B) associated with incomplete spinal cord injuries. Spinal shock is a state of transient physiological reflex depression of cord function below the level of injury with associated loss of all sensorimotor functions. Flacid paralysis, with bowel incontinence and urinary retention, is observed, and sometimes accompanied by sustained priapism. These symptoms tend to last several hours to days, rarely weeks, until the reflex arcs below the level of injury begin to function again. This state is transient (C) and resolves in one to three weeks. Alpha-1 vasopressors (e.g., phenylephrine), in addition to dopamine, norepinephrine, and epinephrine (D), should be used to maintain blood pressure and ensure organ perfusion.

A 47-year-old man is thrown violently against the steering wheel of his truck during a motor vehicle collision. On arrival to the ED, the patient is complaining of severe chest pain. His BP is 60/40 mm Hg and his respiratory rate is 40 bpm. Which of the following will best indicate a tension pneumothorax as the cause of this patient's hypotension rather than cardiac tamponade? Decreased breath sounds ECG Jugular venous pressure Pulsus paradoxus

Correct Answer ( A ) Explanation: The life-threatening injuries that occur with blunt chest trauma include massive hemothorax, tension pneumothorax, and cardiac tamponade. It is critical to be able to clinically differentiate these conditions and identify the cause of this patient's hypotension to prevent cardiopulmonary collapse. There are many similarities on exam seen with a tension pneumothorax and cardiac tamponade, but one major difference is that a tension pneumothorax is associated with decreased breath sounds on the side of the injury. The breath sounds in patients with cardiac tamponade should be present and clear. The ECG (B) in cardiac tamponade may exhibit electrical alternans, defined by the changing amplitude or morphology of the QRS complex in a single lead every other beat. Although this is pathognomonic, it is rarely seen. There are no characteristic ECG changes seen in patients with a tension pneumothorax. Additionally, this patient requires immediate intervention, and an ECG is a time-consuming procedure. Elevated jugular venous pressure (C) is present in both cardiac tamponade and tension pneumothorax. However, in severely hypotensive patients, it may be absent with either condition. Pulsus paradoxus (D) is defined as a drop of more than 10 mm Hg in systolic blood pressure with spontaneous inspiration. This is seen in patients with cardiac tamponade and those with chronic pulmonary disease. It is not characteristic of tension pneumothorax.

A 15-year-old boy presents with altered mental status after being hit in the head with a baseball bat. His vital signs are BP 188/110, HR 42, RR 10, and oxygen saturation 99%. What imaging modality will rapidly diagnose this patient's disorder? CT scan of the brain CT scan of the cervical spine Electrocardiogram MRI of the brain

Correct Answer ( A ) Explanation: The patient presents with head trauma and Cushing's reflex indicating the presence of increased intracranial pressure (ICP), the cause of which will most rapidly be found on a non-contrast head CT scan. Cushing's reflex describes the presence of progressive hypertension with bradycardia and diminished respiratory effort in response to increases in ICP. An easy way to remember this is, pressures are high (e.g., blood pressure) and rates are low (e.g., heart and respiratory rate). This triad indicates that ICP is at life-threatening levels and requires immediate identification and intervention to prevent herniation. Patients with impending herniation can be temporarily treated with hyperventilation or administration of osmotic substances to decrease ICP while waiting for definitive management. CT scan of the head without contrast is the first modality that should be utilized as it is rapid and has a high likelihood of finding the culprit lesion. The patient's symptoms are suggestive of a central nervous system process and will not be localized by CT scan of the spine (B). Bradycardia is part of the Cushing reflex. The ECG (C) will show bradycardia but this is due to the increased intracranial pressure and not a primary cardiac etiology. An MRI of the brain (D) will likely show the abnormality causing the symptoms but this procedure takes considerably longer to perform making it suboptimal for this critical patient

A 19-year-old man presents with eye pain and photophobia in his left eye after being punched there two days ago. He describes blurry vision. On exam, you note consensual photophobia. Which of the following is the most appropriate treatment? AHomatropine drops BTimololYour Answer CTopical ketorolac drops DTopical vasoconstrictor

Correct Answer ( A ) Explanation: This patient has traumatic iritis. This is an inflammation inside the eye that occurs after blunt trauma. It is also referred to as iridocyclitis because both the iris and the underlying ciliary body are inflamed. Primary treatment consists of a cycloplegic such as homatropine, which paralyzes the ciliary body resulting in a nonreactive, dilated pupil. Topical steroids may also be used to reduce inflammation but should be administered in consultation with an ophthalmologist and only after an infection has been ruled out. Timolol (B) is used in acute angle glaucoma to decrease aqueous humor production. Topical ketorolac drops (C) may be useful for pain management in other ocular conditions, but they have a limited role in the management of traumatic iritis. Topical vasoconstrictors (D) are helpful in patients with scleritis.

A 55-year-old construction worker presents to the ED after a fall from 20 feet while at work. Per EMS, the patient was confused when they found him with a large hematoma over the right temporal area, swelling of the right maxilla, and deformities to the right shoulder and knee. Appropriate spinal precautions were initiated prehospital. On arrival at the ED, his GCS score is eight with a blood pressure of 162/96, heart rate of 72, and oxygen saturation of 100% on a non-rebreather mask. Which of the following statements is correct regarding the management of this patient's airway? Attempt rapid sequence intubation with etomidate and succinylcholine Cervical spine radiographs should be obtained prior to establishing a definitive airway since the patient's oxygen saturation is 100% Continue oxygenation via non-rebreather face mask and immediately obtain a CT scan of the brain followed by neurosurgical consultation Lidocaine administration is contraindicated due to a paradoxical elevation in intracranial pressure Nasotracheal intubation is an appropriate alternative to orotracheal airway

Correct Answer ( A ) Explanation: Trauma patients with a GCS score less than or equal to eight require immediate airway management. It is suspected that even a single episode of hypoxia in the patient with severe head trauma leads to a poorer prognosis. This patient should be endotracheally intubated using etomidate and succinylcholine. Etomidate is an ideal induction agent in the head-trauma patient. Etomidate has been shown to decrease cerebral oxygen consumption, cerebral blood flow, and intracranial pressure but appears to have minimal effects on cerebral perfusion pressure. Airway management takes priority in this scenario. Given the patient's GCS score of eight in the setting of polytrauma, it is recommended to establish a definitive airway. During endotracheal intubation, the patient's cervical spine should be immobilized to prevent any further injury to the spinal cord. As long as proper cervical spine precautions are taken, cervical radiographs (B) can be obtained after the patient is stabilized. Achieving this, however, can occur with in-line traction and does not require immobilization using a hard collar. Although epidural hematoma is a strong consideration, it is unsafe to take the patient to head CT (C) without first securing the airway. Consulting neurosurgery for patients with severe head trauma is prudent and can occur prior to the return of CT scan results. But the initial priority in such patients is establishment of a definitive airway. There is a reflexive response to laryngoscopy and intubation that increases intracranial pressure, although the precise mechanism is poorly understood. Intravenous lidocaine (D) is thought to reduce intracranial pressure and blunt the response to laryngoscopy and intubation. Although recent reports have questioned the clinical benefit, administration of lidocaine during the pretreatment phase of rapid sequence induction for head injury patients remains a component of current ATLS guidelines. The nasotracheal airway (E) should not be attempted in patients with midface trauma or potential basilar skull fracture because the tube may inadvertently penetrate the intracranial space.

A 17-year-old man presents to the ED with left ear pain after wrestling with his friends in his backyard. On exam, you note the finding in the image above. Which of the following is the most appropriate next step in management? CT imaging of head with thin cuts of the temporal bone Incision and drainage, followed by pressure dressing with next-day follow-up Needle aspiration, followed by loose dressing application and next-day follow-up Symptomatic care with analgesia, ice packs, and urgent referral to plastic surger

Correct Answer ( B ) Explanation: Auricular hematomas in the subperichondrial potential space can occur after blunt trauma to the ear. This is a common injury that occurs in wrestlers. Without proper treatment, the cartilaginous structures of the ear can become permanently deformed resulting in cauliflower ear. The recommended treatment modality is drainage of the hematoma by making a small incision in the helix or antihelix, followed by application of a pressure dressing to maintain the contours of the ear. Next-day follow-up to ensure hematoma reaccumulation has not occurred is also encouraged. CT imaging of the head (A) is not required unless there is suspicion for underlying intracranial injury. Although still widely used, needle aspiration (C) is no longer recommended by many sources because of hematoma reaccumulation. The aspiration is often inadequate, and the hematoma requires additional management. In addition, a pressure dressing, rather than a loose dressing, should be applied. In very small hematomas, symptomatic care (D) might be advisable. Since auricular hematomas are the prelude to the development of a cauliflower ear, it is important to have a low threshold to drain the hematoma.

A 73-year-old man presents after a fall. His GCS score is 14 and vital signs are within normal limits for his age. An image of his ear canal is shown above. What other physical examination finding is associated with this injury? Crepitus over the parietal bone Periorbital ecchymosis Scalp hematoma Septal hematoma

Correct Answer ( B ) Explanation: Basilar skull fractures are often diagnosed clinically based on characteristic findings on the physical examination. Basilar fractures are linear breaks in the base of the skull, most commonly through the temporal bone. The injury may also cause a dural tear. The presence of a basilar skull fracture implies significant force in the traumatic injury although does not necessarily predict significant intracranial pathology. The image above demonstrates hemotympanum, a collection of blood behind the tympanic membrane visualized on inspection of the ear canal. Periorbital ecchymosis is also known as the Raccoon sign. Other physical examination findings associated with a basilar skull fracture include blood in the ear canal, rhinorrhea, otorrhea, retroauricular hematoma (Battle sign), cranial nerve deficits (facial paralysis, tinnitus, impaired hearing, nystagmus). Rhinorrhea and otorrhea with clear liquid suggest a CSF leak secondary to the basilar skull fracture. Patients with a CSF leak are at risk of developing meningitis and some suggest administering prophylactic antibiotics. Depending on the location of the injury, cranial nerves may become entrapped if they pass through the basal foramina. The fracture is diagnosed definitively on CT scan of the head, often ordered with thin cuts through the temporal bone. Crepitus over the parietal bone (A) is not associated with a basilar skull fracture. Crepitus implies subcutaneous air or possibly the sensation when broken pieces of bone rub together. The parietal skull is not the location of a basilar skull injury. A scalp hematoma (C) is often associated with an underlying skull fracture. In children under the age of two, the presence of a scalp hematoma anywhere on the skull with the exception of the forehead is an indication to perform imaging to rule out a fracture or intracranial injury. A septal hematoma (D) is seen on inspection of the nasal cavity with nasal injuries. Patients with nasal trauma need the nares inspected to identify a septal hematoma. In order to prevent necrosis of the septum, the hematoma must be drained.

A 49-year-old woman presents to the ED with right shoulder pain after falling from her bicycle while riding at a high speed. Her exam is unremarkable except that her right arm is locked in abduction over her head as seen in the image above. Which of the following is the likely diagnosis? Anterior shoulder dislocation Inferior shoulder dislocation Posterior shoulder dislocation Sternoclavicular joint dislocation Superior shoulder dislocation

Correct Answer ( B ) Explanation: Inferior shoulder dislocations are uncommon but can be quite serious. This dislocation is also referred to as luxatio erecta meaning, to place upward, which refers to the characteristic presentation of the arm in this image. This injury is unlikely to be missed on clinical exam because the patient holds the arm elevated at 180 degrees and cannot adduct it. It looks similar to a person asking a question. The humeral head can be palpated along the lateral chest wall. Inferior shoulder dislocations are always accompanied by detachment of the rotator cuff. The axillary artery and brachial plexus are often injured as well because the humeral head tears through the inferior capsule. Inferior shoulder dislocations are generally managed by traction-countertraction reduction under procedural sedation. Anterior dislocations (A) are the most common shoulder dislocation and one of the most common joint dislocations seen by an emergency provider. The patients usually have their arm to the side holding it in slight abduction and external rotation. There is usually a deformity seen at the glenohumeral joint, and a sulcus sign (a visible gap along the lateral edge of the acromion process) is often noted. Posterior dislocations (C) are associated with forceful trauma such as high-speed motor vehicle collisions, seizures, and electric shock. Subtle to detect on radiographs, these injuries are commonly missed. On exam, the patients arm will be in adduction with internal rotation, and will be unable to externally rotate it. Sternoclavicular joint dislocation (D) can be either anterior or posterior and involves complete rupture of the sternoclavicular and costoclavicular ligaments. Anterior dislocations are mainly cosmetic, posterior dislocations have the potential to cause serious complications such as pneumothorax, laceration of the superior vena cava, occlusion of the subclavian artery, and rupture of the trachea. Superior shoulder dislocations (E) are extremely rare and occur from an extreme force in the superior and anterior direction. The arm is usually held against the body in slight internal rotation.

A 29-year-old man presents to the ED after a MVC. A pelvic fracture is identified on radiography. His vital signs are stable. The decision is made to place a Foley catheter, but blood is noted at the urethral meatus. Which of the following is an appropriate next step? Consult a urologist Obtain a CT scan to evaluate for urethral injury Perform a retrograde urethrogram Place a condom catheter

Correct Answer ( C ) Explanation: In general, a Foley catheter should not be placed in the setting of suspected urethral injury. In such cases, it is recommended that further testing be performed to evaluate for urethral injury. A retrograde urethrogram should be performed. If there is no contrast extravasation, then a Foley catheter can safely be inserted. A urologist should be consulted (A) if a urethral injury is confirmed by the retrograde urethrogram. A CT scan (B) is a poor study to identify urethral injuries. A condom catheter (D) does not allow for accurate urine output measurements and may delay identification of a urethral injury.

An 18-year-old woman who was a restrained driver presents to the ED after a moderate-speed head-on collision and is complaining of lower back pain. Vital signs are T 37.2°C, BP 120/65 mm Hg, HR 85 BPM, and RR 16. Physical exam reveals an alert female with a soft abdomen and slight paraspinal lumbar pain to palpation. FAST exam performed four hours after presentation is negative for free fluid. Urine pregnancy test is negative, and the microscopic urinalysis reveals 25 RBC/HPF. What is the next appropriate step in the management of this patient? CT scan with contrast of the abdomen and pelvis Discharge with urology follow-up Foley catheter placement and discharge Retrograde urethrogram

Correct Answer ( B ) Explanation: Microscopic hematuria is defined as a >10 RBCs per high-power field (HPF) without evidence of gross blood. Most experts believe that there is limited utility for exact quantification of RBC content by microscopic analysis unless associated with systolic hypotension. In general, however, the presence of microscopic hematuria is not considered indicative of severe underlying renal trauma and such patients can be safely discharged with urology follow-up. CT scan with contrast (A) is recommended in the setting of gross hematuria, signs of shock, or when there is suspicion of an intra-abdominal injury. Placing a Foley catheter (C) is indicated in cases of urinary retention or select urethra injuries when done with direct visualization by a urologist. Retrograde urethrogram (D) is indicated for suspected urethra injury.

An 18-year-old woman presents to the ED immediately after rear-ending another vehicle. She states she is six months pregnant. Her vital signs are heart rate 95 bpm, blood pressure 102/74 mm Hg, respirations 18/min, and oxygen saturation 98% on room air. She has very mild abdominal pain and bedside FAST exam is negative. On bedside ultrasound, you identify the fetus and note a heart rate of 140 bpm with fetal activity. What is the next appropriate step in management? Administer Rho(D) immunoglobulin Begin cardiotocographic monitoring Discharge the patient with next day follow-up with her obstetrician Perform a Kleihauer-Betke test

Correct Answer ( B ) Explanation: Minor trauma does not exempt the fetus from significant injury. It is estimated that 1% to 3% of all minor trauma results in fetal loss, most commonly from placental abruption. Once the mother is stabilized, the focus of care is on the fetus. For the fetus greater than 24 weeks gestation, continuous cardiotocographic monitoring is the next step. Cardiotocography is more sensitive for detecting placental abruption than is ultrasound. If the patient is asymptomatic and not experiencing uterine contractions and fetal heart tracing is reassuring after four hours of monitoring, she can be discharged with obstetrical follow-up. Rho(D) immunoglobulin (A) is administered to Rh-negative mothers when there is a possibility of feto-maternal hemorrhage to prevent sensitizing the mother to Rh antigen. Sensitization can be prevented if the Rho(D) immunoglobulin is administered within 72 hours of trauma. When >24 weeks gestation, all pregnant patients who sustain blunt trauma should undergo cardiotocographic monitoring due to the risk for placental abruption; next day follow up is not appropriate (C).The Kleihauer-Betke test (D) can quantify the volume of feto-maternal hemorrhage by identifying fetal hemoglobin within the maternal circulation. The main utility of the test is to help determine the dosing for Rho(D) immunoglobulin in Rh-negative trauma patients.

A 20-year old man presents after jamming his index finger during a basketball game. When he tries to straighten his finger, the distal tip remains flexed as shown above. Which of the following describes the pathophysiology of this injury? Defect of the central slip causing volar migration of the lateral bands Disruption of the extensor tendon mechanism at the distal interphalangeal phalangeal (DIP) joint Disruption of the ulnar collateral ligament Inflammation of the abductor pollicis longus and extensor pollicis brevis tendons

Correct Answer ( B ) Explanation: The patient has a mallet finger. A mallet finger is characterized by a closed disruption of the distal extensor apparatus, often occurring when there is sudden forceful flexion of an extended finger, such as when struck by a ball. Because the disrupted extensor tendon no longer connects with the distal phalanx, the DIP joint is held in flexion due to unopposed action of the flexor digitorum profundus. A mallet finger injury is the most common hand injury seen in athletes. On examination, the distal tip of the finger is flexed at the DIP joint. Pain, swelling, and tenderness may be present over the DIP joint. Passive extension is usually intact, but patients are unable to actively extend the distal phalanx. Radiographs may show an associated dorsal avulsion fracture. The treatment involves immobilizing the DIP joint in slight hyperextension for 6-8 weeks to allow tendon healing. The proximal interphalangeal (PIP) joint and metacarpophalangeal (MCP) joints should be allowed to move freely. A defect of the central slip causing volar migration of the lateral bands (A) is seen in a boutonniere deformity. Disruption of the ulnar collateral ligament (C) is the mechanism leading to gamekeeper's thumb. Inflammation of the abductor pollicis longus and extensor pollicis brevis tendons (D) is seen in de Quervain's tenosynovitis.

A gravid female reportedly in the third trimester arrives via EMS after being struck by a car at a high speed while she was crossing the street. On the radio, paramedics report that they are performing CPR because the patient has no vital signs. Accordingly, a perimortem c-section is planned as soon as the patient arrives. Which of the following would result in the highest likelihood of survival for the fetus? Bedside ultrasound demonstrates fetal heart of 60 Full-term fetus Performance of a Pfannenstiel incision Prolonged CPR

Correct Answer ( B ) Explanation: The primary goal of a perimortem cesarean section is fetal and maternal survival. This procedure should be performed only when the fundal height is above the umbilicus as fetal prognosis improves with advanced gestational age. A full-term fetus increases the chances of survival in this situation due to its greater ability to tolerate hypoperfusion and anoxia compared to a preterm fetus. A bedside ultrasound (A) will cause unnecessary delay in delivering the fetus and is not needed when perimortem c-section is planned. Survival of the mother and infant is greatest when the procedure is performed within five minutes of maternal arrest. Continuous CPR is important for resuscitation of mother and fetus; however, if the mother sustained blunt trauma and is pulseless, her mortality is near 100%. Prolonged CPR (D) (greater than 25 minutes) results in near certain nonviability of the fetus. A transverse lower uterine, also known as a Pfannenstiel incision (C), is the standard obstetrical approach for elective cesarean sections because it provides the best cosmetic outcome. As this is not a concern in the perimortem patient, a large vertical midline incision should be performed from the symphysis pubis to 4-5 cm below the xiphoid process. The incision should be deep enough to incise through all the layers of the abdominal wall down to the peritoneal cavity. After the bladder is reflected inferiorly, a vertical incision in the uterus should be made large enough to deliver the fetus.

A 21-year-old man presents with a stab wound to the right chest. His vitals are HR 157, BP 81/43, RR 28, and oxygen saturation 91%. The patient is intubated and packed red blood cells are started. Physical examination reveals a bleeding wound to the right chest, a midline trachea and decreased breath sounds on auscultation of the right hemithorax. Which of the following should be performed next? Application of a pelvic binder Placement of a right thoracostomy tube Right chest thoracotomy Transfer to operating room

Correct Answer ( B ) Explanation: This patient has a stab wound to the right chest that may be causing hemorrhagic shock from a hemothorax or obstructive shock from a pneumothorax or pericardial tamponade. The presence of a midline trachea makes pneumothorax as the sole cause of hypotension unlikely. Tension pneumothoraces cause hypotension by impeding venous return but will usually also have tracheal deviation. Hemothorax is more likely and a chest thoracostomy tube can both be diagnostic and guide further management. If the chest tube puts out >1500 mL of blood immediately or >200 mL/hour for 3 hours, the patient likely has a significant intrathoracic injury that will require urgent operating room thoracotomy. Removal of blood from the hemothorax will allow the right lung to expand and improve oxygenation and possibly blood pressure. A pelvic binder (A) is necessary for the management of a pelvic fracture as it can reduce the fracture and tamponade bleeding. The mechanism of this patient's injury is inconsistent with causing a pelvic injury. An Emergency Department thoracotomy (C) is only indicated in patients with penetrating trauma who initially have vital signs but lose them either en route to the hospital (with EMS) or in the ED. Thoracotomy is a potentially life saving procedure allowing the physician direct access to the left chest for identification and treatment of cardiac tamponade, cardiac laceration, open cardiac massage and cross-clamping of the aorta. This patient should not be transferred to the operating room (D) without first placing a thoracostomy tube.

A 16-year-old girl presents with knee pain. The patient was dancing when she suddenly developed pain in the right knee and was unable to walk. Physical examination reveals the above image. What management is indicated? Admit for operative repair Closed reduction CT scan of the knee Joint aspiration

Correct Answer ( B ) Explanation: This patient presents with a lateral patellar dislocation and should have closed reduction attempted. Patellar dislocations are common in adolescents. They typically occur with quadriceps contraction combined with valgus and external rotation. It is frequently seen in dancing and in sports where players perform "cut" moves. Lateral dislocations are the most common, are obvious on physical examination and are amenable to closed reduction. In lateral dislocations, the knee is held in moderate flexion and the patella can be seen and palpated lateral to the joint. Closed reduction is performed by gently extending the knee and applying medial pressure to the lateral edge of the patella. Operative repair (A) is often necessary for intercondylar and superior dislocations but not for lateral dislocations. CT scan of the knee (C) is usually unnecessary. Joint aspiration (D) is indicated in the workup of a septic joint.

A 72-year-old man presents after a fall down two stairs. He has a headache and a very small scalp hematoma. Which of the following abnormalities is seen on his CT scan of the brain? Epidural hematoma Subarachnoid hemorrhage Subdural hematoma Subdural hygroma

Correct Answer ( C ) Explanation: A subdural hematoma is a collection of blood between the dura and arachnoid matter. Most commonly this occurs from shear force causing an injury of the bridging veins. Older patients are at higher risk because as the brain atrophies with age, the bridging veins are under more tension. On CT scan, the blood is crescent-shaped and does not cross the midline although it can cross suture lines. Surgical management is determined by the size of the hematoma, the presence of midline shift (typically greater than 5 mm) or neurologic deterioration. An epidural hematoma (A) is bleeding outside of the dura below the skull. A common cause of an epidural hematoma is direct trauma to the temporoparietal region of the head injuring the middle meningeal artery. As a result of arterial bleeding, epidural hematomas progress rapidly and carry significant risk for morbidity and mortality. On CT scan, an epidural hematoma does not cross suture lines. A subarachnoid hemorrhage (B) may result from trauma and on CT scan blood is present within the cerebrospinal fluid and caused by injury to the small subarachnoid vessels. It is the most common abnormality seen on CT scan after head injury. A subdural hygroma (D) appears as a fluid collection in the same location of subdural hematoma on CT scan but has the same density of cerebrospinal fluid. It is unclear whether a tear in the dura allows cerebrospinal fluid to escape into this area or whether it is effusion from an injured vessel.

A 21-year-old man presents after a chemical exposure to the eye. He was emptying a bottle when the liquid splashed into his right eye. The bottle is not available. Irrigation is started with normal saline. Which of the following is the most proper endpoint of irrigation? Absence of fluorescein uptake Normal intraocular pressure pH 7.0-7.2 Two liters

Correct Answer ( C ) Explanation: Chemical exposures to the eye cause severe pain and irritation. Without quick intervention, depending on the nature of the acidic or alkali, burns will occur. Alkali burns are more common than acidic burns and also potentially more dangerous due to the interaction with lipids in the cornea and concern for deep liquefactive necrosis. Treatment of all chemical burns to the eye is with copious irrigation. If only one eye is involved, care is taken to avoid spilling any of the fluid into the opposite eye. Water can be used immediately and then switched to normal saline or lactated Ringers. The use of a topical anesthetic facilitates irrigation and helps to ease the pain of the burn. Initial irrigation should consist of two liters of liquid. However, the goal is to achieve a pH that is as close to neutral at 7.0-7.2 as possible. This is measured with litmus paper. If this is not available, a urine dipstick can also be used for pH ranges. It is more common for alkali burns to require more irrigation than acidic burns. After irrigation, the cornea must be examined with a slit lamp. The absence of fluorescein uptake (A) may not be achieved if a corneal abrasion has occurred during the burn. Therefore this is not an endpoint of therapy. Normal intraocular pressure (B) is seen in most chemical burns because there is no involvement of the intraocular portion of the eye if treated rapidly. Two liters (D) of fluid is the starting volume of irrigation but additional fluid may be necessary if normal pH has not been achieved with two liters.

An 18-year-old hurdler gets tripped-up and falls, contacting the running-track through his hyperextended left wrist. He presents to the ED with dorsolateral wrist pain, erythema, edema, and anatomic snuffbox tenderness. Orthopedic consultation is obtained. Initial AP, lateral, and oblique radiographs are negative for fracture. He is splinted and discharged. His discharge paperwork should contain which of the following directions? Obtain MRI in four weeks if pain persists Obtain primary care medical clearance examination for upcoming orthopedic surgery Obtain repeat radiographs in two weeks Obtain rheumatological consultation for wrist inflammation

Correct Answer ( C ) Explanation: Falling on an outstretched hand (FOOSH injury; "outstretched" refers to wrist hyperextension) is a common cause of scaphoid fracture, which represents the most common fractured carpal bone. The most common fracture site on the scaphoid is the waist. Since the blood supply to the scaphoid enters distally, any > 1 mm fracture or displacement is worrisome for being complicated by osteonecrosis and nonunion. A scaphoid fracture should be suspected any time there is snuff box tenderness. Wrist and scaphoid X-rays may be negative in the setting of acute fracture. Therefore, patients with snuff box tenderness or suspicion for scaphoid fracture should be placed in a thumb spica splint with repeat radiographs or MRI in 10-14 days Waiting four weeks for repeat MRI (A) or plain radiographs is too long. Repeat imaging should occur within 10-14 days. It is unknown if a true fracture is present at this time, as such, surgery (B) would not be arranged at discharge. This wrist erythema and edema is related to trauma, and not to an underlying rheumatologic condition (D)

A 19-year-old man presents after twisting his ankle. When stepping off a curb, he inverted his ankle and now has pain on the lateral malleolus and along the side of the foot. Which of the following is the most appropriate management of the injury shown in the X-ray above? Elastic wrap Hard sole shoe Posterior splint Walking cast boot

Correct Answer ( C ) Explanation: Metatarsal fractures are responsible for one third of foot fractures. Fractures at the base of the metatarsal are uncommon in the first through fourth metatarsals. The isolated fracture at the base of the fifth metatarsal is common and management depends on which form of the two fractures occurs. The Jones fracture is a transverse fracture occurring in the diaphysis of the bone at least 1.5 cm distal to the end of the bone. The Jones fracture tends to displace with further weight bearing. With improper immobilization there is a higher percentage of nonunion of the fracture fragments. Therefore, patients should be immobilized in a posterior splint and given crutches for non-weight bearing ambulation. The pseudo-Jones fracture is an avulsion fracture of the tuberosity at the base of the fifth metatarsal. The injury often occurs as a result of a lateral ankle strain with tension at the attachment of the peroneus brevis tendon. These patients are treated with a compression dressing and weight bearing as tolerated. Providers may elect to give the patients a hard sole shoe. An elastic wrap (A) is often used in the management of an acute sprain. An elastic wrap may be a component of the Jones wrap which includes a bulky dressing to immobilize the foot. A hard sole shoe (B) is sometimes used in the management of the proximal tuberosity fracture or other clinically insignificant fractures of the foot like a phalanx fracture immobilized with buddy tape. A walking cast boot (D) is sometimes used for lower leg fractures like small fractures of the distal fibula.

A 19-year-old man presents after getting tackled in a football game. He complains of pain in the right shoulder. On physical examination, you palpate mobility of the right distal clavicle. His X-ray is shown below. What is the appropriate next step? Figure of eight sling Shoulder MRI Sling and orthopedic follow-up Surgery

Correct Answer ( C ) Explanation: The acromioclavicular (AC) joint is one of three joints making up the shoulder along with the glenohumeral and sternoclavicular joints. The AC joint attaches the lateral portion of the clavicle to the acromion. The AC joint most commonly becomes injured in contact sports after a fall or direct blow to the shoulder. The acromioclavicular and coracoclavicular ligaments are relatively weak and susceptible to injury. Injuries are divided into six types. In type I (most common), the AC joint is sprained and there is no separation of the two bones. In Type II injuries, the joint space is widened with minimal upward displacement of the clavicle. Type III injuries involve complete disruption of the joint and more significant upward displacement. Types IV, V and VI have more significant displacements of the clavicle in other parts of the axis. Type I and II AC joint separations are managed with a sling and orthopedic follow-up within a couple of weeks. Type III is also managed with a sling but more expeditious orthopedic follow-up because of the possible need for surgical management. Figure of eight slings (A) were previously used for stability after shoulder dislocations and AC separations, but outcomes are similar with sling and early range of motion in low-grade separations. A shoulder MRI (B) is not necessary for this patient with a low-grade separation. An MRI is helpful in the diagnosis of a rotator cuff injury which does not have the obvious plain film radiographic changes as an AC separation. Surgery (D) is required in Types IV, V and VI injuries and may be considered in Type III injuries although conservative management in Type III is often effective

A 16-year-old boy presents with left wrist pain after a fall while skateboarding in which he landed on his outstretched hand. He has tenderness and swelling over the dorsum of the wrist on the radial side and proximal thumb in between the abductor pollicis longus and extensor pollicis longus tendons. Wrist radiographs are negative for a fracture. Which of the following is the most appropriate management? Discharge home with anti-inflammatories Emergent orthopedic consult Immobilization in a thumb spica splint Perform an incision and drainage

Correct Answer ( C ) Explanation: The patient has tenderness over the anatomical snuffbox, which is suggestive of an underlying scaphoid fracture. The scaphoid is the most commonly fractured carpal bone. Signs and symptoms include dorsal radial wrist pain, swelling, and tenderness within the anatomic snuffbox. Pain may also be elicited with axial compression of the first metacarpal. Scaphoid fractures can be difficult to detect radiographically. In 15% of the cases, radiographs taken at the time of injury are negative for fracture. A dedicated scaphoid view, with the wrist pronated and in ulnar deviation, may better visualize a fracture. If a scaphoid fracture is suspected and radiographs are negative, the patient should be immobilized in a thumb spica splint and referred for repeat clinical evaluation and radiographs in approximately 7-10 days. Alternatively, MRI has a sensitivity of nearly 100% for detecting radiographically occult scaphoid fractures. Scaphoid fractures have a relatively high incidence of avascular necrosis and nonunion, with a higher risk in fractures at the proximal pole owing to a relatively sparse blood supply. Therefore, patients with scaphoid fractures should be referred to orthopedics for close follow-up. Patients should be seen on an urgent basis, but an emergent orthopedics consult (B) is not necessary for an uncomplicated scaphoid fracture. Incision and drainage (D) is the treatment for an abscess, not a suspected fracture. Discharge home with anti-inflammatories (A) is incorrect as the patient has a suspected scaphoid fracture and should be immobilized and provided with close orthopedic follow-up.

A 17-year-old man presents with chest pain and difficulty breathing after an MVC. His vitals are BP 120/78, HR 123, RR 32, and oxygen saturation 91% on non-rebreather facemask. Physical examination reveals the finding seen in the video above. Chest X-ray shows multiple rib fractures without a pneumothorax. What management is indicated? Chest thoracostomy tube placement Intravenous antibiotics Intubation and mechanical ventilation Thoracotomy

Correct Answer ( C ) Explanation: The patient presents with a flail chest injury evidenced by the paradoxical chest wall movement. and multiple rib fractures seen on X-ray. Flail chest is an uncommon injury in trauma. It occurs when there are three or more adjacent ribs fractured at two different points creating a freely moving segment of ribs. Physical examination reveals a section of the chest that moves paradoxically (in with inspiration and out with exhalation). They are almost always associated with underlying pulmonary contusions. These patients often develop respiratory insufficiency, hypoxemia and atelectasis. Although flail segment should be easily diagnosed by physical examination, it can be confirmed by X-ray or CT scan. Management focuses on supportive care particularly good respiratory care. Intubation is often required to maintain oxygenation and ventilation although patients who do not have respiratory compromise and are awake and alert can be tried on non-invasive positive pressure ventilation first. Operative fixation is often required in these cases. Flail chest is associated with an 8-34% mortality rate. Chest thoracostomy placement (A) is indicated for patients with traumatic pneumothorax but do not have a role in treatment of pulmonary contusions or flail chest. Intravenous antibiotics (B) prophylactically are not indicated. Thoracotomy (D) is indicated for patients with loss of vital signs in the Emergency Department after blunt or penetrating injury or for suspected great vessel injury or pericardial tamponade in hemodynamically unstable patients.

A 16-year-old girl presents with right thumb pain after a fall while skiing. Physical examination reveals pain and swelling of the right thumb. X-ray of the thumb is negative. Valgus stress at the metacarpophalangeal joint results in increased pain and deviation of 40 degrees. What treatment is indicated? CT scan of the thumb Sugar tong splint and urgent referral for surgical management Thumb spica and urgent referral for surgical management Thumb spica for 4 weeks and primary care follow up

Correct Answer ( C ) Explanation: The patient presents with rupture of the ulnar collateral ligament (UCL) and requires immobilization and urgent surgical management. Injury to the UCL was initially described in Scottish gamekeepers (hence Gamekeeper's thumb). The injury was developed through the repetitive motion of twisting the necks of rabbits. Today, the injury is most commonly seen in skiers who receive the injury during a fall while holding a ski pole. The mechanism of injury is forced abduction of the thumb resulting in a tear of the UCL near its insertion at the proximal phalanx. Physical examination reveals swelling and tenderness along the ulnar surface of the thumb and difficulty with making a pinching motion. Valgus stress of the UCL can differentiate a partial tear from a complete rupture of the ligament. Stress should be applied to the metacarpophalangeal (MCP) joint in full extension and at 30 degrees of flexion. If there is more than 35 degrees of joint laxity or 15 degrees of laxity beyond the unaffected thumb, a complete UCL rupture should be suspected. Both partial tears and complete ruptures should be placed in a thumb spica splint. Partial tears typically will recover completely with immobilization whereas complete ruptures invariably need surgical repair. A CT scan of the thumb (A) is not necessary for the diagnosis of a UCL rupture. Sugar tong splinting (B) does not immobilize the first MCP or first IP joints. A thumb spica and follow up with primary care (D) is appropriate for a partial tear but not for a complete rupture.

A 16-year-old boy presents with headache, nausea and vomiting after a fall. The patient fell from a tree with a brief loss of consciousness. A minute later he was awake and only complained of a mild headache but he has since worsened. A non-contrast CT scan is shown above. What management is indicated? Admit for repeat non-contrast head CT Discharge with follow-up with neurology Emergent neurosurgical consultation Obtain MRI of the brain

Correct Answer ( C ) Explanation: This patient presents with a classic story for an epidural hematoma confirmed by imaging and requires immediate evaluation for prompt surgical hematoma evacuation. Epidural hematoma results from trauma leading to rupture of the middle meningeal artery. Classically, patients report a brief loss of consciousness followed by a lucid period and then a precipitous decline. Non-contrast head CT (NCHCT) scan is diagnostic for this disease. A NCHCT will show a biconvex or lens shaped hyperdense area in the temporal region of the side of trauma in the epidural space. Depending on the size of the hematoma, there may also be midline shift and compression of the ventricles. As the hematoma continues to expand, it can cause uncal herniation leading to brain compromise and respiratory arrest. In an obtunded patient, the physical examination may reveal a unilateral dilated and unresponsive pupil (aka "blown pupil") on the side of the hematoma. Emergency management of an epidural hematoma should focus on airway protection if necessary and emergent neurosurgical consultation for hematoma evacuation. In patients with signs of increased intracranial pressure or impending herniation, hyperventilation should be initiated followed by osmotic agents like mannitol. Stable patients with traumatic subdural hematoma or subarachnoid hemorrhage may be admitted for serial NCHCT (A) after neurosurgical evaluation. These intracranial bleeds may not increase in size and may not require surgical intervention. However, this does not apply to epidural hematomas. In the absence of an intracranial hematoma, the patient's symptoms may be consistent with a concussion, which would require outpatient neurology follow up (B). MRI of the brain (D) is not necessary in epidural hematoma as the NCHCT is diagnostic.

A 42-year old woman presents with right leg pain. She states she stepped off a curb and felt something pop above her heel. On physical examination she has limited plantar flexion. Which physical examination test will help make the diagnosis? Lachman's test McMurray's test Posterior sag sign test Thompson test

Correct Answer ( D ) Explanation: Achilles tendon rupture classically happens in the setting of sudden plantar flexion against resistance. This occurs when jumping (taking a jump shot in basketball) or stepping into a hole. Patients often report hearing a pop when the injury occurs. Patients complain of severe pain in the lower calf. Patients with a rupture should not be able to stand on their toes on the affected side. On physical examination, there is often a palpable defect in the middle portion of the tendon. However, this is not always obvious and additional modalities like point-of-care ultrasound can be employed. The Thompson test is performed by having the patient lie prone on the stretcher with the distal portion of the leg extending past the stretcher. The provider squeezes the mid portion of the calf which should cause plantar flexion of the foot. With a complete rupture, plantar flexion is absent and the test is considered positive. Partial ruptures may have a negative test. In the ED, patients with Achilles ruptures are placed in a posterior splint with 20 to 30 degrees of plantar flexion (equinas position). Lachman's test (A) is performed to evaluate the integrity of the anterior cruciate ligament. The knee is flexed to 20 or 30 degrees and with one hand the examiner stabilizes the thigh while with the other hand pulls the tibia anterior trying to identify inappropriate tibial excursion. McMurray's test (B) is used to evaluate for a meniscal tear. The patient lies supine with the knee hyperflexed. With one hand the examiner holds the foot and with the other grasps the knee. At the same time, the examiner flexes and extends the knee while internally and externally rotating the tibia on the femur. The test is considered positive if clicking is heard or the knee locks. The posterior sag sign test (C) is performed to evaluate the posterior cruciate ligament. In this test, the patient lies supine on the bed with a pillow under the distal thigh and the heel resting on the bed. The knee is then flexed to the point of maximal relaxation (45 to 90 degrees) and the tibia is observed for posterior sag. If the tibia sags backwards the test is considered positive.

A 34-year-old man presents to the ED after he was injured in an explosion. The patient was in close proximity to the explosive device and was thrown into a nearby wall, causing a clavicular fracture. The clavicular fracture represents which category of blast injury? Primary blast injury Quaternary blast injury Secondary blast injury Tertiary blast injury

Correct Answer ( D ) Explanation: Blast injuries are divided into five categories, primary, secondary, tertiary, quaternary, and quinary injuries. Tertiary injuries are created when the blast wave propels victims' bodies into objects or large objects strike the body. Typical tertiary blast injuries include crush injuries and blunt trauma. The fractured clavicle in this scenario is classified as a tertiary blast injury. A primary blast injury (A) results from the direct effect produced by contact from the blast shockwave with the body. This creates shear and stress forces on tissues. Typical injuries include tympanic membrane rupture, blast lung, ocular injuries, and concussions. A secondary blast injury (C) is produced by impact of primary fragments (pieces of the exploding device) or secondary fragments (fragments from the surrounding environment). Typical injuries include penetrating trauma, amputations, or lacerations. Quaternary blast injuries (B) include burns, inhalation injury, exposure to toxic substances, and injury from an environmental contamination that was created as a result of the explosive device. Quinary injuries result from additives such as bacteria or radiation.

You obtain a radiograph of a patient who was in a MVC. His GCS is 15. While being observed in the ED, the patient requests increasing doses of pain medication and is complaining of a deep, burning, unrelenting pain to his left lower extremity. He also states that he now feels tingling in his calf. Which of the following statements is correct regarding this patient's condition? Application of a compression bandage will help reduce swelling and limit muscle necrosis Elevating the affected limb above 20 degrees will help to limit myonecrosis Fasciotomy is indicated with compartment pressure of 20 mm Hg The condition is most frequently associated with fractures of the tibia The first signs of the diagnosis usually begins with pallor and loss of pulses and then progresses to pain and paresthesias

Correct Answer ( D ) Explanation: Compartment syndrome is a serious emergency complication that should be considered whenever pain and paresthesias occur in an extremity after a fracture within an enclosed osseofascial space. It is caused by increased pressure within the compartment space that prevents adequate tissue perfusion. Compartment syndrome is most commonly associated with closed long-bone fractures of the tibia, but it can occur with isolated soft tissue trauma and even in open fractures. It has been described in a variety of situations such as prolonged procedures in the lithotomy position, the tuck position (knees tucked to chest for lumbar surgery), bedridden patients, from a spontaneous hemorrhage, and even the application of excessive traction in the reduction of a fracture. Compression dressings (A) and external wrappings should be avoided; increased compression will worsen perfusion. Elevating the limb (B) results in a reduction in the local arteriovenous gradient and may be counterproductive and exacerbate compartment syndrome. It is best to keep the extremity level or slightly elevated (<10 degrees). Pressures < 30 mm Hg (C) generally do not produce compartment syndrome. The best measure of adequate limb perfusion, however, is not absolute compartment pressure but rather the differential between diastolic blood pressure and absolute compartment pressure. A pressure differential <30 mm Hg is considered by most to be an indication for emergent fasciotomy. However, with strong clinical suspicion, a fasciotomy may be required at any pressure differential as this is largely a clinical diagnosis. The initial presentation of compartment syndrome (E) usually begins with pain with passive stretching of the muscle groups, paresthesias with decreased sensation, and pain that is out of proportion to exam. Pallor and the loss of pulses are late and ominous findings.

Which of the following should be used as the best guide to adequate fluid resuscitation in a severely burned adult patient? Normal central venous pressure Normalization of heart rate and blood pressure Parkland formula Urine output of 0.5 to 1 mL/kg/h

Correct Answer ( D ) Explanation: Fluid resuscitation is one of the most important aspects in the care of burn patients. Prior to WWII, many burn patients died from hypovolemic shock from under-resuscitation. Moreover, a study of transferred burn patients found that smaller burns tended to be overestimated and over-resuscitated, whereas larger burns tended to be underestimated and under-resuscitated. Therefore, it is critical to appropriately monitor the fluid status of a burn patient to optimize outcome. This can be achieved by careful observation with reassessment of clinical signs of tissue turgor, and, most importantly, urinary output. The target urine output in an adult is 0.5 to 1 mL/kg/h; in children, 1 to 2 mL/kg/h. Although normalization of heart rate and blood pressure (B) and a normal central venous pressure (> 10 mm Hg) (A) are good predictors of adequate tissue perfusion, they are not as reliable as urine output. The Parkland formula (C) is the most commonly used fluid resuscitation formula in the burn patient. However, objective measurements of normovolemia supersede the Parkland formula.

A 25-year-old man presents to the ED after being stabbed in his chest. Which of the following is the most likely injured chamber of the heart? Left atrium Left ventricle Right atrium Right ventricle

Correct Answer ( D ) Explanation: Penetrating cardiac injuries are one of the leading causes of death in the setting of urban violence. The initial laceration can result in either exsanguination if the cardiac lesion communicates with the pleural cavity or cardiac tamponade if hemorrhage is contained within the pericardium. The right ventricle is the most commonly injured chamber of the heart (43% of incidents) due to its anatomic location. The left ventricle (B) or right atrium (C) is affected 20% of the time. About 33% of penetrating cardiac wounds involve multiple chambers. More than one wound to the heart is associated with a worse prognosis. The left atrium (A) is least exposed and injured least frequently.

A 40-year-old carpenter presents to the ED via EMS with altered mental status and visible chest wall bruising after a ceiling collapsed on him. The patient could not get out from under the debris until EMS arrived 20 minutes later. Vital signs are BP 100/55 mm Hg, HR 116, RR 24, oxygen saturation 93% on room air. His GCS is 9. The patient is placed on a cardiac monitor, oxygen therapy is initiated, and large-bore intravenous access is obtained. A portable chest x-ray is obtained with findings shown above. Which of the following is the most appropriate next step in management? CT scan of the head, cervical spine, chest, abdomen, and pelvis Diagnostic peritoneal lavage (DPL) Rapid sequence intubation Tube thoracostomy

Correct Answer ( D ) Explanation: The chest x-ray reveals a large left pneumothorax. The lung is bunched up in the region of the hilum, and there are no lung markings in most of the left lung field. For traumatic pneumothoraces, a large (36 or 40 French) chest tube should be placed to decompress the pneumothorax and drain any concomitant hemothorax. Given the altered mental status and blunt mechanism of injury, this patient may benefit from whole-body imaging (A); however, the patient's airway, breathing, and circulation must first be addressed. Patients with a GCS of 8 or below generally require endotracheal intubation. This patient's GCS is 9, but he ultimately may require intubation (C). The most immediate need is for management of his pneumothorax. Moreover, as intubation leads to an increase in positive intrathoracic pressure, there is the potential to convert a relatively stable traumatic pneumothorax into an unstable tension pneumothorax. Thus tube thoracostomy should precede intubation once a pneumothorax (especially a large one) is identified on chest X-ray. A diagnostic peritoneal lavage (B) can be used to detect free fluid (blood) in the abdomen but is rarely performed in the trauma setting since bedside ultrasound is widely available.

A 35-year-old man presents to the ED after injuring himself while playing basketball. He states he felt a pop in his right lower extremity while jumping for a rebound. He can ambulate, but with a limp. On exam, squeezing his right calf elicits no plantar flexion of his foot. What is the appropriate disposition for this injury? Admission to hospital for serial compartment pressure measurements Elastic wrap, crutches, weight-bearing as tolerated, and orthopedic follow-up within seven days Splint in dorsiflexion, keep non-weight-bearing, orthopedic follow-up Splint in plantarflexion, keep non-weight-bearing, orthopedic follow-up

Correct Answer ( D ) Explanation: The patient has an Achilles tendon rupture suggested by the absence of plantar flexion with squeezing of the calf (Thompson test). This injury is most common in middle-aged recreational athletes. Patients often note a "popping" sensation followed by acute weakness and the inability to continue activity. A defect may be palpable on examination of the distal Achilles tendon. Management in the ED includes immobilization of the extremity in a short-leg splint, keeping the foot in plantarflexion. The patient should be non-weight-bearing until seen by an orthopedic surgeon. Definitive repair is surgical. This injury is not at risk to develop compartment syndrome (A). An elastic wrap (B) alone is insufficient. In an effort to maintain approximation of the proximal and distal aspects of the ruptured tendon, the foot should be splinted in plantar, not dorsiflexion (C)

A patient presents with facial trauma and dental pain. Examination reveals a broken eighth tooth with a pink spot on the inferior surface. What type of injury is this? Dental subluxation Ellis class I Ellis class II Ellis class III

Correct Answer ( D ) Explanation: The patient presents with an Ellis class III dental fracture evidenced by the pink spot indicating pulp exposure. The structure of a tooth involves three main layers: the enamel (outer most layer), the dentin (middle layer) and the pulp (inner most layer). The tooth is held in the alveolar socket by cementum and the periodontal ligaments. Dental fractures are classified by the Ellis system in order to improve communication by providers. An Ellis class III fracture involves all three layers and the patient will typically have severe pain (unless the neurovascular supply is disrupted). These injuries should be emergently referred to oral surgery or dental as they are at high risk for complications. An Ellis class I fracture (B) involves fracture of the enamel only and is typically painless. There is no emergent treatment required for this injury. An Ellis class II fracture (C) involves the enamel and the dentin. Patients experience sensitivity to hot or cold and examination reveals exposed yellow dentin. Treatment involves covering the tooth with calcium hydroxide or foil and urgent referral to oral surgery or dental. A dental subluxation (A) is an injury to the attachment apparatus of the tooth without radiographic evidence of dislodgement. Patients will present with loosening of the tooth.

A 13-year-old boy with no past medical history presents to urgent care with a headache three days after a closed head injury. The patient states that he stood up from kneeling and hit the top of his head on a wood cabinet. There was no loss of consciousness or seizure activity. In addition to the headache, he complains of difficulty concentrating at school and dizziness. His physical examination is unremarkable. What management is indicated? CT scan of the head with contrast CT scan of the head without contrast MRI of the brain Referral to primary care physician

Correct Answer ( D ) Explanation: The patient presents with minor head trauma and complaints consistent with a concussion and should have follow-up arranged with their primary care provider or concussion specialist. A concussion is a minor traumatic brain injury (TBI) that is often seen in MVCs and collision sports (football, hockey). It is typically caused by a rotational injury or an acceleration-deceleration injury. Patients will present with a number of non-specific symptoms including headaches, dizziness, confusion, amnesia, difficulty concentrating, and blurry vision but do not have focal neurologic findings. Despite the absence of severe intracranial injury, patients can have chronic and debilitating symptoms from concussions. Neurology referral is recommended, as patients should have functional testing and tracking of their symptoms for resolution. It is vital to council patients to avoid contact sports or activities that increase the risk of recurrent injury as these patients are at risk for more severe injury with a second impact. In the absence of focal neurologic findings, absence of antiplatelet or anticoagulant use, and minor trauma, imaging is not needed (A, B, C).

A 42-year-old man presents to the ED seven days after tripping, falling at a construction site, and striking his head. The fall was associated with a five second loss of consciousness. He reports increased forgetfulness and difficulty sleeping. Otherwise, his exam is normal. What is the most appropriate next step in management? Admit to the hospital for observation Obtain a brain MRI Obtain a head CT scan Outpatient workup

Correct Answer ( D ) Explanation: The patient's clinical presentation is consistent with postconcussive syndrome. After mild traumatic brain injury, most symptoms usually resolve within six hours. However, up to 25% of individuals suffer from postconcussive syndrome. The condition can cause a variety of symptoms such as headache, difficulty concentrating, and irritability. Appropriate management includes non-narcotic analgesics for headache and outpatient workup with neuropsychiatric testing, and possibly advanced imaging. Given that the patient's injury was sustained seven days ago, he does not require clinical observation (A). Some patients with head trauma who have no obvious injury on the initial CT scan should be considered for clinical observation. These include extremes of age and those on anticoagulants. An MRI (B) is more sensitive for identifying concussion-related findings, but this information would have no bearing on the acute management. A clinically significant injury is not likely to be identified on a head CT scan (C) from an injury that occurred seven days prior.

Which of the following organs is the most commonly injured in adult blunt abdominal trauma? Bladder Intestine Liver Spleen

Correct Answer ( D ) Explanation: The spleen is the most common organ injured in adults in blunt abdominal trauma. Most blunt abdominal trauma results from motor vehicle collisions. Many splenic injuries, including high-grade lacerations, may be managed conservatively in the hemodynamically stable patient without other significant intraabdominal injuries. The liver (C) is the second most commonly injured intraabdominal organ in blunt abdominal trauma and most commonly injured in all abdominal trauma. Similarly to splenic injuries, liver lacerations may also be managed conservatively in the stable patient. Intestines (B) are the most common hollow viscous organ injured, but the frequency of this follows both spleen and liver. When seatbelts in rear seats of cars only had lap belts, the sudden deceleration caused compression of the bowel between the external belt and the spinal column causing injury. Bladder (A) injuries are less common and frequently associated with significant pelvic fractures.

A 3-year-old boy presents with a burn at the commissure of his lip. The child bit into an appliance electrical cord that was plugged in. Mom reports bleeding occurred at home and has now resolved. The child is in no distress at the time of evaluation. What is the most serious complication? Cardiac dysrhythmia Cosmetic disruption Damage to developing dentition Delayed labial artery bleeding

Correct Answer ( D ) Explanation: This burn caused by sucking or biting on an electric cord is the most common type of electrical burn in children under age 4. Typically the current forms an arc causing localized burn sometimes involving the orbicularis oris. The labial artery is anatomically located near the commissure of the lip. When the eschar formed from the burn separates, delayed labial artery bleeding may occur. Patients will need observation and often referral to a burn center for these injuries. Cardiac dysrhythmia (A) is unlikely in this type of injury. Given the mechanism of biting or sucking, the current typically forms a local arc and does not travel through the region of the myocardium. Cosmetic disruption (B) is possible after this injury particularly if muscle damage to the orbicularis oris has occurred. Patients should follow up with either a plastic or oral surgeon familiar with this injury. The burn may cause damage to developing dentition (C) as well as to the facial bones. Once the danger from delayed bleeding passes, surgeons with expertise in electrical injuries can follow the child and perform reparative surgery if necessary.

33-year-old man presents to the ED complaining of pain to his left eye after he was hit by a football. He reports decreased vision and photophobia. Which of the following statements is correct regarding his injury? A teardrop shaped pupil is a common finding on physical exam Fundoscopic exam will reveal a "blood and thunder" appearance of the retina Placement of an eye shield is contraindicated secondary to the risk of developing a pseudomonal infection Prevention of further hemorrhage is the principal goal of treatment Rebleeding typically occurs seven days after the initial bleed

Correct Answer ( D ) Explanation: This patient has a hyphema of the left eye. This results from ocular trauma that causes bleeding into the anterior chamber. The bleeding originates from vessels in the ciliary body or iris. The blood tends to layer over time, and left undisturbed, will form a visible meniscus when the patient sits upright. Patients typically complain of pain, photophobia, and possibly blurred vision secondary to obstructing cells. Intraocular pressure should be measured because acute glaucoma may be caused by RBC clogging of the trabecular meshwork with impedance to aqueous outflow. Prevention of further hemorrhage is the principal treatment goal. Most rebleeding occurs within the first 72 hours and, when present, tends to be more extensive than the initial event. Patients should be instructed to rest in the supine position with their head slightly elevated. A teardrop shaped pupil (A) is suggestive of a ruptured globe. Though it can occur concomitantly with a hyphema, it is uncommon. The "blood and thunder" finding on fundoscopy (B) is characteristic of central retinal vein occlusion. An eye shield (C) is recommend in the setting of a hyphema to prevent further trauma from manipulation. Eye shields are contraindicated in patients with corneal abrasions due to the increased risk of a pseudomonal infection. Rebleeding (E), the most common complication of a hyphema, generally will occur within the first 72 hours from the time of injury. Patients should avoid antiplatelet and anticoagulant medications during the healing phase.

A 46-year-old woman presents as a restrained driver who required prolonged extraction from a high-speed MVC. In the ED, her HR is 125, BP 80/50, RR 22, and pulse oximetry 97% on room air. Exam reveals mobility of her right iliac crest and a large volume of fluid in Morrison's pouch on the FAST exam. A chest radiograph is normal. Two large-bore IVs are placed, fluid resuscitation is initiated, and blood products are ordered. Which of the following is the most appropriate next step in the patient's management? Activation of the interventional radiology suite CT scan of the abdomen and pelvis Immediate placement of an external fixation device by an orthopedic surgeon Transfer to the operating room for laparotomy

Correct Answer ( D ) Explanation: This patient has sustained blunt abdominal trauma, a management challenge to both emergency medicine physicians and trauma surgeons. Motor vehicle collisions are the most common cause of blunt abdominal trauma. The spleen is the solid organ most likely to be injured. In patients who are hemodynamically unstable, the source of bleeding is usually from the abdomen, pelvis, or thorax. Chest and pelvis radiographs are performed to evaluate for injury to the thorax and pelvis, respectively. A FAST ultrasound is highly sensitive for intraperitoneal bleeding. This patient is hypotensive and has a positive FAST ultrasound. Consequently, the next most appropriate step is exploratory laparotomy. Interventional radiology (A) is a critical component in the management of pelvic injures with suspected arterial bleeding, and hemodynamically stable liver or spleen injuries that can benefit from angioembolization. However, when hypotension is present with blunt abdominal trauma and free intraperitoneal fluid, immediate exploratory laparotomy is needed. In some trauma centers, arterial embolization can be performed in the operating room concurrent with exploratory laparotomy. CT scan of the abdomen and pelvis (B) is appropriate in the normotensive patient or the hypotensive patient that responds to fluid resuscitation and has a negative FAST exam. Placement of an external fixation device (C) is necessary for patients with disruption of the pelvis, most commonly from an open-book fracture. However, this procedure should not delay the transfer of the patient to the operating room and can be performed alongside the trauma surgeon.

An 83-year-old woman presents with left hip pain after a mechanical fall. Her examination is unremarkable except for pain with range of motion of the left hip. Hip and pelvis X-rays are unremarkable. The patient continues to be unable to ambulate. What management is indicated? Bone scan of the hip CT scan of the hip Discharge home with a walker MRI of the hip and pelvis

Correct Answer ( D ) Explanation: This patient's presentation is concerning for an occult hip fracture, which can be diagnosed with an MRI. Hip fracture is more common in older patients and more common in women than in men. Patients with large hip fractures often present with an externally rotated and shortened lower extremity. However, some patients with hip fractures that are small will not have these findings. Patients will typically have pain with straight leg raise or log roll of the affected leg. In patients with negative X-rays, ambulation should be attempted. However, if the patient is unable to ambulate or can only ambulate with marked difficulty, further imaging should be pursued to rule out an occult hip fracture. MRI of the hip is the diagnostic imaging modality of choice because of its high sensitivity and specificity as well as an accuracy of 100%. A bone scan of the hip (A) is sensitive and specific only when performed in the non-acute setting. CT scan of the hip (B) is commonly ordered because of its availability but has a 66% misdiagnosis rate. Patients who remain unable to ambulate after negative X-rays should not be discharged home (C) as missing an occult fracture can lead to increased morbidity.

A 28-year-old woman presents with intense pain, tearing, and ocular foreign body sensation in both eyes that began yesterday. She denies any trauma, past medical problems, or contact lens use and reports that she works as ski patrol for a local resort. Her physical examination is significant for bilateral decreased visual acuity, injected conjunctiva, and diffuse punctuate corneal lesions with a discrete superior border. Her pupils are equal, round, and reactive to light. Given this patient's clinical presentation, which of the following is the most likely diagnosis? Epidemic keratoconjunctivitis Episcleritis Iritis Optic neuritis Ultraviolet keratitis

Correct Answer ( E ) Explanation: This patient has ultraviolet keratitis, also known as snow blindness. This is a radiation burn that occurs when an individual comes in close contact with an ultraviolet-ray-containing light source. It may be caused by sun lamps, tanning booths, or exposure to intense reflected sunlight, particularly in high-altitude environments. Patients usually present about 6-12 hours post-exposure, complaining of eye pain, blepharospasm, tearing, photophobia, and foreign body sensation. Fluorescein staining reveals superficial punctate epithelial surface irregularities, which usually cover the entire surface of the cornea. This condition is usually referred to as superficial punctate keratitis (SPK). If the patient's eyelid was partially closed during the exposure, a well-demarcated line may distinguish normal from damaged corneal epithelium. Treatment consists of topical NSAIDs or oral pain medication while the use of antibiotics or cycloplegics is controversial. Ophthalmic antibiotics have not been shown to improve healing, but in high risk individuals (chronic contact lens users or immunocompromised individuals) they can be beneficial. Some sources advocate cycloplegics to relieve photophobia, but the pupillary dilation can be distressing to patients. While topical anesthetics may be required during the examination, they should not be prescribed for further use due to an increased risk of corneal ulceration. Epidemic keratoconjunctivitis (EKC) (A) is a highly contagious adenovirus infection, which is distinguished from typical viral conjunctivitis by involvement of the conjunctiva and cornea. A painful keratitis involving the central cornea may develop after one week. Episcleritis (B) is a common, benign inflammatory condition of the episclera and most often occurs in young adults. It is usually idiopathic but may be associated with systemic conditions such as diabetes mellitus, systemic lupus erythematosus, rheumatoid arthritis, and inflammatory bowel disease. Patients typically complain of a foreign body sensation, mild pain, photophobia, and lacrimation. The condition is usually self-limiting. Iritis (C), also known as anterior uveitis, is an inflammation of the anterior portions of the uvea (iris, ciliary body, and choroid). It is often idiopathic but may be associated with systemic diseases such as rheumatoid arthritis, sarcoidosis, Reiter syndrome, and tuberculosis. Patients report pain, tearing, and photophobia, and may have decreased visual acuity. Exam typically reveals ciliary flush with a miotic pupil that may be distorted due to the formation of synechiae (iris adheres to either the cornea [anterior synechia] or lens [posterior synechia]). The classic finding on slit lamp exam are cells (sign of inflammation or bleeding) and flare (elevated concentration of plasma proteins from inflamed, leaking intraocular blood vessels) in the anterior chamber. Optic neuritis (D) presents with sudden or rapidly progressing monocular vision loss in patients younger than 50 years. There may be pain on movement of the globe, and the pupillary light response is diminished in the affected eye (afferent pupillary defect). Although optic neuritis is often an isolated condition, it is associated with demyelinating disorders such as multiple sclerosis.

Which of the following is correct when estimating an adult burn patient's percent of total body surface area (TBSA) affected? AEach arm is approximately 18% BThe area covered by the patient's palm is approximately 5% CThe entire head is approximately 18% DThe perineum is approximately 1%

In adults, the head is estimated to be 9% (C), the anterior and posterior torso are each 18%, the arms are 9% each (A) and the legs are 18% each. The area covered by the individual's palm (B) is also approximately 1%. Adjustments to the "Rule of Nines" are made for pediatric patients to account for their relatively larger head and smaller legs. In pediatric patients, the head is estimated to be 18%, the anterior and posterior torso are each 18%, the arms are 9% each and the legs are 14% each.


Kaugnay na mga set ng pag-aaral

Intermediate Financial Management Exam One

View Set

Section 7: Fixed-Period (P) Systems

View Set

Psych- Ch. 10: Emotion and Motivation

View Set

Bio - study question on Lecture 8

View Set

Ch. 9 SCM: Managing Inv in the Supply Chain

View Set

Drug Laws and Regulations, Relevant CAMS, CAM

View Set

REVIEW OF PATHOLOGY Questions Chapter 15. The Lung

View Set